JIPMER Medical Entrance Examination 2019 Question Paper With Answer Key

JIPMER SOLVED PAPER-2019

Physics

1. Find the current in the 8Ω resistance in the given circuit.

(a) 2 A

(b) 3 A

(c) 4 A

(d) 5 A

Answer: (b)

2. In the given circuit find the heat produced between A and B (given C = 1 μF)

(a) 50 μJ

(b) 60 μJ

(c) 70 μJ

(d) 100 μJ

Answer: (a)

3. In the given circuit, if power rating of heater is 1000 Watt at 100 Volt, find the resistance R in figure so that heater produces power of 62.5 W.

(a) 5 Ω

(b) 7 Ω

(c) 10 Ω

(d) 8 Ω

Answer: (a)

4. Ba-122 has half life of 2 min. Experiment has to be done using Ba-122 and it takes 10 min to set up the experiment. If initially 80 g of Ba-122 was taken, how much Ba was left when experiment was started?

(a) 2.5 g

(b) 5 g

(c) 10 g

(d) 20 g

Answer: (b)

5. Find the value of θ in the given diagram.

(a) sin1(2/√3)

(b) sin1(1/√3)

(c) sin1(1/2)

(d) sin1(1/√2)

Answer: (b)

6. Find the position of final image from first lens. Given focal length of each lens is 10 cm.

(a) 40 cm

(b) 50 cm

(c) 45 cm

(d) 55 cm

Answer: (b)

7. If velocity of light in air is c, what will be the velocity of light in a medium of refractive index 1.4?

(a) 2.1 × 108 m/s

(b) 1.4 × 108 m/s

(c) 2.8 × 108 m/s

(d) None of these

Answer: (a)

8. If ratio of terminal velocity of two drops falling in air is 3 : 4, then what is the ratio of their surface area?

(a) 2/3

(b) 3/4

(c) 4/3

(d) 3/2

Answer: (b)

9. If frequency (v) of light is 5 × 106 Hz and speed of light in air is 3 × 108 m/s. Find the ratio of wavelength of light in a medium of refractive index ‘2’ to air

(a) 1/2

(b) 3

(c) 3/2

(d) 2

Answer: (a)

10. Find the maximum radius of circle so that the block can complete the circular motion.

(a) 5 cm

(b) 3 cm

(c) 2 cm

(d) 4 cm

Answer: (c)

11. Find the height of liquid in capillary tube, if surface tension of liquid = S, radius of capillary tube = r and acceleration due to gravity = g.

(a)

(b)

(c)

(d) None of these

Answer: (a)

12. Following circuit will act as

(a) NOR gate

(b) NAND gate

(c) AND gate

(d) OR gate

Answer: (a)

13. The rate of decrease of kinetic energy is 9.6 J/s. Find the magnitude of force acting on particle when it’s speed is 3 m/s.

(a) 3.2 N

(b) 4.8 N

(c) 2.4 N

(d) 5.6 N

Answer: (a)

14. Maximum acceleration in SHM

(a) ω2A

(b) ω2A/2

(c) ω2A2

(d) 0

Answer: (a)

15. If frequencies are (v – 1), (v + 1), then find beats.

(a) 2

(b) 1

(c) 2

(d) 4

Answer: (a)

16. If mass (M), distance (L) and time (T) are fundamental quantities, then find dimension of torque.

(a) [ML2T2]

(b) [MLT2]

(c) [MLT]

(d) [ML2T]

Answer: (a)

17. Find the correct diagram of electric lines of forces for negative charge.

(a)

(b)

(c)

(d)

Answer: (c)

18. A circuit contain two resistance R1 and R2 in series. Find the ratio of input voltage to voltage of R2.

(a)

(b)

(c)

(d)

Answer: (b)

19. Find the current drawn from the battery just after the switch is closed.

(a) E/2R

(b) 2E/R

(c) E/R

(d) 2E/3R

Answer: (c)

20. Find the average velocity when a particle complete the circle of radius 1m in 10 sec.

(a) 2 m/s

(b) 3.14 m/s

(c) 6.28 m/s

(d) zero

Answer: (d)

21. Speeds of a particle at 3rd and 8th sec are 20 m/s and zero respectively, then average acceleration between 3rd and 8th sec will be

(a) 3 m/s2

(b) 4 m/s2

(c) 5 m/s2

(d) 6 m/s2

Answer: (b)

22. If object starts from rest and covers angle of 60 rad in 10 sec in circular motion magnitude of angular acceleration will be

(a) 1.2 rad/sec2

(b) 1.5 rad/sec2

(c) 2 rad/sec2

(d) 2.5 rad/sec2

Answer: (a)

23. One object of mass 20 kg is moving with speed 10 m/s in west direction another object of mass 10 kg is moving with 15 m/s in North direction. Both collide and stick together. Choose the correct alternative.

(a) There K.E. is conserved as it is inelastic collision

(b) There K.E. is conserved on it is elastic collision

(c) There momentum is conserved as it is inelastic collision

(d) There momentum is conserved as it is elastic collision

Answer: (c)

24. One rod of length 2 m and thermal conductivity 50 unity is attached to another rod of length 1 m and thermal conductivity 100 unit. Temperature of free ends are 70°C and 50°C respectively. Then temperature of junction point will be

(a) 60°C

(b) 54°C

(c) 64°C

(d) 68°C

Answer: (b)

25. If three objects are thrown with velocity then find the ratio of their maximum heights.

(a) 2 : 6 : 1

(b) 2 : 5 : 3

(c) 2 : 4 : 7

(d) 3 : 7 : 4

Answer: (a)

26. Find the distance between two consecutive nodes, if for a string T = 0.25 N, μ = 1 mg/m f = 250 Hz.

(a) 2m

(b) 1m

(c) 3m

(d) 1.5m

Answer: (b)

27. If escape velocity on earth surface is 11.1 km/h, then find the escape velocity on moon surface. If mass of moon is 1/81 times of mass of earth and radius of moon is 1/4 times radius of earth.

(a) 2.46 km/h

(b) 3.46 km/h

(c) 4.4 km/h

(d) None of these

Answer: (a)

28. Two disc having mass ratio 1/2 and diameter ratio 2/1 then find ratio of moment of inertia.

(a) 2 : 1

(b) 1 : 1

(c) 1 : 2

(d) 2 : 3

Answer: (a)

29. For a battery the electromotive force is 1.5 V. Terminal voltage is 1.25 V. Power supplied to external resistance is 2.5 W. Find the internal resistance of battery.

(a) 0.125 Ω

(b) 0.25 Ω

(c) 0.5 Ω

(d) 0.1 Ω

Answer: (a)

30. A projectile of mass 1 kg is projected with a speed of 10 m/s at an angle of 60° from the horizontal when projectile is at its highest point, its magnitude of angular momentum (about point of projection)

(a)

(b)

(c) 75 Nm/s

(d) 150 Nm/s

Answer: (b)

31. Two wire of same material having radius in ratio 2 : 1 and lengths in ratio 1 : 2. If same force is applied on them, then ratio of their change in length will be

(a) 1 : 1

(b) 1 : 2

(c) 1 : 4

(d) 1 : 8

Answer: (d)

32. If frequency of a photon is 6 × 1014 Hz, then find its wavelength.

[Take, speed of light, c = 3 × 108 m/s]

(a) 500 Å

(b) 500 nm

(c) 200 Å

(d) 200 nm

Answer: (b)

33. Find resonance frequency in the given circuit

(a) 1/√LC

(b) 2/√LC

(c) 1/2√LC

(d) 4/√LC

Answer: (a)

34. If a ball of mass 0.1 kg hits the ground from the height of 20 m and bounce back to the same height, then find out the force exerted on the ball if the time of impact is 0.04 sec.

(Take, g = 10 m/s2)

(a)

(b)

(c)

(d)

Answer: (a)

35. A galvanometer of 50 Ω resistance has 25 divisions. A current of 4 × 104 A gives a deflection of one division. To convert this galvanometer into a voltmeter having a range of 25 V, it should be connected with a resistance of

(a) 2500 Ω as a shunt

(b) 245 Ω as a shunt

(c) 2550 Ω in series

(d) 2450 Ω in series

Answer: (d)

36. A man applying force of 20 N on an object at an angle of 60° from the horizontal. If object moved by 20 m in horizontal direction, then work done by the force on the object is

(a) 300 J

(b) 400 J

(c) 100 J

(d) 200 J

Answer: (d)

37. What is dimensions of energy in terms of linear momentum (P), area (A) and time (T)

(a) [P1A1T1]

(b) [P2A2T1]

(c) [P1A1/2T1]

(d) [P1/2A1/2T1]

Answer: (c)

38. Ratio of charge on positron to mass of positron is approximately

(a) +2 × 1011

(b) +5 × 1012

(c) −2 × 1011

(d) −5 × 1011

Answer: (a)

39. In the given circuit, find voltage across 12 Ω

(a) 12 Volt

(b) 36 Volt

(c) 72 Volt

(d) 48 Volt

Answer: (d)

40. Potential difference is given as

V(x) = −x2y volt. Find electric field at a point (1, 2)?

(a)

(b)

(c)

(d)

Answer: (c)

41. A planet of radius R has a time period of revolution T. Find time period of planet of radius 9R?

(a) 3√3T

(b) 9T

(c) 27T

(d) 9√3T

Answer: (c)

42. A toy truck of mass 2m elastically collides with a toy car of mass m. speed of truck is v and car is at rest. Find the velocity of car after collision.

(a) 4v/3

(b) v/3

(c) v

(d) 2v/3

Answer: (a)

43. If resistivity of copper is 1.72 × 108 Ω-m and number of free electrons in copper is 8.5 × 1028/m3. Find the mobility.

(a) 4.25 × 103 m2/CΩ

(b) 6.8 × 103 m2/CΩ

(c) 8.5 × 103 m2/CΩ

(d) 3.4 × 103 m2/CΩ

Answer: (a)

44. Van De graff generator is used to

(a) create a high potential of range of few million volts

(b) create a low potential of range of few million volts

(c) to de-accelerate projectile; like protons, deuterons etc.

(d) it can not be used to study collision experiments in physics

Answer: (a)

45. Coercivity and retentivity of soft iron is

(a) high coercivity, high retentivity

(b) low coercivity, high retentivity

(c) low coercivity, low rententivity

(d) high coercivity, low retentivity

Answer: (c)

46. In YDSE angular fringe width is 0.1ʹ wavelength of light used is 6000 Å. Find the distance between two slits.

(a) 2 μm

(b) 3 μm

(c) 5 μm

(d) 6 μm

Answer: (d)

47. Lenz law is based on principle of conservation of

(a) Linear momentum

(b) Energy

(c) Charge

(d) Mass

Answer: (b)

48. Two objects of mass m each moving with speed u m/s collide at 90°, then final momentum is (assume collision is inelastic)

(a) mu

(b) 2 mu

(c) √2 mu

(d) 2√2 mu

Answer: (c)

49. A particle is projected at an angle of 30° from the horizontal with a speed of 10 m/s on the earth. It’s time of flight, range, velocity of impact are respectively T, R, V. If this projectile is projected on the moon then its time of flight, range and velocity of impact is (assume gravitational field on moon is g/6 m/s2)

(a) 6T, 6R, 6V

(b) 6T, 6R, V

(c) T, R, V

(d) T, 6R, 6V

Answer: (b)

50. A particle is moving with 10 m/s in a circle of radius 5m, find out magnitude of average velocity if particle moved by 60° in 1 sec.

(a) 5 m/s

(b) 10 m/s

(c) 5√3 m/s

(d) 20 m/s

Answer: (a)

51. A particle of mass 7 kg executing circular motion with time period of 11 sec. Find out centripetal force, if radius circle is 10 m.

(a) 30/7 N

(b) 40/7 N

(c) 30 N

(d) 160/7 N

Answer: (d)

52. A stationary wave equation is given as y = 20 sin(20x) cos (1000 t).

What will be the speed of stationary wave?

(a) 20 m/s

(b) 50 m/s

(c) 2 cm/s

(d) not defined

Answer: (d)

53. Huygen’s principle does not use

(a) Reflection

(b) Refraction

(c) Diffraction

(d) Point of spectra origin

Answer: (d)

54. When a light ray enters from oil to glass on oil-glass interface, then velocity of light changes by a factor of

[given, μoil = 2, μglass = 3/2]

(a) 4/3

(b) 3/4

(c) 3

(d) 1

Answer: (a)

55. For CE configuration n-p-n transistor.

Which of the following statement is correct?

(a) IC = IE + IB

(b) IB = IE + IC

(c) IE = IC + IB

(d) All of these

Answer: (c)

56. Psychrometer is used to measure

(a) relative humidity

(b) pressure

(c) temperature

(d) density

Answer: (a)

57. If same charge q is placed inside a sphere and cube having radius 1 m and side 2 m respectively. What will be the ratio of flux passing through them?

(a) 1 : 1

(b) 1 : 8

(c) 8 : 1

(d) 1 : 2

Answer: (a)

58. Induction furnace uses ……….. to produce heat.

(a) eddy current

(b) resistance

(c) capacitor

(d) None of these

Answer: (a)

59. The given combination will work as

(a) NAND gate

(b) OR gate

(c) AND gate

(d) NOR gate

Answer: (c)

60. In which radiation atomic number and mass number are not affected?

(a) α-radiation

(b) γ-radiation

(c) β-radiation

(d) None of these

Answer: (b)

Chemistry

61. Final product is

(a) propene

(b) propanol

(c) cyclopropane

(d) propane-1, 2-diol

Answer: (a)

62. Which of the following can from H-bond?

(a) NH­3

(b) R-CN

(c) R-O-R

(d) R-Br

Answer: (a)

63. Which of the following amino acid is optically inactive?

(a) Glycine

(b) Valine

(c) Alanine

(d) Histidine

Answer: (a)

64. In which of the following, oxidation state of phosphorus is +3?

(a) Orthophosphoric acid

(b) Pyrophosphoric acid

(c) Orthophosphorous acid

(d) Meta phosphoric acid

Answer: (c)

65. If react with Cl2 in presence of light and then react witih Na metal in dry ether, final product is

(a)

(b)

(c)

(d)

Answer: (d)

66.

What is suitable reagent (X)?

(a) H2 | Pt

(b) B6|THF, H2O2|OH

(c) Br2 | HCl

(d) HBr | H2O2

Answer: (b)

67. What is the oxidation number of Cr in Na2Cr2O7?

(a) 2

(b) 6

(c) 10

(d) 16

Answer: (b)

68. What is the value of γ for mono atomic gas (ideal gas) ?

(a) 7/5

(b) 4/3

(c) 5/2

(d) None of these

Answer: (d)

69. Formula for half-life of a zero order reaction is

(a) C0/K

(b) C0/2K

(c) 2C0/K

(d) 2C0/2K

Answer: (b)

70. In an ideal gas equation which is constant?

(a) Temperature

(b) Pressure

(c) Volume

(d) Universal gas constant

Answer: (d)

71. If two atoms have equal number of electron, it is called

(a) isoelectronic

(b) isotone

(c) isobar

(d) None of these

Answer: (a)

72. Hund’s rule state that

(a) number of two e can be in two separate orbitals

(b) number of two e can be present with similar spin in a orbital

(c) no one e can exist in d orbital

(d) None of the above

Answer: (d)

73. 19F1, 16O2 20Ne choose the correct statement

(a) Both O2 and F are isoelectronic

(b) All given have equal to of e

(c) F and Ne have equal number of e

(d) All of the above

Answer: (d)

74. Which method is used to find halogen in organic compound?

(a) Duma’s method

(b) Leibig’s method

(c) Kjeldahl method

(d) Carius method

Answer: (d)

75. Which of the following is not iso electronic with H2S?

(a) F2 gas

(b) oxide ion

(c) Ca+2

(d) Sc+3

Answer: (b)

76. Number of σ and π bonds in C2H2 is respectively

(a) 2, 3

(b) 3, 2

(c) 4, 2

(d) 2, 4

Answer: (b)

77. If molarity of Cu+2 ions is 3 × 104 express this in ppm

(a) 0.3

(b) 0.2

(c) 0.1

(d) 0.6

Answer: (a)

78. Which of the following has octet around central atom?

(a) PF5

(b) SF6

(c) CCl4

(d) BF3

Answer: (c)

79. Which of the following is correct?

(a) SO32 ⇒ Tetrahedral

(b) NO2 ⇒ Trigonal planar

(c) ClO4 ⇒ Tetrahedral

(d) NH3 ⇒ Trigonal planar

Answer: (c)

80. If 50 ml of 0.1 HBr is mixed with 50 ml 0.2 M NaOH, find pH of resulting mixture

(a) 2.7

(b) 12.7

(c) 10.7

(d) 1.3

Answer: (b)

81. IUPAC name of is

(a) 4-hydroxy-3-methyl cyclohexene

(b) 2-methyl cyclohex-3-en-1-ol

(c) 3-hydroxy-2-methyl cyclohexene

(d) 1-Hydroxy-3-methyl cyclohexene

Answer: (b)

82. Bisphenol A is

(a) Phenol + Ethanol

(b) Phenol + Aniline

(c) Phenol + Acetone

(d) Phenol + Ethanal

Answer: (c)

83. If equilibrium constant is 2.6 × 108 at 298 K, then find the value of ∆G°.

(a) −48.14 kJ

(b) −63.2 kJ

(c) −23.3 kJ

(d) +40 kJ

Answer: (a)

84. Which of the following is not correct?

(a) PhCH2Br > PhCHBrCH3 > PhCBr(CH3)2(SN1)

(b) R – I > R – Br > R – Cl (SN2)

(c) CH2 = CH – Cl < CH2 = CH – CH2 – Cl < PhCH2 – Cl (SN1)

(d) R – Cl < R – Br < R – I (SN1)

Answer: (a)

85. Invert sugar is mixture of

(a) D-glucose + D-fructose

(b) L-glucose + D-fructose

(c) L-glucose + D-glucose

(d) L-glucose + L-glucose

Answer: (a)

86. Which of the following is not allotrope of carbon?

(a) Diamond

(b) Fullerene

(c) Soot

(d) Graphite

Answer: (c)

87. Calculate molarity of one litre solution of 22.2 gm CaCl2.

(a) 0.4 M

(b) 0.2 M

(c) 0.8 M

(d) 0.6 M

Answer: (b)

88. If V = 1 litre, 10 mole of H2 and 10 mole of N2 gas are mixed temperature 26°C, then calculate pressure of the gas.

(a) 491 atm

(b) 300 atm

(c) 550 atm

(d) 600 atm

Answer: (a)

89. Which mixture is immiscible?

(a) C5H12 + Alcohol

(b) Alcohol + Amine

(c) H2O + Alcohol

(d) Alkane + Alkane

Answer: (a)

90. Silicon carbide is example of

(a) molecular solid

(b) metallic solid

(c) ionic crystal

(d) covalent solid

Answer: (d)

91. Which type of defect present in NaCl?

(a) Frenkel defect

(b) Schottky defect

(c) F-centre defect

(d) None of these

Answer: (b)

92. Which one follow 18 electron octet rule?

(a) [Mn(CO)5]

(b) [Cr(CO)5]

(c) [Fe(CO)5]

(d) None of these

Answer: (c)

93.

Suitable reagent for conversion of following reaction?

(a) NaBH4

(b) Ni/H2

(c) Zn/Hg/HCl

(d) Red P + Hi

Answer: (a)

94. Bordeaux mixture consist of

(a) CuSO4 + Ca(OH)2

(b) CuSO4 + CaCl2

(c) ZnSO4 + Mg(OH)2

(d) FeSO4 + Ba(OH)2

Answer: (a)

95. BrO­3 changes into Br2 in an acidic medium of a unbalanced equation. How many electron should be present on the balanced equation?

(a) 10 electron in left

(b) 6 electron in left

(c) 3 electron in left

(d) 3 electron in left

Answer: (a)

96. Frenkel defect is present in which of the following?

(a) NaOH

(b) NaI

(c) AgBr

(d) None of these

Answer: (c)

97. Butter is a colloidal solution of

(a) solid-solid

(b) liquid-solid

(c) solid-liquid

(d) gas-solid

Answer: (b)

98. Azimuthal quantum number (ℓ) defined

(a) shape of orbitals

(b) orientation of orbitals

(c) energy of orbitals

(d) size of orbitals

Answer: (a)

99. 2KHCO3 →… + CO2 + H2O find amount of gases formed (in lit). When amount of KHCO3 is 33 gm.

(a) 5.6

(b) 11.2

(c) 7.46

(d) 22.4

Answer: (c)

100. final product C is

(a) CH3CH2OH

(b) CH3CH2 – Br

(c) CH3 – CH2 – I

(d)

Answer: (b)

101. Glucose IUPAC name of Y and Z

(a) Hexa hydroxyl heptanoic acid, heptane

(b) Hepta hydroxyl hexanoic acid, hexane

(c) Penta hydroxyl hexanoic acid, hexane

(d) Hepta hydroxyl hexanoic acid, heptane

Answer: (d)

102. Write the IUPAC name of given structure

(a) Para bromo benzene

(b) 1, 4-di bromo benzene

(c) Both (a) and (b)

(d) Meta bromo benzene

Answer: (b)

103. Which of the following statement is correct for isotope of carbon?

(a) graphite is conductor of electricity

(b) diamond have all sp3 carbon

(c) graphite is more stable thermodynamically than diamond

(d) All are correct

Answer: (d)

104. Write IUPAC name of the following

(a) 2-bromo-2-ethyl hexanal

(b) 3-bromo-2-methyl hexanal

(c) 2-methyl-3-bromo hexanal

(d) 3-bromo-2-formyl hexane

Answer: (b)

105. Most common isotopes of hydrogen (non-radioactive) is

(a) protium

(b) deuterium

(c) tirium

(d) All of these

Answer: (a)

106. How many ions obtain after dissociation of this complex [Co(NH3)6]Cl3?

(a) 3

(b) 2

(c) 5

(d) 4

Answer: (d)

107. Given, The potential for the cell

Cr|Cr3+(0.1M) || Fe2+(0.01 M)Fe is

(a) −0.339 V

(b) −0.026 V

(c) 0.26 V

(d) 0.3 V

Answer: (c)

108. The magnetic moment of Ni2+ ion (atomic number of Ni = 28) in BM unit is

(a) 1.73

(b) 4.81

(c) 5.96

(d) 2.84

Answer: (d)

109. Which one of the following nitro compound does not react with nitrous acid?

(a)

(b)

(c)

(d)

Answer: (c)

110. The main product obtained from phenol with PCl5 is

(a) BHC

(b) hexachlorobenzene

(c) chlorobenzene

(d) triphenyl phosphate

Answer: (d)

111. An element with mass number 81 contains 31.7% more neutrons as compared to protons. It symbol is

(a) 40X81

(b) 35X81

(c) 81X317

(d) 40X81

Answer: (b)

112. Which of the following will not be formed when calcium acetate is distilled with calcium formate?

(a) Acetone

(b) Propanal

(c) Ethanal

(d) Methanal

Answer: (b)

113. If half-life of a surface is 36 minutes. Find amount left after 2 hrs. Initial amount is 10 gm?

(a) 1 g

(b) 2 g

(c) 3 g

(d) 4 g

Answer: (a)

114. Moles of N2 = 0.4 and moles of O2 = 0.1 find (partial pressure) N2 = ? at atmospherim pressure

(a) 0.2 atm

(b) 0.8 atm

(c) 0.6 atm

(d) 0.4 atm

Answer: (b)

115. What is hybridization of O in H2O?

(a) sp

(b) sp3

(c) sp2

(d) No hybridization

Answer: (b)

116. Which of the following act as epimeric pair?

(a) Glucose and fructose

(b) Fructose and mannose

(c) Glucose and mannose

(d) Glucose and sucrose

Answer: (c)

117. Which can not behave as nucleophile for SN2 reaction?

(a) H2O

(b) CN

(c) NH2

(d) I

Answer: (a)

118. Which of the following rate gas is the abundant in air?

(a) He

(b) Ne

(c) Ar

(d) Kr

Answer: (c)

119. Saccharin an artificial sweetener is manufactured form

(a) toluene

(b) cyclohexane

(c) starch

(d) cellulose

Answer: (a)

120. During nitration of benzene the attacking electrophile is

(a) NO3

(b) NO2

(c) NO2+

(d) HNO3

Answer: (c)

Biology

121. Verticillaster inflorescence is found in

(a) mustard

(b) Ocimum

(c) China rose

(d) sunflower

Answer: (b)

122. Zooidogamy is found in

(a) Cedrus

(b) Pinus

(c) Cycas

(d) Both (b) and (c)

Answer: (c)

123. Dark reaction

(a) in light

(b) in dark

(c) requires product of light reaction

(d) All of the above

Answer: (c)

124. Vernalisation is

(a) low pH treatment

(b) low temperature treatment

(c) high temperature treatment

(d) high pH treatment

Answer: (b)

125. An interaction in which one organism is benefitted and other is unaffected

(a) predation

(b) commensalism

(c) mutualism

(d) parasitism

Answer: (b)

126. Karyotype of Down’s syndrome has how many chromosomes?

(a) 43

(b) 46

(c) 47

(d) 45

Answer: (c)

127. Hardest substance in plant kingdom is

(a) saple

(b) corolla

(c) sporopollenin

(d) anther

Answer: (c)

128. Which of the following is not a micronutrient?

(a) B

(b) Zn

(c) Mn

(d) Mg

Answer: (d)

129. Cell placed in hypertonic solution is shown by

(a)

(b)

(c)

(d) Both (a) and (b)

Answer: (a)

130. Genetic variation cannot be effectively brought about by

(a) transduction

(b) mutation

(c) transformation

(d) amplification

Answer: (d)

131. Which of the following is not considered as a secondary messenger?

(a) Acetylcholine

(b) cAMP

(c) Ca2+

(d) diacylglycerol

Answer: (a)

132. Which is also called molecular glue

(a) DNA gyrase

(b) DNA helicase

(c) DNA ligase

(d) DNA polymerase

Answer: (c)

133. In DNA 20% bases are adenine. What percentage of bases are pyrimidines?

(a) 30%

(b) 60%

(c) 50%

(d) 20%

Answer: (c)

134. Most advanced invertebrates are

(a) Arthropoda

(b) Annelida

(c) Mollusca

(d) Cephalopoda

Answer: (d)

135. Gastrulation means

(a) conversion of blastula into morula

(b) formation of 3 germs layers

(c) a phase in which organogenesis takes places

(d) a phase characterized by inner cell mass

Answer: (b)

136. Traditional anesthesia is obtained from

(a) Datura

(b) poppy

(c) Cannabis

(d) erythroxylum

Answer: (d)

137. Band 3 protein in RBC has the function of

(a) it mediates the exchange of cellular HCO3 with Cl in plasma

(b) it is a cytoplasmic protein

(c) it mediates the exchange of cations between cell and plasma

(d) Both (a) and (b)

Answer: (a)

138. Hypothalamus can regulate

(a) anger

(b) patience

(c) thermoregulation

(d) pleasure

Answer: (c)

139. Choose correct option for larva of housefly.

(a) Grub

(b) Cypris

(c) Maggot

(d) Wriggler

Answer: (c)

140. Chromosome walking is

(a) used in genetic mapping

(b) observed during mitosis

(c) to see position of ribosomes on nuclear membrane

(d) to check the expression of mutant genes

Answer: (a)

141. Which one is only found in plant?

(a) Ribosome

(b) Mitochondria

(c) Glyoxysomes

(d) Lysosome

Answer: (c)

142. Kranz anatomy is found in

(a) C3-plants

(b) Only in moncots

(c) Both C4 and C3-plants

(d) C4-plants

Answer: (d)

143. What will be the DNA content and number of chromosomes in a cell of after S-phase as compared to the gamete of the same organism?

(a) Same DNA content but double chromosome number

(b) Four times DNA content but double chromosome number

(c) Same DNA content but half chromosome number

(d) Half DNA content but double chromosome number

Answer: (b)

144. Radula is a part of which animal.

(a) Mollusca

(b) Poriferans

(c) Coelenterata

(d) Annelida

Answer: (a)

145. Chordae tendineae is found in

(a) heart of cockroach

(b) heart of Amphibians

(c) heart of Reptiles

(d) heart of Mammals

Answer: (d)

146. Length of oesophagus is

(a) 25 cm

(b) 55 cm

(c) 33 cm

(d) 45 cm

Answer: (a)

147. Citrulline structure

(a)

(b)

(c)

(d)

Answer: (a)

148. Bradykinin is a

(a) antibody

(b) hormone

(c) enzyme

(d) receptor

Answer: (b)

149. Reproduction without actual fertilization is called as

(a) parthenogenesis

(b) parthenocarpy

(c) pseudo-reproduction

(d) All of these

Answer: (a)

150. Coacervates were discovered by

(a) Huxley

(b) Fox

(c) Oparin

(d) Miller

Answer: (c)

151. Which of the following is not a stop codon?

(a) UAA

(b) UAC

(c) UAG

(d) UGA

Answer: (b)

152. Which of the following is uricotelic?

(a) Insects

(b) Birds

(c) Lizards

(d) All of these

Answer: (d)

153. To make chromosomal studies easier, chromosomes are classified into certain groups. So, the chromosome number 21, 22, and Y are listed in

(a) A

(b) D

(c) E

(d) G

Answer: (d)

154. Choose the incorrect about gemmules.

(a) They resist dessication

(b) They are internal buds

(c) They are asexual structure which are produce by binary fission

(d) They can give rise to new organisms

Answer: (c)

155. Choose the correct regarding antibodies.

(a) IgA – Helps in allergic reaction

(b) IgG – Cross placenta

(c) IgE – Found in secretions

(d) IgM – Exists as dimer

Answer: (b)

156. During DNA replication, supercoilling is relaxed by

(a) primase

(b) polymerase

(c) DNA topoisomerase

(d) SSBPs

Answer: (c)

157. Structure analogous to the placenta in mammals.

(a) Chorion

(b) Amnion

(c) Yolk sac

(d) Allantois

Answer: (a)

158. In annealing

(a) primers anneal to the DNA template

(b) Taq polymerase adds nucleotides to the annealed primer

(c) two strands of the DNA separate

(d) temperature is 92°C

Answer: (a)

159. Which of the following is not true?

(a) β – galactosidase – lac operon

(b) Photorespiration – C3 – plants

(c) Krebs cycle – mitochondrial matrix

(d) Sothern hybridization – related to identification of RNA sequence

Answer: (d)

160. Carcinogen present in cigarette smoke is

(a) benzopyrene

(b) nicotine

(c) carbon monoxide

(d) All of these

Answer: (a)

161. Atlas is

(a) 1st cervical vertebrae

(b) 2nd cervical vertebrae

(c) 1st thoracic vertebrae

(d) 2nd lumbar vertebrae

Answer: (a)

162. Typhoid is caused by

(a) penumonia

(b) filarial worm

(c) Salmonella typhii

(d) None of the above

Answer: (c)

163. Chl-b differ from chl-a by

(a) −CH3 group

(b) CHO group

(c) COOH group

(d) CH2OH group

Answer: (b)

164. Number of chromosomes present in secondary spermatocyte is

(a) 22

(b) 23

(c) 24

(d) 25

Answer: (b)

165. Flame cells are present in

(a) Aschelminthes

(b) Platyhelminthes

(c) Annelida

(d) Cephalochordata

Answer: (b)

166. Which one of the labeled part utilizes fructose as a source of energy?

(a) Head

(b) Tail

(c) Acrosome

(d) Middle piece

Answer: (d)

167. A normal man and a normal woman both having one of their parent albino, if get married then what is the probability of their child of being albino?

(a) 20%

(b) 25%

(c) 75%

(d) 80%

Answer: (b)

168. Which is the correct sequence of modification and transport of secretory vesicles?

(a) RER → Cell membrane → Secretory vesicles → Golgi body

(b) RER → Secretory vesicles → Cell membrane

(c) RER → Cell membrane → Secretory vesicles → Golgi body

(d) SER → Golgi body → Cell membrane → Secretory vesicles

Answer: (c)

169. Among the following choose the correct epimers.

(a) Glucose, galactose

(b) Glucose, fructose

(c) Glucose, ribose

(d) Gluconic acid, glucose

Answer: (a)

170. Gynaecomastia is present in which syndrome?

(a) Turner

(b) Klinefelter

(c) Down

(d) None of the above

Answer: (b)

171. In gluconeogenesis, which of the following cannot act as substrate?

(a) Fructose

(b) Palmitic acid

(c) Glycine

(d) Glycerol

Answer: (a)

172. Identify the diagram of heterocyst.

(a)

(b)

(c)

(d)

Answer: (a)

173. Level of LH is maximum

(a) just before ovulation

(b) just after ovulation

(c) at the time of ovulation

(d) during menstrual bleeding phase

Answer: (c)

174. Number of chromosomes present in spermatogonium is

(a) 46

(b) 23

(c) 48

(d) 43

Answer: (a)

175. Which one of them is odd with respect to human male?

(a) absence of one X-chromosome

(b) addition of one X-chromosome

(c) presence of XY-chromosome

(d) absence of Barr body

Answer: (a)

176. Eutrophication is caused due to

(a) accumulation of minerals

(b) effect of UV-C

(c) accumulation of metals only

(d) accumulation of zooplankton

Answer: (a)

177. Synaptonemal complex is formed

(a) during anaphase

(b) during metaphase

(c) during prophase-II

(d) during prophase-I of meiosis

Answer: (d)

178. Which one is the oldest in following?

(a) Gymnosperms

(b) Angiosperms

(c) Bryophytes

(d) None of these

Answer: (c)

179. RNA binds to mRNA through

(a) anticodon loop

(b) T Ψ C loop

(c) amino acid binding loop

(d) D loop

Answer: (a)

180. Find odd one out.

(a) Stamen

(b) Stigma

(c) Style

(d) Ovary

Answer: (a)

English and Quantitative Reasoning

Directions (Q. Nos. 181-185) Read the following passage carefully and answer the questions given below.

In this passage, Ria and Gargi give their opinions about a new painting by Marcello Vasquez. Ria, ‘The new painting is spectacular! It is certainly the best work ever completed by Marcello Vasquez. I like the new painting for several reasons. First, the painting is large. It almost covers the entire wall! Second, the painting is inexpensive. Normally, a painting like this would cost thousands of dollars. But this painting costs less than $500. Third, the painting is colourful. Red and green dominate the canvas, peppered with bright spots of yellow.’

Gargi, ‘The new painting is horrendous! Marcello Vasquez should be ashamed of his work. I haven’t seen a painting this bad in a long time. I dislike the new painiting for several reasons. First, the painting is far too large. It almost covers the entire wall! I think small paintings are better than large ones. Second, the painting is inexpensive. This tells me that Vasquez must have been unable to sell the painting for a higher price. Now, he is desperate. As a result, it appears he is trying to give the painting away. Third the painting is colourful. Although I usually like a painting to be colourful. Although I usually like a painting to be colourful, I strongly dislike the colours Vasquez uses. To p u tint plainly, the combination of red, green and yellow is ugly.’

181. With respect to how they like new painting Ria and Gargi?

(a) agree

(b) strongly agree

(c) disagree

(d) strongle disagree

Answer: (d)

182. Ria and Gargi both view the new painting as

I. Large II. Inexpensive III. Colourful

Select the correct answer using the codes given below

(a) Only I

(b) I and II

(c) II and III

(d) All of these

Answer: (d)

183. In the second paragraph Gargi said it can be understood that ‘desperate’ belongs to which of the following word groups?

(a) Passionate, enthusiastic, excited

(b) Idiotic, silly, foolish

(c) Anxious, worried, troubled

(d) Improverished, broke poor

Answer: (c)

184. As used in the passage, which of the following words always has/have negative meaning?

I. Spectacular II. Horrendous IIII. Inexpensive

Select the correct answer using the codes given below

(a) Only I

(b) Only II

(c) II and III

(d) All of these

Answer: (b)

185. Gargi and Ria go to see another painting. The painting is small, expensive and colourful, who would probably like this painting?

(a) Ria

(b) Gargi

(c) Both Ria and Gargi

(d) Neither Ria nor Gargi

Answer: (b)

Directions (Q. Nos. 186-187) Each of the following question consist of a word, followed by four words or group of words. Select the antonym of the word given below.

186. Expansion

(a) Depression

(b) Negligent

(c) Precise

(d) Enlargement

Answer: (c)

187. Thrift

(a) Parsimony

(b) Dismantle

(c) Prodigality

(d) Essential

Answer: (c)

Directions (Q. Nos. 188-190) Choose the correct option out of the choices given below.

188. He will abide ………. his friend Suresh in all circumstances.

(a) with

(b) by

(c) along

(d) to

Answer: (a)

189. He should not be blind ………….. the misdeeds of his son Rahul.

(a) at

(b) to

(c) with

(d) on

Answer: (b)

190. God had destine him ………. the post of Prime Minister of India

(a) for

(b) to

(c) with

(d) on

Answer: (a)

191. In a certain code language, ‘SAFER’ is written as ‘5@3#2’ and ‘RIDE’ is written as ‘2©%#’, how would ‘FEDS’ be written in that code?

(a) 3#©5

(b) 3@%5

(c) 3#%5

(d) 3#%2

Answer: (c)

192. Find the missing number from the given response.

(a) 72

(b) 720

(c) 7200

(d) 38

Answer: (b)

193. If the first and second letters in the word DEPRESSION were interchanged, also the third and fourth letters, the fifth and the sixth letters and so on, then which of the following would be seventh letter from the right.

(a) O

(b) P

(c) R

(d) S

Answer: (b)

194. Today is Thursday. The day after 59 days will be

(a) Sunday

(b) Monday

(c) Tuesday

(d) Wednesday

Answer: (a)

195. Which of the following represents coal mines, factories and fields?

(a) 

(b) 

(c)

(d)

Answer: (d)

196. Find out the missing term in the series.

1, 8, 27, ?, 125, 216

(a) 52

(b) 58

(c) 64

(d) 65

Answer: (c)

197. If ‘+’ means ‘×’, ‘−’ means’ ‘+’, ‘x’ means ‘÷’ and ‘÷’means ‘−’, then 6 – 9 + 8 × 3

(a) −2

(b) 6

(c) 10

(d) 12

Answer: (c)

198. What is the water image of

(a)

(b) 

(c)

(d)

Answer: (b)

199. A piece of paper is folded and punched as shown in the figure below.

How will it appear when unfolded.

Answer: (b)

200. Here are some words translated from an artificial language. Mallon piml means blue light mallon tifl means blue berry arpan tifl means rasp berry?

Which word could means ‘light house’?

(a) tiflmallon

(b) pimlarpan

(c) mallonarpan

(d) pimldoken

Answer: (d)

JIPMER Medical Entrance Exam Previous Year Question Paper 2018 With Answer Key

JIPMER SOLVER PAPER-2018

1. The magnetic moment of an electron orbiting in a circulate orbit of radius r with a speed v is equal to

(a)  evr/2

(b)  evr

(c)  er/2v

(d) None of these

Answer: (a)

2. If point charges Q1 = 2 × 107 C and Q2 = 3 × 107 C are at 30 cm separation, then find electrostatic force between them

(a)  2 × 103 N

(b)  6 × 103 N

(c)  5 × 103 N

(d) 1 × 103 N

Answer: (b)

3. If a machine perform 4000 J output work and 1000 J inside loss due to friction, then the find efficiency?

(a)  80%

(b)  30%

(c)  25%

(d) 60%

Answer: (a)

4. Dimension of force is

(a)  [M2L1T−1]

(b)  [M1L1T−2]

(c)  [M2L−1T−2]

(d) [M1L1T−1]

Answer: (b)

5. The efficiency of an ideal gas with adiabatic exponent γ for the shown cyclic process would be

(a) 

(b) 

(c) 

(d) 

Answer: (a)

6. Find Rnet between A and B.

(a)  60 Ω

(b)  40 Ω

(c)  70 Ω

(d) 20 Ω

Answer: (b)

7. Velocity is given by v = 4t(1 – 2t), then find time at which velocity is maximum

(a)  0.25 s

(b)  1 s

(c)  0.45 s

(d) 4 s

Answer: (a)

8. Find current (i) in circuit shown in figure.

(a)  0.5 A

(b)  0.2 A

(c)  1 A

(d) 2 A

Answer: (a)

9. Find VP – VQ in the circuit shown in figure.

(a)  6.68 V

(b)  8 V

(c)  4.65 V

(d) 7 V

Answer: (c)

10. If a capacitor having capacitance 2F and plate separation of 0.5 cm will have area

(a)  1130 cm2

(b)  1130 m2

(c)  1130 km2

(d) None of these

Answer: (c)

11. Find the capacitance in shown figure

(a) 

(b) 

(c) 

(d) 

Answer: (a)

12. Find ratio of radius of gyration of a disk and ring of same radii at their tangential axis in plane.

(a) 

(b) 

(c)  1

(d) 2/3

Answer: (a)

13. If compressibility of material is 4 × 105 per atm, pressure is 100 atm and volume is 100 cm3, then find ∆V = ?

(a)  0.4 cm3

(b)  0.8 cm3

(c)  0.6 cm3

(d) 0.2 cm3

Answer: (a)

14. If speed of sound in air is 340 m/s and in water is 1480 m/s. If frequency of sound is 1000 kHz, then find wavelength in water

(a)  2.96 mm

(b)  1.48 mm

(c)  0.74 mm

(d) 1 mm

Answer: (b)

15. 1000 N force in required to lift a hook and 1000 N force is requires to lift a load slowly. Find power required to lift hook with load with speed v = 0.5 m/s

(a)  5 kW

(b)  1.5 kW

(c)  5.5 kW

(d) 4.5 kW

Answer: (c)

16. If minimum deviation = 30°, then speed of light in shown prism will be

(a) 

(b) 

(c) 

(d) 

Answer: (a)

17. A current i is flowing through the wire of diameter (d) having drift velocity of electrons vd in it. What will be new drift velocity when diameter of wire is made d/4?

(a)  4vd

(b)  vd/4

(c)  16vd

(d) vd/16

Answer: (c)

18. How much intense is 80dB sound in comparision to 40 dB?

(a)  102

(b)  104

(c)  2

(d) 1/2

Answer: (b)

19. Find i in shown figure

(a)  0.2 A

(b)  0.1 A

(c)  0.3 A

(d) 0.4 A

Answer: (b)

20. A force of 10 N acts on a body of mass 0.5 kg for 0.25s starting from rest. What is its momentum now?

(a)  0.25 N/s

(b)  2.5 N/s

(c)  0.5 N/s

(d) 0.75 N/s

Answer: (b)

21. Which of the following is fusion process?

(a) 

(b) 

(c)  Uranium decay

(d) None of the above

Answer: (a)

22. A ball of 0.5 kg collided with wall at 30° and bounced back elastically. The speed of ball was 12m/s. The contact remained for 1s. What is the force applied by wall on ball?

(a)  12√3 N

(b)  √3 N

(c)  6√3 N

(d) 3√3 N

Answer: (c)

23. A electron (e) is accelerated by V volts experiences a force F, when it enters in a uniform magnetic field. What will the force experienced when it is accelerated by 2V?

(a)  √2F

(b)  F

(c)  2F

(d) F/2

Answer: (a)

24. Kinetic energy of a particle is increased by 4 times. What will be the relation between initial and final momentum?

(a)  p2 = 2p1

(b)  p2= p1/2

(c)  p2 = p1

(d) p2 = 4p1

Answer: (a)

25. What is the range of a projectile thrown with velocity 98 m/s with angle 30° from horizontal?

(a)  490√3 m

(b)  245√3 m

(c)  980√3 m

(d) 100 m

Answer: (a)

26. If the efficiency of an engine is 50% and its work output is 500 J, then find input.

(a)  1000 J

(b)  500 J

(c)  100 J

(d) 250 J

Answer: (a)

27. The efficiency of a heat engine is 1/6. Its efficiency double when the temperature of sink decrease by 62°C, its efficiency doubles. Then, what is the temperature of source?

(a)  470 K

(b)  372 K

(c)  542 K

(d) 1042 K

Answer: (b)

28. A ball is thrown upwards with a speed u from a height h above the ground. The time taken by the ball to hit the ground is

(a) 

(b) 

(c) 

(d) 

Answer: (c)

29. A mass of Hg is executing SHM which is given by  What is the maximum kinetic energy?

(a)  3 J

(b)  6 J

(c)  9 J

(d) 18 J

Answer: (d)

30. A uniform rod of mass m and length l0 is pivoted at one end and is hanging in the vertical direction. The period of small angular oscillations of the rod is

(a) 

(b) 

(c) 

(d) 

Answer: (d)

31. A box of mass 8 kg is placed on a rough inclined plane of inclination 30°. Its downward motion can be prevented by applying a horizontal force F, then value of F for which friction between the block and the incline surface is minimum, is

(a)  80/√3

(b)  40√3

(c)  40/√3

(d) 80√3

Answer: (a)

32. A mass M is hung with a light inextensible string as shown in the figure. Find the tension of the horizontal string.

(a)  √2 Mg

(b)  √3 Mg

(c)  2 Mg

(d) 3 Mg

Answer: (b)

33. A Carnot engine absorbs 6 × 105 cal at 227° The work done per cycle by the engine, if its sink is maintained at 127°C is

(a)  15 × 108 J

(b)  15 × 104 J

(c)  5 × 105 J

(d) 2 × 104 J

Answer: (c)

34. A machine gun fires 360 bullets per minute, with a velocity of 600 ms1. If the power of the gun is 5.4 kW, then mass of each bullet is

(a)  5 kg

(b)  0.5 kg

(c)  5 g

(d) 0.5 g

Answer: (c)

35. A rain drop of radius 0.3 mm has a terminal velocity in air 1 ms1. The viscosity of air is 18 × 105 Find the viscous force on the rain drops.

(a)  2.05 × 107 N

(b)  1.018 × 107 N

(c)  1.05 × 107 N

(d) 2.058 × 107 N

Answer: (b)

36. Find density of ethyl alcohol

(a) 

(b) 

(c) 

(d) 

Answer: (a)

37. An atomic power nuclear reactor can deliver 300 MW. The energy released due to fission of each nucleus of uranium atoms U238 is 170 MeV. The number of uranium atoms fissioned per hour will be

(a)  30 × 1025

(b)  4 × 1022

(c)  10 × 1020

(d) 5 × 1015

Answer: (b)

38. How much should the temperature of a brass rod be increased so as to increase its length by 1%? (Given, α for brass is 0.00002°C1).

(a)  300°C

(b)  400°C

(c)  500°C

(d) 550°C

Answer: (c)

39.  what is time period?

(a)  0.04 s

(b)  1 s

(c)  0.006 s

(d) 0.02 s

Answer: (a)

40. In the fusion reaction  the masses of deuteron, helium and neutron expressed in amu are 2.015, 3.017 and 1.009, respectively. If 1 kg of deuterium undergoes complete fusion, find the amount of total energy released. (1 amu = 931.5 MeV).

(a)  9.0 × 1013 J

(b)  20 × 105 J

(c)  5 × 1016 J

(d) 8 × 105 J

Answer: (a)

41. A prism of crown glass with refracting angle of 5° and mean refractive index = 1.51 is combined with a flint glass prism of refractive index = 1.65 to produce no deviation. Find the angle of flit glass.

(a)  3.92°

(b)  4.68°

(c)  5.32°

(d) 7.28°

Answer: (a)

42. Water flows through a horizontal pipe of variable cross-section at the rate of 20 L min1. What will be the velocity of water at a point where diameter is 4 cm?

(a)  0.2639 ms1

(b)  0.5639 ms1

(c)  0.4639 ms1

(d) 0.3639 ms1

Answer: (a)

43. The coefficient of volume expansion of glycerine is 49 × 105°C1. What is the fractional change in density for a 30°C rise in temperature?

(a)  0.0155

(b)  0.0145

(c)  0.0255

(d) 0.0355

Answer: (b)

44. Two slits are separated by a distance of 0.5 mm and illuminated with light of λ = 6000 Å. If the screen is placed 2.5 m, from the slits. The distance of third bright image from the centre will be

(a)  1.5 mm

(b)  3 mm

(c)  6 mm

(d) 9 mm

Answer: (d)

45. Calculate the dispersive power for crown glass from the given data

μV = 1.523

and         μR = 1.5145

(a)  0.01639

(b)  1.05639

(c)  0.05639

(d) 2.05639

Answer: (a)

46. A particle doing SHM having amplitude 5 cm, mass 0.5 kg and angular frequency 5 rad/s is at 1 cm from mean position. Find potential energy and kinetic energy.

(a)  KE = 625 × 104 J, PE = 150 × 103 J

(b)  KE = 150 × 104 J, PE = 625 × 104 J

(c)  KE = 625 × 104 J, PE = 625 × 104 J

(d) KE = 150 × 103 J, PE = 150 × 104 J

Answer: (b)

47. A organ pipe open on both ends in the nth harmonic is in resonance with a source of 1000 Hz. The length of pipe is 16.6 cm and speed of sound in air is 332 m/s. Find the value of n.

(a)  3

(b)  2

(c)  1

(d) 4

Answer: (c)

48. R = 65 ± 1 Ω L, l = 5 ± 0.1 mm and d = 10 ± 0.5 mm. Find error in calculation of resistivity.

(a)  21%

(b)  13%

(c)  16%

(d) 41%

Answer: (b)

49. A solid floats with 1/4th of its volume above the surface of water, the density of the solid is

(a)  750 kg m3

(b)  650 kg m3

(c)  560 kg m3

(d) 450 kg m3

Answer: (a)

50. The work done in blowing a soap bubble of surface tension 0.06 Nm1 from 2 cm radius to 5 cm radius is

(a)  0.004168 J

(b)  0.003168 J

(c)  0.003158 J

(d) 0.004568 J

Answer: (b)

51. The force of attraction between two charges 8 μC and −4 μC is 0.2 N. Find the distance of separation

(a)  1.2 cm

(b)  12 m

(c)  120 m

(d) 0.12 m

Answer: (a)

52. A runner starts from O and goes to O following path OQRO in 1 hr. What is net displacement and average speed?

(a)  0, 3.57 km/hr

(b)  0, 0 km/hr

(c)  0, 2.57 km/hr

(d) 0, 1 km/hr

Answer: (a)

53. A disc of moment of inertia 2 kg-m2 revolving with 8 rad/s is placed on another disc of moment of inertia 4 kg-m2 revolving 4 rad/s. What is the angular frequency of composite disc?

(a)  4 rad/s

(b)  3/16 rad/s

(c)  16/3 rad/s

(d) 16/5 rad/s

Answer: (c)

54. In a L-C circuit, angular frequency at resonance is ω. What will be the new angular frequency when inductor’s inductance is made two times and capacitor’s capacitance is made four times?

(a)  ω/2√2

(b)  ω/√2

(c)  2ω

(d) 2ω/√2

Answer: (a)

55. Electron revolving with speed v is producing magnetic field B at centre. Find relation between radius of path B and v?

(a)  B ∝ V ∝ 1/r

(b)  B ∝ V ∝ 1/r2

(c)  B ∝ V2 ∝ 1/r

(d) B ∝ V2 ∝ 1/r2

Answer: (b)

56. A regular hexagone of side a. A wire of length 24 a is coiled on that hexagone. If current in hexagone is I, then find the magnetic moment.

(a)  6√3 la2

(b)  3√3 la2

(c) 

(d) 6 la2

Answer: (a)

57. At what speed will the velocity of a stream of water be equal to 20 cm of mercury column? (Taking, g = 10 ms2)

(a)  6.4 ms1

(b)  7.3756 ms1

(c)  6.4756 ms1

(d) None of these

Answer: (b)

58. At what distance (in metre) from the centre of the Moon, the intensity of gravitational field will be zero? (Take, mass of Earth and Moon as 5.98 × 1024 kg and 7.35 × 1022 kg respectively and the distance between Moon and Earth is 3.85 × 108)

(a)  zero

(b)  3.85 × 107

(c)  8 × 108

(d) 3.46 × 108

Answer: (b)

59. The refractive index of glass is 1.5. The speed of light in glass is

(a)  3 × 108 m/s

(b)  2 × 108 m/s

(c)  1 × 108 m/s

(d) 4 × 108 m/s

Answer: (b)

60. R is the radius of Earth and ω is its angular velocity and gp is the value of g at the poles. The effective value of g at a latitude λ = 60°.

(a) 

(b) 

(c) 

(d) 

Answer: (a)

Chemistry

61. Which one of the following species is paramagnetic in nature?

(a)  V4+

(b)  Sc3+

(c)  V5+

(d) Zn2+

Answer: (d)

62. Given that, for the reaction,

∆Hf(HCl) = −93 kJ/mol

B.E (H2) = 434 kJ/mol

B.E (Cl2) = 242 kJ/mol

The bond dissociation energy of HCl is

(a)  232 kJ/mol

(b)  331 kJ/mol

(c)  431 kJ/mol

(d) 530 kJ/mol

Answer: (c)

63. The pair of structures given below represents

(a)  Chain isomers

(b)  Position isomers

(c)  Tautomers

(d) Metamers

Answer: (b)

64. In the extraction of copper from is sulphide ore, the metal is formed by the reduction of Cu2O with

(a)  FeS

(b)  CO

(c)  Cu2S

(d) SO2

Answer: (c)

65. For the following reaction,

2NH3(g) + CO2(g) ⇌ NH2CONH2 (aq) + H2O (l)

∆G° = −13.6 kJ mol1 at 298 K

Then, log Keq for this equilibrium is

(a)  −3.41

(b)  2.38

(c)  120

(d) 1500

Answer: (b)

66. Xenon hexatluoride reacts with silica to form a xenon compound X. The oxidation state of xenon of X is

(a)  +2

(b)  +4

(c)  +6

(d) 0

Answer: (c)

67. Neopentyl chloride reacts with benzene in presence of a lewis acid (AlCl3) to form

Answer: (b)

68. Lead is extracted from galena (PbS), either by following ways.

Auto-reduction process is

(a)  I

(b)  II

(c)  Both (a) and (b)

(d) None of these

Answer: (b)

69. In the reactoion,

The major product formed is

(a)  C6H5 – CH2 – CHO

(b)  C6H5 – COOH

(c)  C6H5 – CH = CH – COOH

(d) C6H5 – CH = CH – COCH3

Answer: (c)

70. Which one of the following represents the correct structure of cellulose?

(a)  Branched, α(1, 4) and β(1, 6) – glucose

(b)  Unbranched, α(1, 6) – glucose

(c)  Branched, α(1, 4) – glucose

(d) Unbranched, β(1, 4) – glucose

Answer: (d)

71. The correct formula for plaster of Paris is

(a)  CaSO4 ∙ 4H2O

(b)  CaSO4 ∙ 2H2O

(c)

(d) CaSO4 ∙ 1H2O

Answer: (c)

72. The value of KC for the reaction, 2A ⇌ B + C is 2 × 103. At a given time, the composition of reaction mixture is [A] = [B] = [C] = 3 × 104 In which direction, the reaction will proceed?

(a)  Forward direction

(b)  Reverse direction

(c)  No net reaction occur

(d) None of these

Answer: (b)

73. Which of the following chemicals are used to manufacture methyl isocyanate that caused “Bhopal tragedy”?

(i) Methylamine  (ii) Phosgene

(iii) Phosphine    (iv) Dimethylamine

(a)  (i) and (ii)

(b)  (iii) and (iv)

(c)  (i) and (iii)

(d) (ii) and (iv)

Answer: (a)

74. The correct order of ease of adsorption of the hydrated alkali metal ions on an ion exchange resin is

(a)  Rb+ > K+ > Na+ > Li+

(b)  Li+ > K+ > Na+ < Rb+

(c)  Na+ < Li+ < K+ < Rb+

(d) K+ < Na+ < Rb+ < Li+

Answer: (a)

75. Identify the major product that is obtained on treatment of CH3CH2CH(F)CH3 with CH3O/CH3OH is

(a)  CH3CH2(OCH3CH3

(b)  CH3CH = CHCH3

(c)  CH3CH2CH = CH2

(d) CH3CH2CH2CH2OCH3

Answer: (b)

76. Among the following, the non-aromatic heterocyclic compound is

(a)  piperidine

(b)  pyrrole

(c)  furan

(d) pyridine

Answer: (a)

77. If Na+ ioin is larger than Mg2+ ions and S2 ion is larger than Cl ion, which of the following will be stable salt in water?

(a)  Sodium chloride

(b)  Sodium sulphide

(c)  Magnesium sulphide

(d) Magnesium chloride

Answer: (c)

78. 

Product P is

(a) 

(b) 

(c) 

(d) 

Answer: (b)

79. Which of the following is correct statement?

(a)  Aluminium dissolves in dilute mineral acids but is made passive by concentrated nitric acid.

(b)  Aluminium vessel can be used as storage of NaOH.

(c)  Alminium chloride does not dissociate in water.

(d) AlF3 is soluble in anhydrous HF.

Answer: (a)

80. If molar conductivity of Ca2+ and Cl ions are 119 and 71S cm2mol1 respectively, then the molar conductivity of CaCl2 at infinite dilution is

(a)  215 S cm2mol1

(b)  340 S cm2mol1

(c)  128 S cm2mol1

(d) 261 S cm2mol1

Answer: (d)

81. An atom forms face centred cubic crystal with density d = 8.92 g/mL and edge length a = 3.6 × 108 The molecular mass of atom in amu is

(a)  98 amu

(b)  63 amu

(c)  32 amu

(d) 93 amu

Answer: (b)

82. In a crystalline solid, having formula XY2O4, oxide ions are arranged in cubic close packed lattice while, cations X are present is tetrahedral voids and cations Y are present in octahedral voids. The percentage of tetrahedral voids occupied by X is

(a)  12.5%

(b)  25%

(c)  50%

(d) 75%

Answer: (a)

83. Conversion of oxygen into ozone is non-spontaneous at

(a)  low temperature

(b)  all temperature

(c)  high temperature

(d) room temperature

Answer: (c)

84. A 100 mL flask contained H2 at 200 torr, and a 200 mL flask contained He at 100 torr. The two flask were then connected so that each gas filled their combined volume. Assume, no change in temperature, total pressure is

(a)  104 torr

(b)  163.33 torr

(c)  279 torr

(d) 133.33 torr

Answer: (d)

85. For the dot structure shown, the most likely elements X = … and Y = … are

(a)  carbon, fluorine

(b)  carbon, hydrogen

(c)  carbon, oxygen

(d) oxygen, carbon

Answer: (a)

86. Valence electrons in the element X are 3 and that in element Y is 6. Most probable compound formed from X and Y is

(a)  X2Y

(b)  XY2

(c)  X6Y3

(d) X2Y3

Answer: (d)

87. A compound, containing only carbon, hydrogen and oxygen has molecular weight of 44. On Complete oxidation, it is converted into a compound of molecular weight is 60. The original compound is

(a)  alcohol

(b)  acid

(c)  aldehyde

(d) ether

Answer: (c)

88. Number of photons emitted by a 100 W (Js1) yellow lamp in 1.0 s is

(λ of yellow light is 560 nm)

(a)  1.6 × 1018

(b)  1.4 × 1018

(c)  2.8 × 1020

(d) 2.1 × 1020

Answer: (c)

89. In Kjeldahl’s method, the nitrogen present in an organic compound in quantitatively converted into

(a)  gaseous ammonia

(b)  ammonium sulphate

(c)  ammonium phosphate

(d) ammonia

Answer: (d)

90. In the reversible reaction,

 the rate of disappearance of NO2 is equal to

(a) 

(b)  2k1[NO2]2 – 2k2[N2O4]

(c)  2k1[NO2]2 – k2[N2O4]

(d) (2k1 – k2) [NO2]

Answer: (b)

91. If E°(Zn2+, Zn) = −763 V and E°(Fe2+, Fe) = −0.44 V, then the emf of the cell Zn|Zn2+ (a = 0.001)|| Fe2+ (a = 0.005)|Fe is

(a)  equal to 0.323 V

(b)  less than 0.323 V

(c)  greater than 0.323 V

(d) equal to 1.103 V

Answer: (c)

92. For the electrochemical cell,

Ag|AgCl|KCl||AgNO3|Ag+, the overall cell reaction is

(a)  Ag+ + KCl → AgCl(s) + K+

(b) 

(c)  AgCl(s) → Ag+ + Cl

(d) Ag+ + Cl → AgCl(s)

Answer: (c)

93. Which one of the following electrolytes has the same value of van’t Hoff factor (i) as that of K4[Fe(CN)6]?

(a)  Al2(SO4)3

(b)  K2SO4

(c)  K3[Fe(CN)6]

(d) Al(NO3)3

Answer: (a)

94. The van’t Hoff factor for 0.1 M Ba(NO3)2 solution is 2.74. The degree of dissociation is

(a)  91.3%

(b)  87%

(c)  100%

(d) 74%

Answer: (b)

95. The IUPAC name of the following compound is

(a)  3-propylpent-1-ene

(b)  3-ethylpent-1-ene

(c)  4-ethylhex-1-ene

(d) 3-ethylhex-1-ene

Answer: (a)

96. Bleeding due to a cut can be stopped by applying ferric chloride solution in the laboratory. This is due to

(a)  coagulation of negatively charged blood particles by Fe3+ ions.

(b)  coagulation of positively charged blood particles by Cl ions.

(c)  reaction taking place between ferric ions and the haemoglobin forming a complex

(d) common element, iron, in both FeCl3 and haemoglobin.

Answer: (b)

97. Relative stabilities of the following carbocations will be in the order

(a)  C > B > A

(b)  C < B < A

(c)  B > C > A

(d) C > A > B

Answer: (d)

98. Among the following species, the one which become paramagnetic on heating at 850 K is

(a)  Fe3O4

(b)  MnO

(c)  ZnFe2O4

(d) Both (a) and (c)

Answer: (a)

99. Which of the following is not a surfactant?

(a)  CH3 – (CH2)15 – N+(CH3)3Br

(b)  CH3 – (CH2)14 – CH2NH2

(c)  CH3 – (CH216 – CH2OSO2 Na+

(d) OHC – (CH2)14 –CH2 – COONa+

Answer: (b)

100. In the complex, [Cr(NH3)4Cl2]+, the oxidation number of Cr is

(a)  +4

(b)  +3

(c)  +2

(d) +6

Answer: (b)

101. Arrange the following polymers in increasing order of their intermolecular forces.

Nylon-6, 6, buna-S, polythene, neoprene

(a)  Neoprene < buna-S < nylon-6, 6, < polythene

(b)  Neoprene < buna-S < polythene < nylon-6, 6

(c)  Polythene < nylon-6. 6 < buna-S < neoprene

(d) Polythene < buna-S < nylon-6, 6 < neoprene

Answer: (b)

102. The product of electrolysis of aqueous NaCl solution on anode and cathode respectively is

(a)  H2 and Cl2

(b)  Cl2 and H2

(c)  Na and Cl2

(d) Na and O2

Answer: (a)

103. In adiabatic conditions, 2 mole of CO2 gas at 300 K is expanded such that its volume becomes 27 times. Then , the work done is (CV = 6 cal mol1 and γ = 1.33)

(a)  1400 cal

(b)  1000 cal

(c)  900 cal

(d) 1200 cal

Answer: (d)

104. The compounds and are distinguished by

(a)  C6H5SO2Cl and OH in H2O

(b)  AgNO3 in H2O

(c)  dilute HCl

(d) HNO2 and β-naphthol

Answer: (d)

105. Alkali metals readily dissolved in liquid ammonia to give blue coloured solutions. The blue colour is believed to be due to

(a)  ammoniated cations

(b)  ammoniated anions

(c)  ammoniated electrons

(d) ammoniated cations and ammoniated electrons

Answer: (c)

106. 

What is the product ‘P’ is the above reaction?

(a)  Methyl ethanoate

(b)  Ethenoic acid

(c)  Propanal

(d) Methyl proppanoate

Answer: (a)

107. If osmotic pressure of 4% (w/v) solution of sucrose is same as 2% (w/v) solution of ‘X’, then the molecular mass of X (g/mol) is

(a)  171

(b)  205.2

(c)  570

(d) None of these

Answer: (a)

108. Which of the following is disproportionation reaction?

(a)  CH4 + 2O2 → CO2 + 2H2O

(b)  CH4 + 4Cl2 → CCl4 + 4HCl

(c)  2F2 + 2OH → 2F + OF2 + H2O

(d) 2NO2 + 2OH → NO2 + NO3 + H2O

Answer: (d)

109. Among the following the suitable reagent for Wurtz reaction is

(a)  Na/alcohol

(b)  Na/ether

(c)  Zn/ether

(d) Zn/alcohol

Answer: (b)

110. Which type of drugs come under anti-microbial drugs?

(a)  Antiseptics, antacids, antithista mines

(b)  Tranquilizers, antiseptics, antifertility

(c)  Tranquilizers, analgesics, antiseptics

(d) Antiseptics, antibiotics, disinfectants

Answer: (d)

111. Which of the following pair behaves as strong and weak base hydride respectively NH3, PH3, AsH3, SbH, BiH3

(a)  NH3, PH3

(b)  NH3, BiH3

(c)  PH3, BiH3

(d) BiH3, AsH3

Answer: (b)

112. Among the following the correct formula of Marshall’s acid is

(a)  H2S2O6

(b)  H2S2O8

(c)  H2SO4

(d) H2SO5

Answer: (b)

113. When hydrogen peroxide is added to acidified solution of dichromate Cr2O72, a deep blue coloured complex of chromic peroxide CrO5 is formed. Oxidation state of Cr in CrO5 is

(a)  +5

(b)  +3

(c)  +6

(d) −10

Answer: (c)

114. When phenol is treated with CHCl3 and NaOH followed by acidification, salicylaldehyde is obtained. Which of the following species is involved in the above mentioned reaction as intermediates?

(a) 

(b) 

(c) 

(d) 

Answer: (a)

115. Which of the following is the correct sequence of step involved in the mechanism of Cannizzaro reaction?

(a)  Nucleophilic attack, transfer of H and transfer of H+

(b)  Transfer of H, transfer of H+ and nucleophilic attack

(c)  Transfer of H+, nucleophilic attack and transfer of H

(d) Electrophilic attack by OH, transfer of H+ and transfer of H

Answer: (a)

116. 

What is the product is the above reacton?

(a) 

(b) 

(c) 

(d) 

Answer: (d)

117. Red blood cells are placed in a solution and neither a haemolysis nor crenation occurs. Therefore, the solution is

(a)  hypertonic

(b)  hypotonic

(c)  isotonic

(d) isotopic

Answer: (c)

118. If M is element of actinoids series, the degree of complex formation decreases in the order

(a)  M4+ > M3+ > MO22+ > MO2+

(b)  M4+ > MO22+ > M3+ > MO2+

(c)  MO2+ > MO22+ > M3+ > M4+

(d) MO22+ > MO2+ > M4+ > M3+

Answer: (b)

119. A chemical reaction was carried out at 300 K and 280 K. The rate constant were found to be k1 and k2 respectively, then

(a)  k2 = 4k1

(b)  k2 = 0.25 k1

(c)  k2 = 2k1

(d) k2 = 0.5k1

Answer: (b)

120. Which of the following compound can exhibit tautomerism?

(a) 

(b) 

(c) 

(d) 

Answer: (b)

Biology

121. Which of the following is not a derivative of cholesterol?

(a)  Vitamin-B

(b)  Vitamin-D

(c)  Bile salts

(d) Steroid

Answer: (a)

122. Shape of chloroplast of Ulothrix is

(a)  star-shaped

(b)  band-shaped

(c)  girdle-shaped

(d) spinal-shaped

Answer: (c)

123. What is the function of Kupffer’s cells?

(a)  Bile secretion

(b)  Digestion of lipid

(c)  Phagocytosis

(d) Digestion of protein

Answer: (c)

124. Which one is an anti-allergic antibody?

(a)  IgA

(b)  IgG

(c)  IgE

(d) IgD

Answer: (c)

125. Loss of water from body occurs by all of the following except

(a)  muscles

(b)  lungs

(c)  kidney

(d) skin

Answer: (a)

126. Pollenkitt is present in

(a)  anemophily

(b)  entomophily

(c)  malacophily

(d) zophily

Answer: (b)

127. Which motile-stage of p rotozoans is helpful in feeding?

(a)  Pseudopodium

(b)  Cilia

(c)  Flagella

(d) Tentacles

Answer: (a)

128. Function of hypothalamus is

(a)  themoregulation

(b)  water balance

(c)  control of hormone function

(d) All of the above

Answer: (d)

129. Inhibin is composed of

(a)  glycoprotein

(b)  lipoprotein

(c)  steroid

(d) amino acid derivative

Answer: (a)

130. What is ribotide?

(a)  Ribose + Uracil + Phosphate

(b)  Deoxyribose + Uracil + Phosphate

(c)  Deoxyribose + Thymine + Phosphate

(d) Ribose + Thymine + Phosphate

Answer: (a)

131. How many molecules of pyruvic acid are formed in glycolysis?

(a)  2

(b)  1

(c)  15

(d) 16

Answer: (a)

132. Molecular formula of chlorophyll-b is

(a)  C55H70O6N4Mg

(b)  C55H72O5N4Mg

(c)  C55H70O5N4Mg

(d) C54H70O6N4Mg

Answer: (a)

133. Non-essential amino acid is

(a)  valine

(b)  arginine

(c)  histidine

(d) lysine

Answer: (b)

134. Non-disjunction in meiosis results in

(a)  trisomy

(b)  normal diploid

(c)  gene mutation

(d) None of these

Answer: (a)

135. Caryopsis is present in

(a)  wheat

(b)  groundnut

(c)  coconut

(d) mango

Answer: (a)

136. AB blood group shows

(a)  codominance

(b)  incomplete dominance

(c)  polygenic inheritance

(d) pleiotropy

Answer: (a)

137. What is the symmetry of medusa?

(a)  Bilateral

(b)  Radial

(c)  Asymmetrical

(d) Biradial

Answer: (b)

138. Which one is parasitic alga?

(a)  Oedogonium

(b)  Cephaleuros

(c)  Spirogyra

(d) Cladophora

Answer: (b)

139. RQ of malic acid is

(a)  1.9

(b)  1.49

(c)  1.33

(d) 1

Answer: (c)

140. Which one is present in the urine of pregnant woman?

(a)  hCG

(b)  LH

(c)  Oestrogen

(d) FSH

Answer: (a)

141. 

Above diagram represents

(a)  anaphase-I

(b)  metaphase-I

(c)  telophase-I

(d) prophase-I

Answer: (a)

142. Histamine is secreted by

(a)  mast cells

(b)  Kupffer’s cells

(c)  oxyntic cells

(d) neutrophils

Answer: (a)

143. Peyer’s patches are present in

(a)  ileum

(b)  jejunum

(c)  duodenum

(d) sacculus rotandous

Answer: (a)

144. The correct sequence is

(a)  Zygota → Cleavage → Morula → Blastula → Gastrula

(b)  Cleavage → Zygote → Morula → Blastula → Gastrula

(c)  Zygote → Morula → Blastula → Cleavage → Gastrula

(d) Zygote → Blastula → Morula → Cleavage → Gastrula

Answer: (a)

145. Purkinje’s fibres are found in

(a)  heart

(b)  liver

(c)  brain

(d) lungs

Answer: (a)

146. Gene transfer is present in

(a)  biolistics

(b)  hybridisation

(c)  tissue culture

(d) vegetative propagation

Answer: (a)

147. Which one is used in tissue culture?

(a)  Explant

(b)  Somaclones

(c)  Hybridization

(d) None of these

Answer: (a)

148. What is common between a eukaryotic and prokaryotic flagella?

(a)  Same structure

(b)  Both are used for locomotion

(c)  Composed of same proteins

(d) Both are extension of cell membrane

Answer: (b)

149. Heterotrichous thallus is shown by which organism?

(a)  Chlamydomonas

(b)  Ectocarpus

(c)  Spirogyra

(d) Volvax

Answer: (b)

150. What is acrosomal reaction?

(a)  Contact of sperms with eggs

(b)  Digestion of zona pellucida

(c)  Disintegration of acrosome

(d) Contact of acrosome and nucleus of egg

Answer: (b)

151. Daily requirement of vitamin-A for adult women is

(a)  500 micrograms

(b)  700 micrograms

(c)  900 micrograms

(d) 300 micrograms

Answer: (b)

152. Which of these is incorrect for C4-plants?

(a)  Kranz anatomy

(b)  CO2 acceptor is PEP

(c)  PEPCase in mesophyll

(d) RuBisCO in mesophyll

Answer: (d)

153. Rouleaux formation is related to which of the cell/tissue?

(a)  RBCs

(b)  WBCs

(c)  Platelets

(d) Monocytes

Answer: (a)

154. Which of the following is not a plant growth inhibitor?

(a)  Dormin

(b)  IAA

(c)  Ethylene

(d) ABA

Answer: (b)

155. What is the RQ of glucose?

(a)  One

(b)  Less than one

(c)  More than one

(d) Infinite

Answer: (a)

156. Brunner’s gland is located in

(a)  duodenum

(b)  jejunum

(c)  ileum

(d) stomach

Answer: (a)

157. Perianth is found in which family?

(a)  Cruciferae

(b)  Solanaceae

(c)  Liliaceae

(d) Malvaceae

Answer: (c)

158. Which trait is shown by given pedigree analysis?

(a)  Autosomal dominant

(b)  X-Linked dominant

(c)  Autosomal recessive

(d) X-Linked recessive

Answer: (a)

159. Which of the following is involved in passive immunity?

(a)  IgA

(b)  IgE

(c)  IgM

(d) IgD

Answer: (a)

160. Fimbraie are associated with which organ?

(a)  Fallopian tube

(b)  Uterus

(c)  Vagina

(d) Ovary

Answer: (a)

161. Linker-DNA is attached to

(a)  H1

(b)  H2A

(c)  H2B

(d) H3

Answer: (a)

162. To which of the following, repressor protein is attached?

(a)  Operator

(b)  Inducer

(c)  Promoter

(d) Structural gene

Answer: (a)

163. Palmella stage is present in

(a)  Aspergillus

(b)  Cystopus

(c)  Chlamydomonas

(d) None of these

Answer: (c)

164. Formation of corpus luteum is induced by

(a)  LH

(b)  Oestrogen

(c)  FSH

(d) Progesterone

Answer: (a)

165. Cell wall of fungi is composed of

(a)  chitin

(b)  pectin

(c)  cellulose

(d) mannans

Answer: (a)

166. Which of the following mRNA can be transcripted as

(a)  AUG, UGA, UUU

(b)  UAA, UAV, UGG

(c)  UAG, UGA, UUV

(d) UGA, UUV, UGG

Answer: (a)

167. Which is incorrect with reference to chloroplast?

(a)  Presence in algae and plants

(b)  Releases O2

(c)  Occurs only in cells with aerobic respiration

(d) None of the above

Answer: (c)

168. Proteins are needed in diet because

(a)  all amino acids are not available in body

(b)  during fasting, body utilizes proteins

(c)  proteins act as building blocks of our body.

(d) All of the above

Answer: (d)

169. Which of the following pituitary hormones works indirectly?

(a)  MSH

(b)  TSH

(c)  GH

(d) Oxytocin

Answer: (b)

170. XYZ → composition

(a)  Super female

(b)  Hermaphrodite

(c)  Male

(d) Super male

Answer: (d)

171. Which is present at 5’ end of eukaryotic mRNA?

(a)  Poly-A tail

(b)  Modified C at 5’

(c)  7 mG

(d) Poly-C

Answer: (c)

172. Conditions required for cyclic photophosphorylation are

(a)  aerobic condition, low light intensity

(b)  aerobic, condition, optimum light intensity

(c)  anaerobic condition, low light intensity

(d) anaerobic condition, optimum light intensity

Answer: (b)

173. What is/are is the function(s) of calcium?

(a)  Blood clotting

(b)  Muscular contraction

(c)  Nerve conduction

(d) All of these

Answer: (d)

174. What is the role of sterol in cell membrane?

(a)  Stability

(b)  Communication with other cells

(c)  Secretion

(d) Transport

Answer: (a)

175. Where is sacculus rotandus located?

(a)  Between duodenum and jejunum

(b)  Between ileum and caecum

(c)  Caecum and colon

(d) Colon and rectum

Answer: (b)

176. 

Above diagram represents

(a)  metaphase-I

(b)  anaphase-I

(c)  metaphase-II

(d) anaphase-II

Answer: (a)

177. Omega 3 fatty acid is present in

(a)  sunflower oil

(b)  flax seed oil

(c)  groundnut oil

(d) butter

Answer: (b)

178. Which is the 21st amino acid?

(a)  Pyrrolysine

(b)  Selenocysteine

(c)  Cystine

(d) Histidine

Answer: (b)

179. What is incorrect about inhibin?

(a)  It is a lipoprotein

(b)  Decreases FSH secretion

(c)  Molecular weight is between 10k-30k Dalton

(d) Secreted by Sertoli cells

Answer: (a)

180. Eyespot is seen in

(a)  Chlamydomonas

(b)  Ulothrix

(c)  Spirogyra

(d) Polysiphonia

Answer: (a)

English and Quantitative Reasoning

Directions (Q. Nos. 181-183) Choose the word which best expresses the meaning of the underlined word in the sentence.

181. He used to regale us with antedotes.

(a)  box

(b)  flatter

(c)  entertain

(d) tire

Answer: (c)

182. Millionaires become accentric in their old age.

(a)  irritable

(b)  peculiar

(c)  dull

(d) miserly

Answer: (b)

183. The date complied by the organization was very useful.

(a)  analysed

(b)  enclosed

(c)  collected

(d) published

Answer: (c)

Direction (Q. Nos. 184-186) Fill in the blanks.

184. Some regions of our country still remain ….. to the average man.

(a)  inaccessible

(b)  impossible

(c)  impermeable

(d) invincible

Answer: (a)

185. The employees are entitled to ………. dividends from the annual surplus.

(a)  submit

(b)  deposit

(c)  dedcut

(d) receive

Answer: (d)

186. His arrogance is preceeded only by his ……… ignorance.

(a)  eternal

(b)  abysmal

(c)  meteoric

(d) diabolic

Answer: (b)

Direction (Q. Nos. 187-190) Choose the word which is closest to the opposite in meaning of the underlined word in the sentence.

187. The treaty was ratified by the heads of states.

(a)  set aside

(b)  unsettled

(c)  destroyed

(d) annulled

Answer: (d)

188. Many people try to resist reforms in the society.

(a)  repel

(b)  welcome

(c)  accept

(d) fight

Answer: (b)

189. He was always prepared to refute the idea.

(a)  agree

(b)  subscribe

(c)  recommend

(d) endorse

Answer: (d)

190. We should not belittle others achievements.

(a)  recommend

(b)  preuse

(c)  encourage

(d) inspire

Answer: (c)

191. If GOODNESS is coded as HNPCODTR, then how GREATNESS can be written in that code?

(a)  HQZSMFRT

(b)  HQFZUFRTM

(c)  HQFZUODTR

(d) HQFZUMFRT

Answer: (d)

192. Find the missing number in the series given below

40, 30, 22, 16, ?

(a)  10

(b)  12

(c)  15

(d) 17

Answer: (b)

193. What comes in place of question mark (?) in the series given below?

P3C, R5F, T8I, V12L, ?

(a)  Y17O

(b)  X16M

(c)  X17O

(d) X16O

Answer: (c)

194. Find the missing number in the following figure.

(a)  13

(b)  15

(c)  17

(d) 19

Answer: (b)

195. There are 25 boys in a horizontal row. Rahul was shifted by three laces towards his right side and he occupies the middle position in the row. What was his original position from the left end of the row?

(a)  15th

(b)  16th

(c)  12th

(d) 10h

Answer: (d)

196. Pointing to a man in a photograph, a woman said, “His brother’s father is the only son of my grandfather.” How is the woman related to the man in the photograph?

(a)  Mother

(b)  AUnt

(c)  Sister

(d) Daughter

Answer: (c)

197. Five villages P, Q, R, S and T situated close to each other. P is to the West of Q, R is to the South of P, T is to the North of Q and S is to the East of T. Then, in which direction R is with respect to S?

(a)  North-West

(b)  South-East

(c)  South-West

(d) Data inadequate

Answer: (c)

198. If the day after tomorrow is a Sunday, what was it day before yesterday?

(a)  Wednesday

(b)  Thursday

(c)  Friday

(d) Saturday

Answer: (a)

199. Four positions of a dice given below. Identify the number at the bottom when top is 6.

(a)  1

(b)  3

(c)  4

(d) 5

Answer: (a)

200. Choose the figure from the given answer figures that will complete the pattern of the given problem figure.

Answer: (a)

 

JIPMER Medical Entrance Exam Previous Year Question Paper 2009 With Answer Key

JIPMER MBBS Entrance Exam – 2009

Physics 

1. In radioactive decay process, the negatively charged emitted β-particles are

(a)   the electrons present inside the nucleus

(b)   the electrons produced as a result of the decay of neutrons inside the nucleus

(c)   the electrons produced as a result of collisions between atoms

(d)   the electrons orbiting around the nucleus

Answer: (b)

2. The maximum number of possible interference maxima for slit-separation equal to twice the wavelength in Young’s double-slit experiment, is

(a)   infinite

(b)   five

(c)   three

(d)   zero

Answer: (b)

3. Two spherical conductors B and C having equal radii and carrying equal charges in them repel each other with a force F when kept apart at some distance. A third spherical conductor having same radius as that of B but uncharged, is brought in contact with B, then brought in contact with C and finally removed away from both. The new force of repulsion between B and C is

(a)   F/4

(b)   3F/4

(c)   F/8

(d)   3F/8

Answer: (d)

4. In gamma ray emission from a nucleus

(a)   both the neutron number and the proton number change

(b)   there is no change in the proton number and the neutron number

(c)   only the neutron number changes

(d)   only the proton number changes

Answer: (b)

5. A particle starting from the origin (0, 0) moves in a straight line in the (x, y) plane. Its coordinates at a later time are (√3, 3). The path of the particle makes with the x-axis an angle of

(a)   30°

(b)   45°

(c)   60°

(d)   0°

Answer: (c)

6. A wheel has angular acceleration of 30 rad/s2 and an initial angular speed of 2.00 rad/s. In a time of 2 s it has rotated through the angle (in radian) of

(a)   6

(b)   10

(c)   12

(d)   4

Answer: (b)

7. The resistance of an ammeter is 13 Ω and its scale is graduated for a current upto 100 A. After an additional shunt has been connected to this ammeter it becomes possible to measure currents upto 750 A by this meter. The value of shunt resistance is

(a)   20 Ω

(b)   2 Ω

(c)   0.2 Ω

(d)   2 kΩ

Answer: (b)

8. Under the influence of a uniform magnetic field a charged particle is moving in a circle of radius R with constant speed v. The time period of the motion

(a)   depends on v and not on R

(b)   depends on both R and v

(c)   is independent of both R and v

(d)   depends on R and not on v

Answer: (c)

9. The primary and secondary coils of a transformer have 50 and 1500 turns respectively. If the magnetic flux ϕ linked with the primary coil is given by ϕ0 + 4t, where ϕ is in weber, t is time in second and ϕ0 is constant, the o utput voltage across the secondary coil is

(a)   90 V

(b)   120 V

(c)   220 V

(d)   30 V

Answer: (b)

10. The frequency of a light wave in a material is 2 × 104 Hz and wavelength is 5000 Å. The refractive index of material will be

(a)   1.40

(b)   1.50

(c)   3.00

(d)   1.33

Answer: (c)

11. A car moves from X to Y with a uniform speed vu and returns to Y with a uniform speed vd. The average speed for this round trip is

(a) 

(b) 

(c) 

(d) 

Answer: (a)

12. A particle executes simple harmonic oscillation with an amplitude a. The period of oscillation is T. The minimum time taken by the particle to travel half of the amplitude from the equilibrium position is

(a)   T/4

(b)   T/8

(c)   T/12

(d)   T/2

Answer: (c)

13. A block B is pushed momentarily along a horizontal surface with an initial velocity v. If μ is the coefficient of sliding friction between B and the surface, block B will come to rest after a time

(a)   v/gμ

(b)   gμ/v

(c)   g/v

(d)   v/g

Answer: (a)

14. Two radioactive substances A and B have decay constants 5λ and λ respectively. At t = 0 they have the same number of nuclei. The ratio of number of nuclei of A to those of B will be (1/e)2 after a time interval

(a)   1/4λ

(b)   4λ

(c)   2λ

(d)   1/2λ

Answer: (d)

15. A transformer is used to light a 100 W and 110 V lamp from a 220 V mains. If the main current is 0.5 A, the efficiency of the transformer is approximately

(a)   30%

(b)   50%

(c)   90%

(d)   10%

Answer: (c)

16. A charged particle (charge q) is moving in a circle of radius R with uniform speed v. The associated magnetic moment μ is given by

(a)   qvR/2

(b)   qvR2

(c)   qvR2/2

(d)   qvR

Answer: (a)

17. A steady current of 1.5 A flows through a copper voltameter for 10 min. If the electrochemical equivalent of copper is 30 × 10−5 gC−1, the mass of copper deposited on the electrode will be

(a)   0.40 g

(b)   0.50 g

(c)   0.67 g

(d)   0.27 g

Answer: (d)

18. In a mass spectrometer used for measuring the masses of ions, the ions are initially accelerated by an electric potential V and then made to describe semicircular paths of radius R using a magnetic field B. If V and B are kept constant, the ratio  will be proportional to

(a)   1/R

(b)   1/R2

(c)   R2

(d)   R

Answer: (b)

19. Three resistances P, Q, R each of 2 Ω and an unknown resistance S form the four arms of a Wheatstone’s bridge circuit. When a resistance of 6 Ω is connected in parallel to S the bridge gets balanced. What is the value of S?

(a)   2 Ω

(b)   3 Ω

(c)   6 Ω

(d)   1 Ω

Answer: (b)

20. Two satellites of earth, S1 and S2, are moving in the same orbit. The mass of S­1 is four times the mass of S2. Which one of the following statements is true?

(a)   The time period of S1 is four times that of S2

(b)   The potential energies of earth and satellite in the two cases are equal

(c)   S1 and S2 are moving with the same speed

(d)   The kinetic energies of the two satellites are equal

Answer: (c)

21. An observer moves towards a stationary source of sound, with a velocity one-fifth of the velocity of sound. What is the percentage increase in the apparent frequency?

(a)   Zero

(b)   0.5%

(c)   5%

(d)   20%

Answer: (a)

22. A coil of inductance 300 mH and resistance 2 Ω is connected to a source of voltage 2 V. The current reaches half of its steady state value in

(a)   0.05 s

(b)   0.1 s

(c)   0.15 s

(d)   0.3 s

Answer: (b)

23. The refractive index of glass is 1.520 for red light and 1.525 for blue light. Let D1 and D2 be angles of minimum deviation for red and blue light respectively in a prism of this glass, then

(a)   D1 < D2

(b)   D1 = D2

(c)   D1 can be less than or greater than D2 depending upon the angle of prism

(d)   D1 > D2

Answer: (a)

24. A particle of mass 100 g is thrown vertically upwards with a speed of 5 m/s. The work done by the force of gravity during the time the particle goes up is

(a)   −0.5 J

(b)   −1.25 J

(c)   1.25 J

(d)   0.5 J

Answer: (b)

25. A mass of M kg is suspended by a weight less string. The horizontal force that s required to displace it until the string makes an angle of 45° with the initial vertical direction is

(a)   Mg(√2 + 1)

(b)   Mg√2

(c)   Mg/√2

(d)   Mg(√2 – 1)

Answer: (d)

26. A force of  acts on O, the origin of the coordinate system. The torque about the point (1, −1) is

(a) 

(b) 

(c) 

(d) 

Answer: (c)

27. If MO is the mass of an oxygen isotope 8O17, Mp and Mn are the masses of a proton and a neutron, respectively, the nuclear binding energy of the isotope is

(a)   (MO – 8Mp) c2

(b)   (MO – 8Mp – 9Mn) c2

(c)   MOc2

(d)   (MO – 17Mn) c2

Answer: (b)

28. A sound absorber attenuates the sound level by 20 dB. The intensity decreases by a factor of

(a)   1000

(b)   10000

(c)   10

(d)   100

Answer: (d)

29. Which of the following parameters does not characterize the thermodynamic state of matter ?

(a)   Temperature

(b)   Pressure

(c)   Work

(d)   Volume

Answer: (c)

30. A charged oil drop is suspended in uniform m field of 3 × 104 V/m so that it neither falls nor rises. The charge on the drop will be (Take the mass of charge = 9.9 × 10−15 kg and g = 10 m/s2)

(a)   3.3 × 10−18 C

(b)   3.2 × 10−18 C

(c)   1.6 × 10−18 C

(d)   4.8 × 10−18 C

Answer: (a)

31. Dimensions of resistance in an electrical circuit, in terms of dimension of mass M, of length L, of time T and of current I, would be

(a)   [ML2T−3T−1]

(b)   [ML2T−2]

(c)   [ML2T−1I−1]

(d)   [ML2T−3I−2]

Answer: (d)

32. An alpha nucleus of energy  bombards a heavy nuclear target of charge Ze. Then the distance of closest approach for the alpha nucleus will be  proportional to

(a)   v2

(b)   1/m

(c)   1/v4

(d)   1/Ze

Answer: (b)

33. The work of 146 kJ is performed in order to compress on kilo mole of a gas adiabatically and in this process the temperature of the gas increases by 7°C. The gas is

(R = 8.3 J mol−1 K−1)

(a)   diatomic

(b)   triatomic

(c)   a mixture of monoatomic and diatomic

(d)   monoatomic

Answer: (a)

34. Diwali rocket is ejecting 50 g of gases/s at a velocity of 400 m/s. The accelerating force on the rocket will be

(a)   22 dyne

(b)   20 N

(c)   20 dyne

(d)   100 N

Answer: (b)

35. A frame made of metallic wire enclosing a surface area A is covered with a soap film. If the area of the frame of metallic wire is reduced by 50%, the energy of the soap film will be changed by

(a)   100%

(b)   75%

(c)   50%

(d)   25%

Answer: (c)

36. Mercury boils at 367°C. However, mercury thermometers are made such that they can measure temperature upto 500°C. This is done by

(a)   maintaining vacuum above mercury column in the stem of the thermometer

(b)   filling nitrogen gas at high pressure above the mercury column

(c)   filling oxygen gas at high pressure above the mercury column

(d)   filling nitrogen gas at low pressure above the mercury column

Answer: (b)

37. In a laboratory four convex lenses L1, L2, L3 and L4 of focal lengths 2, 4, 6 and 8 cm, respectively are available. Two of these lenses form a telescope of length 10 cm and magnifying power 4. The objective and eye lenses are respectively

(a)   L2, L3

(b)   L1, L4

(c)   L1, L2

(d)   L4, L1

Answer: (d)

38. A symmetric double convex lens is cut in two equal parts by a plane perpendicular to the principal axis. If the power of the original lens is 4D, the power of a cut lens will be

(a)   2D

(b)   3D

(c)   4D

(d)   5D

Answer: (a)

39. For a metallic wire, the ratio V/i (V = applied potential difference and i = current flowing) is

(a)   independent of temperature

(b)   increases as the temperature rises

(c)   decreases as the temperature rises

(d)   increases or decreases as temperature rises depending upon the metal

Answer: (b)

40. The potential energy of a molecule on the surface of a liquid compared to one inside the liquid is

(a)   zero

(b)   lesser

(c)   equal

(d)   greater

Answer: (d)

JIPMER MBBS Entrance Exam – 2009

CHEMISTRY

1. Which of the following is not correct?

(a)   Hydrolysis of NCl3 gives NH3 and HOCl

(b)   NH3 is less stable than PH3

(c)   NH3 is a weak reducing reagent compared to PH3

(d)   Nitric oxide in solid state exhibits diamagnetic property

Answer: (b)

2. SiO2 is reacted with sodium carbonate. What is the gas liberated?

(a)   CO

(b)   O2

(c)   CO2

(d)   O­3

Answer: (c)

3. The compound formed at anode in the electrolysis of an aqueous solution of potassium acetate, are

(a)   C2H6 and CO2

(b)   C2H4 and CO2

(c)   CH4 and H2

(d)   CH4 and CO2

Answer: (a)

4. Which of the following is not correct regarding the elecolytic preparation of H2O2?

(a)   Lead is used as cathode

(b)   50% of H2SO­4 is used

(c)   Hydrogen is liberated at anode

(d)   Sulphuric acid undergoes oxidation

Answer: (c)

5. Which of the following is correct?

(a)   The pH of one litre solution containing 0.49 g of H2SO4 is 2.0

(b)   The conjugate base of H2S is S2−

(c)   BF3 is a Lewis base

(d)   Phenolphthalein is colourless in basic medium

Answer: (a)

6. Which of the following is correct?

(a)   Catalyst undergoes permanent chemical change

(b)   Particle size of solute in true solution is 10−3 m

(c)   Starch solution is a hydrosol

(d)   Hydrolysis of liquid ester in the presence of mineral acid is an example of heterogeneous catalysis reactions

Answer: (c)

7. In an oxidation-reduction reaction, MnO4 ion is converted to Mn2+. What is the number of equivalents of KMnO4 (mol. wt. = 158) present in 250 mL of 0.04 M KMnO4 solution ?

(a)   0.02

(b)   0.05

(c)   0.04

(d)   0.07

Answer: (b)

8. Which of the following reagents converts both acetaldehyde and acetone to alkanes?

(a)   Ni/H2

(b)   LiAlH4

(c)   I2/NaOH

(d)   Zn-Hg/conc. HCl

Answer: (d)

9. The heat of formation of CO(g) and CO2(g) are ∆H = −110 and ∆H = −393 kJ mol−1 What is the heat of reaction (∆H) (in kJ mol−1) for the following reaction ?

(a)   −504

(b)   −142.5

(c)   −283

(d)   504

Answer: (c)

10. What is the wavelength (in m) of a particle of mass 6.62 × 10−29 g moving with a velocity o f 103 ms−1 ?

(a)   6.62 × 10−4

(b)   6.62 × 10−3

(c)   10−5

(d)   105

Answer: (c)

11. What is the electrode potential (in V) of the following electrode at 25°C?

Ni2+ (0.1 M) | Ni(s)

(Standard reaction potential of Ni2+ |Ni is 

(a)   −0.28 V

(b)   −0.34 V

(c)   −0.82 V

(d)   −0.22 V

Answer: (a)

12. What is the equation for the equilibrium constant (K­c) for the following reaction?

(a) 

(b) 

(c) 

(d) 

Answer: (c)

13. Which of the following can give a Grignard reagent when reacted with magnesium in dry ether?

(a)   C2H6

(b)   C2H5Cl

(c)   C2H5OH

(d)   C2H5CN

Answer: (b)

14. Which of the following is not correct?

(a)   Al reacts with NaOH and liberate H2

(b)   AlCl3 is a Lewis acid

(c)   Al is used in the manufacture of electrical cables

(d)   NaOH is used during Hall’s process of purification of bauxite

Answer: (d)

15. A 0.5 g/L solution of glucose is found to be isotonic with a 2.5 g/L solution of an organic compound. What will be the molecular weight of that organic compound ?

(a)   300

(b)   600

(c)   900

(d)   200

Answer: (c)

16. t-butyl chloride preferably undergo hydrolysis by

(a)   SN1 mechanism

(b)   SN2 mechanism

(c)   Any of (a) and (b)

(d)   None of these

Answer: (a)

17. Oxidation state of oxygen in F2O is

(a)   +1

(b)   −1

(c)   +2

(d)   −2

Answer: (c)

18. To dissolve argentite ore which of the following is used?

(a)   Na[Ag(CN)2]

(b)   NaCN

(c)   NaCl

(d)   HCl

Answer: (b)

19. If 50% of a radioactive substance dissociates in 15 min, then the time taken by substance to dissociate 99% will be

(a)   50 min

(b)   100 min

(c)   99 min

(d)   150 min

Answer: (c)

20. H – O – H bond angle in H2O is 104.5° and not 109° 28’ because of

(a)   lone pair-lone pair repulsion

(b)   lone pair-bond pair repulsion

(c)   bond pair-bond pair repulsion

(d)   high electronegativity of oxygen

Answer: (a)

21. The reaction,

C6H5CHO + CH3COOC2H5 → C6H5CH = CHCOOC2H5, is called

(a)   Benzoin condensation

(b)   Claisen condensation

(c)   Cannizaro’s reaction

(d)   Perkin reaction

Answer: (b)

22. The best method to separate the mixture of ortho and para nitrophenol (1 : 1) is

(a)   vaporization

(b)   colour spectrum

(c)   distillation

(d)   crystallization

Answer: (c)

23. Iodofrom gives a precipitate with AgNO3 on heating but chloroform does not because

(a)   C – I bond in iodoform is weak and C – Cl bond in chloroform is strong

(b)   chloroform is covalent

(c)   iodoform is ionic

(d)   None of the above

Answer: (a)

24. What are the values of n1 and n2 respectively for Hβ line in the Lyman series of hydrogen atomic spectrum 44 ?

(a)   3 and 5

(b)   2 and 3

(c)   1 and 3

(d)   2 and 4

Answer: (c)

25. The homologue of ethyne is

(a)   C2H2

(b)   C2H6

(c)   C3H8

(d)   C3H4

Answer: (d)

26. A 0.1 aqueous solution of a weak acid is 2% ionized. If the ionic product of water is 1 × 10−4, the [OH] is

(a)   5 × 10−12 M

(b)   2 × 10−3 M

(c)   1 × 10−14 M

(d)   None of these

Answer: (a)

27. Which of the following does not have coordinate bond?

(a)   SO2

(b)   HNO­3

(c)   H2SO3

(d)   HNO2

Answer: (c)

28. The total number of orbitals in the fifth energy level is

(a)   5

(b)   10

(c)   18

(d)   25

Answer: (d)

29. The most probable velocity (in cm/s) of hydrogen molecule at 27°C will be

(a)   19.3 × 104

(b)   17.8 × 104

(c)   24.93 × 109

(d)   17.8× 108

Answer: (b)

30. In III group precipitation, NH4Cl is added before adding NH4OH to

(a)   decrease conc. of OH

(b)   prevent interference of PO43−

(c)   increase conc. of Cl

(d)   increase conc. of OH ion

Answer: (a)

31. Steel is heated to below red heat and then cooled slowly. The process refers to

(a)   hardening

(b)   annealing

(c)   tempering

(d)   nitriding

Answer: (b)

32. What is the wave number of 4th line in Balmer series of hydrogen spectrum?

(R = 1,09,677 cm−1)

(a)   24, 630 cm−1

(b)   24,360 cm−1

(c)   24,730 cm−1

(d)   24,372 cm−1

Answer: (d)

33. 2 g N2O4 is heated in a 1 L vessel till equilibrium state is established

N2O4(g) ⇌ 2NO2(g)

In equilibrium state 50% N2O4 was dissociated, equilibrium constant will be

(mol. wt. of N2O4 = 92)

(a)   0.1

(b)   0.4

(c)   0.3

(d)   0.2

Answer: (d)

34. Disperse phase and dispersion medium in butter are respectively

(a)   solid and liquid

(b)   liquid and solid

(c)   liquid and liquid

(d)   solid and solid

Answer: (b)

35. Which of the following carbonates decomposes readily at low temperatures?

(a)   NaCO3

(b)   K2CO3

(c)   Li2CO3

(d)   Rb2CO3

Answer: (c)

36. The atomic number of an element ‘M’ is 26. How many electrons are present in M-shell of the element in its M3+ state ?

(a)   11

(b)   15

(c)   14

(d)   13

Answer: (d)

37. In which of the following pairs, both molecules possess dipole moment?

(a)   CO2, SO2

(b)   BCl3, PCl3

(c)   H2O, SO2

(d)   CO2, CS2

Answer: (c)

38. Which one of the following reactions is called Rosenmund reaction?

(a)   Aldehydes are reduced to alcohols

(b)   Acids are converted to acid chlorides

(c)   Alcohols are reduced to hydrocabons

(d)   Acid chlorides are reduced to aldehydes

Answer: (d)

39. During, acetylation of amines, what is replace by acetyle group ?

(a)   Hydrogen atom attached to nitrogen atom

(b)   One or more hydrogen atoms attached to carbon atom

(c)   One or more hydrogen atoms attached to nitrogen atom

(d)   Hydrogen atoms attached to either carbon atom or nitrogen atom

Answer: (c)

40. Which is used in alcoholic beverages?

(a)   Methanol

(b)   Ethanol

(c)   Phenol

(d)   Glycerol

Answer: (b)

JIPMER MBBS Entrance Exam – 2009

ZOOLOGY

1. The trisomy for 21st chromosome is called

(a)   Down’s syndrome

(b)   Turner’s syndrome

(c)   Sickle cell anaemia

(d)   Klinefelter’s syndrome

Answer: (a)

2. Which of the following is the largest gland in an adult man ?

(a)   Thymus

(b)   Liver

(c)   Thyroid

(d)   Pancreas

Answer: (b)

3. In Pheretima, septa are absent between which segments?

(a)   3/4 and 9/10

(b)   4/5 and 8/9

(c)   5/6 and 7/8

(d)   7/8 and 6/7

Answer: (a)

4. During emergency which of the following hormone is secreted?

(a)   Aldosterone

(b)   Thyroxine

(c)   Adrenalin

(d)   Calcitonin

Answer: (c)

5. In cockroach, larval and nymphal characters are maintained by

(a)   ecdysone

(b)   salivary glands

(c)   parotid gland

(d)   juvenile hormone

Answer: (d)

6. Which of the following is a transparent tissue?

(a)   Tendon

(b)   Fibrous cartilage

(c)   Hyaline cartilage

(d)   All of these

Answer: (c)

7. Rh factor is present in

(a)   all vertebrates

(b)   all mammals

(c)   all reptiles

(d)   man and rhesus monkey only

Answer: (d)

8. In rabbit, end of a long bone is connected in another by

(a)   tendon

(b)   ligaments

(c)   muscle

(d)   cartilage

Answer: (b)

9. Which of the following cell type is capable of giving rise to other cell types in sponges ?

(a)   Thesocytes

(b)   Pinacocytes

(c)   Cnidocytes

(d)   Archaeocytes

Answer: (d)

10. Thigmotaxis is not shown by

(a)   Paramecium

(b)   Amoeba

(c)   Ascaris

(d)   Hydra

Answer: (c)

11. Which is correctly matched?

(a)   Apiculture – Honey bee

(b)   Pisciculture – Silk moth

(c)   Sericulture – Fish

(d)   Aquaculture – Mosquito

Answer: (a)

12. Animals having a built in thermostat to maintain constant body temperature are known as

(a)   biothermic

(b)   poikilothermic

(c)   oligothermic

(d)   homeothermic

Answer: (d)

13. The intermediate host of Schistosoma is

(a)   snail

(b)   mosquito

(c)   housefly

(d)   sheep

Answer: (a)

14. The vitamin which is essential for blood clotting is

(a)   vitamin-A

(b)   vitamin-B

(c)   vitamin-C

(d)   vitamin-K

Answer: (d)

15. The female genital pore of Pheretima posthuma located upon the segment

(a)   14th

(b)   16th

(c)   18th

(d)   15th

Answer: (a)

16. Polyp phase is absent in

(a)   Hydra

(b)   Aurelia

(c)   Physalia

(d)   Obelia

Answer: (b)

17. In frong heart, there are cardiac muscles which consists of fibres called

(a)   Purkinje fibres

(b)   myonemes

(c)   telodendria

(d)   columnae carnae

Answer: (a)

18. Malpighian tubules are

(a)   excretory organs of insects

(b)   excretory organs of frog

(c)   respiratory organs of insects

(d)   endocrine glands of insects

Answer: (a)

19. LH and FSH are collectively called

(a)   oxytocin

(b)   somatotropins

(c)   luteotropic

(d)   gonadotropins

Answer: (d)

20. Which of the following provides most evident proof evolution?

(a)   Fossils

(b)   Morphology

(c)   Embryo

(d)   Vestigial organs

Answer: (a)

21. In Mollusca, eye is present over a stalk, called

(a)   ostracum

(b)   operculum

(c)   ommatophores

(d)   osphradium

Answer: (c)

22. Turbellarians are free living

(a)   nematodes

(b)   cestodes

(c)   flat worms

(d)   trematodes

Answer: (c)

23. The characteristic larva of phylum-‘Coelenterata’ is

(a)   planula

(b)   cysticercus

(c)   rhabdiform

(d)   wriggler

Answer: (a)

24. In rabbit, head of epididymis present at the head of the testis is called

(a)   vas deferens

(b)   cauda epididymis

(c)   gubernaculum

(d)   caput epididymis

Answer: (d)

25. In blood,CO2 in transported majorly as

(a)   sodium carbonate

(b)   carboxyhaemoglobin

(c)   bicarbonate

(d)   CO2 as such

Answer: (c)

26. Animals undergoes inactive stage during winter, is known as

(a)   aestivation

(b)   hibernation

(c)   adaptation

(d)   acclimatization

Answer: (b)

27. Kupffer cells are present in

(a)   liver

(b)   small intestine

(c)   pancreas

(d)   thyroid gland

Answer: (a)

28. The embryo at 16 celled stage is known as

(a)   morula

(b)   gastrula

(c)   blastula

(d)   blastomere

Answer: (a)

29. Contractile vacuole in protozoan Amoeba is meant for

(a)   respiration

(b)   excretion

(c)   locomotion

(d)   osmoregulation

Answer: (d)

30. Which of the following is important for muscle contraction and nerve impulse transmission?

(a)   Ca2+ ions

(b)   Mg2+ ions

(c)   Both (a) and (b)

(d)   Fe2+ ions

Answer: (a)

31. Which one is component of Ornithine cycle?

(a)   Ornithine, citrulline and alanine

(b)   Ornithine, citruline and arginine

(c)   Amino acid are not used

(d)   Ornithine, citrulline and fumaric acid

Answer: (b)

32. Chromosome complement with 2n – 1 is called

(a)   monosomy

(b)   nullisomy

(c)   trisomy

(d)   tetrasomy

Answer: (a)

33. Which of the following is not vestigial in man?

(a)   Tail vertebrae

(b)   Nails

(c)   Nictitating membrane

(d)   Vermiform appendix

Answer: (b)

34. Small fish get stuck near the bottom of a shark and derives its nutrition from it. This kind of association is called as

(a)   antibiosis

(b)   commensalism

(c)   predation

(d)   parasitism

Answer: (b)

35. The group of Anamniota includes

(a)   reptiles and birds

(b)   birds and mammals

(c)   fishes an amphibians

(d)   reptiles and mammals

Answer: (c)

36. The excretory material of bony fish is

(a)   urea

(b)   protein

(c)   ammonia

(d)   amino acid

Answer: (a)

37. The leucocytes contain which of the following in large quantity?

(a)   Basophils

(b)   Neutrophils

(c)   Eosinophils

(d)   Monocytes

Answer: (b)

38. During inspiration, the diaphragm

(a)   expands

(b)   shows no change

(c)   contracts and flattens

(d)   relaxes to become dome-shaped

Answer: (c)

39. The function of pineal body is to

(a)   lighten the skin colours

(b)   control sexual behaviour

(c)   regulates the period of puberty

(d)   All of the above

Answer: (d)

40. Synsacrum of flowl consists of about

(a)   29 vertebrae

(b)   3 vertebrae

(c)   16 vertebrae

(d)   single vertebrae

Answer: (c)

JIPMER MBBS Entrance Exam – 2009

BOTANY

1. Jumping genes in maize were discovered by

(a)   Hugo de Vries

(b)   Barbara McClintock

(c)   T H Morgan

(d)   Mendel

Answer: (b)

2. Streptomycin is obtained from

(a)   Streptomyces griseus

(b)   S. aureofaciens

(c)   S. venezuelae

(d)   S. ramosus

Answer: (a)

3. Indusium is found in

(a)   algae

(b)   ferns

(c)   moss

(d)   Cycas

Answer: (b)

4. The vacuole is lined by a membrane called

(a)   tonoplast

(b)   jacket

(c)   cell membrane

(d)   tonoplasm

Answer: (a)

5. Agar-agar is obtained from

(a)   Chlorella

(b)   Spirogyra

(c)   Ulothrix

(d)   Gelidium

Answer: (d)

6. DNA element with ability to change position is called

(a)   cistron

(b)   transposon

(c)   intron

(d)   recon

Answer: (b)

7. Initiation codon is

(a)   UUU

(b)   UGA

(c)   AUG

(d)   UAG

Answer: (c)

8. DNA multiplication is called

(a)   translation

(b)   replication

(c)   transduction

(d)   transcription

Answer: (b)

9. Duramen is present in

(a)   inner region of secondary wood

(b)   part of sap wood

(c)   outer region of secondary wood

(d)   region of pericycle

Answer: (a)

10. In plants, water supply is due to

(a)   osmosis

(b)   imbibition

(c)   guttation

(d)   adhesion force

Answer: (d)

11. Most of the economically important fibre yielding plants belong to family

(a)   Malvaceae

(b)   Solanaceae

(c)   Cruciferae

(d)   Poaceae

Answer: (a)

12. Paraffin wax is

(a)   ester

(b)   acid

(c)   monohydric alcohol

(d)   cholesterol

Answer: (a)

13. Which is always present in photochemical smog?

(a)   O3

(b)   CO2

(c)   SO2

(d)   CH4

Answer: (a)

14. In cell cycle, during which phase, chromosomes are arranged in equatorial plate?

(a)   Metaphase

(b)   Anaphase

(c)   Telophase

(d)   Prophase

Answer: (a)

15. The soil which is transported by wind is known as

(a)   colluvial

(b)   eolian

(c)   aluvial

(d)   glacial soil

Answer: (b)

16. Spindle fibre is made up of

(a)   tubulin

(b)   humulin

(c)   intermediate filament

(d)   flagellin

Answer: (a)

17. Lichen is the pioneer vegetation on which succession?

(a)   Hydrosere

(b)   Lithosere

(c)   Psammosere

(d)   Xerosere

Answer: (b)

18. In Pinus, male cone bears a large number of

(a)   ligules

(b)   anthers

(c)   micro-sporophylls

(d)   mega-sporophylls

Answer: (c)

19. Induction of flowering by low temperature treatment is

(a)   vernalization

(b)   cryobiology

(c)   photoperiodism

(d)   prunning

Answer: (a)

20. Decomposers are

(a)   autotrophs

(b)   autoheterotrophs

(c)   organotrophs

(d)   heterotrophs

Answer: (c)

21. Cleavage polyembryony occurs in

(a)   Pinus

(b)   Mini Cycas

(c)   Cycas

(d)   Ephedra

Answer: (a)

22. Edible part of mushroom is

(a)   basidiocarp

(b)   secondary mycelium

(c)   primary mycelium

(d)   tertiary mycelium

Answer: (a)

23. Calyptra is derived from

(a)   archegonia

(b)   capsule

(c)   antheridia

(d)   columella

Answer: (a)

24. ‘Clamp connections’ are observed in

(a)   Basidiomycetes

(b)   Zygomycetes

(c)   Ascomycetes

(d)   Oomycetes

Answer: (a)

25. What is the main cause for the extinction of  some species in tropical forest?

(a)   Deforestation

(b)   Afforestation

(c)   Pollution

(d)   Soil erosin

Answer: (a)

26. Most accepted theory for ascent of sap is

(a)   capillarity theory

(b)   root pressure theory

(c)   pulsation theory

(d)   transpiration pull

Answer: (d)

27. Which of the following is not the feature of gymnosperms?

(a)   Parallel venation

(b)   Perennial plants

(c)   Distinct branches (long and short branches)

(d)   Xylem with vessels

Answer: (d)

28. The presence of diversity at the junction of territories of two different habitats is known as

(a)   bottle neck effect

(b)   edge effect

(c)   junction effect

(d)   Pasteur effect

Answer: (b)

29. In which form does the food transported in plants?

(a)   Sucrose

(b)   Fructose

(c)   Glucose

(d)   Lactose

Answer: (a)

30. In Cycas, pollination takes place in

(a)   3 celled stage

(b)   4 celled stage

(c)   2 celled stage

(d)   1 celled stage

Answer: (a)

31. The bioassay of auxin is

(a)   avena curvature test

(b)   callus formation

(c)   culture of fungus

(d)   seed dormancy

Answer: (a)

32. A eukaryotic gene contains two kinds of base sequences. Which of these plays an important role in protein synthesis?

(a)   Introns

(b)   Exons

(c)   Both (a) and (b)

(d)   None of these

Answer: (b)

33. L-shaped chromosomes are also called

(a)   acrocentric

(b)   telocentric

(c)   sub-metacentric

(d)   None of these

Answer: (c)

34. Which of the following is/are grouped under phanerogams?

(a)   Angiosperms

(b)   Gymnosperms

(c)   Pteridophytes

(d)   Both (a) and (b)

Answer: (d)

35. A bacterium divides after every 35 min., if a culture containing 105 cells per mL is grown, then cell concentration per mL after 175 min. will be

(a)   175 × 105

(b)   125 × 105

(c)   48 × 105

(d)   32 × 105

Answer: (d)

36. Which of the following theory gives the latest explanation for the closure of stomata?

(a)   ABA theory

(b)   Munch theory

(c)   Starch glucose theory

(d)   Active K+ transport theory

Answer: (d)

37. Biological concept of species is mainly based on

(a)   reproductive isolation

(b)   morphological features only

(c)   methods of reproduction only

(d)   morphology and methods of reproduction

Answer: (a)

38. In the treatment of waste water discharge, which treatment stage involves biological treatment?

(a)   Primary treatment

(b)   Secondary treatment

(c)   Tertiary treatment

(d)   Reverse osmosis stage

Answer: (b)

39. Nucellus forms which of the following part of fruit?

(a)   Seed coat

(b)   Perisperm

(c)   Seed

(d)   Raphe

Answer: (b)

40. If root of a flowering plant has 24 chromosomes, then its gamete has how many chromosomes?

(a)   24

(b)   12

(c)   4

(d)   8

Answer: (b)

JIPMER MBBS Entrance Exam – 2009

GENERAL ENGLISH

Directions (Q. 1-5) : In each of the following questions, chose the alternative which best express the meaning of the given idiom/phrase.

1. A pipe dream

(a)   A pleasant dream

(b)   A bad dream

(c)   An impracticable plan

(d)   A foolish idea

Answer: (c)

2. To give up the ghost

(a)   To die

(b)   To make false appearance

(c)   To terrify others by acting in suspicious manner

(d)   To leave useless pursuits

Answer: (a)

3. To disabuse one’s mind

(a)   To conceal something

(b)   To remove a misapprehension

(c)   To banish from one’s mind a thought

(d)   To proceed cautiously so as to avoid risks and dangers

Answer: (b)

4. To wrangle over an ass’s shadow

(a)   To act in a foolish way

(b)   To quarrel over trifles

(c)   To waste time fron petty things

(d)   To do something funny

Answer: (b)

5. To be at daggers drawn

(a)   To be frightened

(b)   To be ready to face danger

(c)   To threaten one

(d)   To be bitter enemy

Answer: (d)

Directions (Q. 6-10) : Choose the correct alternative to fill in the blank

6. According to corporate circles, Datta is pushing through the merger to create a financially ________ company in the processed foods business, the group’s thrust area for the 1990s.

(a)   straight

(b)   powerful

(c)   leading

(d)   acceptable

Answer: (b)

7. A man who is well-bred and honourable in-variably shows ____ for the feelings of other people.

(a)   complacence

(b)   concern

(c)   consideration

(d)   contempt

Answer: (c)

8. You will have to face some practical problems when you start _____ this plan.

(a)   prosecuting

(b)   projecting

(c)   prescribing

(d)   proscribing

Answer: (b)

9. Speeding and blocking are traffic offences which lead to _____ accidents.

(a)   winsome

(b)   urban

(c)   gruesome

(d)   minor

Answer: (c)

10. The more your action and thought are allied and ______, the happier you grow.

(a)   invincible

(b)   divergent

(c)   integrated

(d)   unravelled

Answer: (c)

Directions (Q. 11-15) : In each of the following questions, rearrange the parts P, Q, R and S to make a proper sentence.

11. Around the world, painful terminal diseases/(P), the question of human death/(Q), people are wrestling with /(R), especially in the face of/(S)

(a)   R S Q P

(b)   P Q R S

(c)   R Q S P

(d)   P S Q R

Answer: (c)

12. The specific, ability requirements of the job/(P), depend on the /(Q), required for adequate job performance/(R), intellectual or physical abilities/(S)

(a)   P Q R S

(b)   S R Q P

(c)   P R Q S

(d)   S Q R P

Answer: (b)

13. Freedom is the restricted kind in the sense/(P) , the rich and poor woman/(Q), that a wide gulf separates /(R), which a modern woman enjoys/(S)

(a)   P S R Q

(b)   S R Q P

(c)   R Q P S

(d)   S P R Q

Answer: (d)

14. In life, some rules are /(P), as in business / (Q), they seem almost instinctive / (R), learnt so early that/ (S)

(a)   R S P Q

(b)   Q P S R

(c)   R P S Q

(d)   Q S P R

Answer: (b)

15. Kapil, left in an aeroplane/ (P), after reading a sailing magazine/ (Q), had decided / (R), to build his own boat nine years earlier/ (S)

(a)   P R Q S

(b)   R S Q P

(c)   R Q P S

(d)   P S R Q

Answer: (b)

Directions (Q. 16-20) : In each of the following questions, a part of the sentence is italicized. Below each sentence, three possible substitutions for the italicized part are given. Choose the one which improves the italicized part. If none of the substitutions improves the italicized part, your answer is (d).

16. I would gladly accompany your sister if you had asked me.

(a)   would have gladly accompanied

(b)   was to have gladly accompanied

(c)   will gladly accompany

(d)   No improvement

Answer: (a)

17. The indecisive man was readily persuaded to change his mind again.

(a)   was persuaded ready

(b)   was ready to persuade

(c)   was ready persuaded

(d)   No improvement

Answer: (d)

18. He enjoys to tell stories to children.

(a)   how to tell stories

(b)   telling stories

(c)   to narrate stories

(d)   No improvement

Answer: (b)

19. It is unreasonable to distort the statement of a man simply because he does not agree with your opinions.

(a)   discourage

(b)   denounce

(c)   bend

(d)   No improvement

Answer: (d)

20. Would you find me absent, please don’t forget to leave a message behind.

(a)   As

(b)   Should

(c)   Unless

(d)   No improvement

Answer: (b)

Directions (Q. 21-25) : In each of the following questions, choose the alternative which is most nearly the same in meaning to the word given in italics in the sentence.

21. The rancous shouts of the audience forced the management to call off the show.

(a)   unpleasant

(b)   loud

(c)   harsh

(d)   harmful

Answer: (c)

22. At the Hardy house there was great consternation when Aunt Gertrude saw Joe helping Frank up the stairs.

(a)   surprise

(b)   panic

(c)   gaity

(d)   anxiety

Answer: (a)

23. Methods of spreading the family planning message have been as fecund as they have been abortive.

(a)   fruitful

(b)   failing

(c)   peculiar

(d)   false

Answer: (a)

24. Now the fury of the demonstrators turned against the machines.

(a)   asperity

(b)   passion

(c)   rage

(d)   acrimony

Answer: (c)

25. Everyone who has worked for him hammers home that point because they feel that it is widely unappreciated.

(a)   hints

(b)   stresses

(c)   strikes

(d)   directs

Answer: (b)

Directions (Q. 26-30) : In each of the following questions, choose the alternative which is opposite in meaning to the word given in italics in the sentence.

26. Here debonair manners were noticed by everyone present in the city.

(a)   stiff

(b)   cheerless

(c)   courteous

(d)   pleasant

Answer: (b)

27. Like poverty, affluence can sometimes create its own problems.

(a)   sorrow

(b)   indigence

(c)   exuberance

(d)   opulence

Answer: (b)

28. It is surprising to find her condoning such an act.

(a)   disparaging

(b)   condemning

(c)   disliking

(d)   forbidding

Answer: (b)

29. He proved utterly capricious in his dealings with his friends.

(a)   helpful

(b)   steadfast

(c)   understanding

(d)   obstinate

Answer: (c)

30. As long as he remained in that office, he maintained his

(a)   predominance

(b)   poverty

(c)   subordination

(d)   chaos

Answer: (c)

Directions (Q. 31-35) : In each of the following questions, choose the alternative which can be substituted for the given words/sentence.

31. Medical study of the skin and its diseases

(a)   Dermatology

(b)   Orthopaedics

(c)   Venereology

(d)   Homeopathy

Answer: (a)

32. Code of diplomatic etiquette and precedence

(a)   Formalism

(b)   Statesmanship

(c)   Protocol

(d)   Hierachy

Answer: (c)

33. The branch of medical science which deals with the problems of the old

(a)   Oncology

(b)   Geriatric

(c)   Obsertics

(d)   Endocrinol

Answer: (b)

34. One who promotes the idea of absence of government of any kind, when every man should be a law unto himself

(a)   Anarchist

(b)   Belligerent

(c)   Iconoclast

(d)   Agnostic

Answer: (a)

35. Design made by putting together coloured pieces of glass of stones

(a)   Oleograph

(b)   Mosaic

(c)   Tracery

(d)   Relief

Answer: (b)

Directions (Q. 36-40) : Read the following passage carefully and answer the questions given below it.

Most of us use the products of science-railways, aeroplanes, electricity, wireless and thousands of others-without thinking how they came into existence. We taken them for granted, as if we were entitled to them as a matter of right. And we are very proud of the fact that we live in an advanced age and are ourselves so very ‘advanced’. Now, there is no doubt that our age is a very different one from previous ages and I think it is perfectly correct to say that it is far more advanced. But that is a different thing from saying that we as individuals or groups are more advanced. It would be the height of absurdity to say that because an engine-driver can run an engine and Plato or Socrates could not, the engine-driver is more advanced than, or is superior to Plato or Socrates. But it would be perfectly correct to say that the engine itself is a more advanced method of locomotion than Plato’s chariot was.

36. Which one of the following statements is true?

(a)   An engine-driver is cleverer than Plato or Socrates.

(b)   Plato or Socrates is in way inferior to the engine-driver.

(c)   Plato and Socrates surpassed the engine-driver in every respect.

(d)   The engine-driver cannot be compared to Plato or Aristotle.

Answer: (b)

37. In this passage the author mentions Plato and/or Socrates to emphasise that

(a)   they are/were men of great scholarship.

(b)   people as individuals in the modern age are not more advanced than their predecessors.

(c)   the engine is a better mode of locomotion than Plato’s chariot.

(d)   Plato and Aristotle had greater respect for learning

Answer: (b)

38. According to the author, the present age is far more advanced than

(a)   all the previous ages in some respect.

(b)   the age of Socrates and Aristotle in some respects.

(c)   some of the previous ages in all respects.

(d)   all the previous ages in all respects.

Answer: (a)

39. Many of us make use of machines

(a)   with very little knowledge of their mechanism

(b)   without any knowledge of their historical significance.

(c)   with full knowing of their genesis.

(d)   without knowing how they were invented.

Answer: (d)

40. People today are very proud because they live

(a)   in a philosophically advanced age.

(b)   in a materially advanced age.

(c)   in a scientifically advanced age.

(d)   in a spiritually advanced age.

Answer: (c)

JIPMER Medical Entrance Exam Previous Year Question Paper 2008 With Answer Key

JIPMER MBBS Entrance Exam – 2008

PHYSICS

1. For inelastic collision between two spherical rigid bodies

(a)  the total kinetic energy is conserved

(b)  the total mechanical energy is not conserved

(c)  the linear momentum is not conserved

(d)  the linear momentum is conserved

Answer: (d)

2. We consider a thermodynamic system. If ∆U represents the increase in its internal energy and W the work done by the system, which of the following statements is true ?

(a)  ∆U = −W in an adiabatic process

(b)  ∆U = W in an isothermal process

(c)  ∆U = −W in an isothermal  process

(d)  ∆U = W in an adiabatic process

Answer: (a)

3. Aerofils are so designed that the speed of air

(a)  on top side is more than on lower side

(b)  on top side is less than on lower side

(c)  is same on both sides

(d)  is turbulent

Answer: (a)

4. The time of reverberation of a room A is one second. What will be the time (in seconds) of reverberation of a room, having all the dimensions double of those of room A ?

(a)  2

(b)  4

(c)  1/2

(d)  1

Answer: (a)

5. In refraction, light waves are bent on passing from one medium to the second medium, because, in the second medium

(a)  the frequency is different

(b)  the coefficient of elasticity is different

(c)  the speed is different

(d)  the amplitude is smaller

Answer: (c)

6. Which of the following circular rods, (given radius r and length l) each made of the same material and whose ends are maintained at the same temperature will conduct most heat?

(a)  r = 2r0; l = 2l0

(b)  r = 2r0; l = l0

(c)  r = r0; l = l0

(d)  r = r0; l = 2l0

Answer: (b)

7. For a satellite moving in an orbit around the earth, the ratio of kinetic energy to potential energy is

(a)  2

(b)  1/2

(c)  1/√2

(d)  √2

Answer: (b)

8. 300 J of work is done in sliding a 2 kg block up an inclined plane of height 10 m. Taking g = 10 m/s2, work done against friction is

(a)  200 J

(b)  100 J

(c)  zero

(d)  1000 J

Answer: (b)

9. Interference was observed in an interference chamber when air was present. Now, the chamber is evacuated and if the same light is used, a careful observation will show

(a)  no interference

(b)  interference with  bright band

(c)  interference with dark bands

(d)  interference in which breadth of the fringe will be slightly increased

Answer: (d)

10. When a charged particle moving with velocity  is subjected to a magnetic field of induction  the force on it is non-zero. This implies that

(a)  angle between  is necessarily 90°

(b)  angle between  can have any value other than 90°

(c)  angle between  can have any value other than zero and 180°

(d)  angle between  is either zero or 180°

Answer: (c)

11. A particle moves along a straight line OX. At a time t (in second) the distance x (in metre) of the particle from O is given by x = 40 + 12t – t3

How long would the particle travel before coming to rest?

(a)  24 m

(b)  40 m

(c)  56 m

(d)  16 m

Answer: (c)

12. Which one of the following statements is true?

(a)  Both light and sound waves in air are transverse

(b)  The sound waves in air are longitudinal while the light waves are transverse

(c)  Both light and sound waves in air are longitudinal

(d)  Both light and sound waves can travel in vacuum

Answer: (b)

13. When power is drawn from the secondary coil of the transformer, the dynamic resistance

(a)  increases

(b)  decreases

(c)  remains unchanged

(d)  changes erratically

Answer: (a)

14. Pressure of an ideal gas is increased by keeping temperature constant. What is the effect on kinetic energy of molecules?

(a)  Increase

(b)  Decrease

(c)  No change

(d)  Can’t be determined

Answer: (c)

15. What maximum frequency can be reflected from ionosphere?

(a)  5 MHz

(b)  6 GHz

(c)  5 kHz

(d)  500 MHz

Answer: (a)

16. A telescope has focal length of objective and eye-piece as 200 cm and 5 cm respectively. What is magnification of telescope?

(a)  40

(b)  80

(c)  50

(d)  0.01

Answer: (a)

17. A small disc of radius 2 cm is cut from a disc of radius 6 cm. If the distance between their centres is 3.2 cm, what is the shift in the centre of mass of the disc?

(a)  0.4 cm

(b)  2.4 cm

(c)  1.8 cm

(d)  1.2 cm

Answer: (a)

18. Hydrogen bomb is based upon

(a)  fission

(b)  fusion

(c)  chemical reaction

(d)  transmutation

Answer: (b)

19. What is not true for equipotential surface for uniform electric field?

(a)  Equipotential surface is flat

(b)  Equipotential surface is spherical

(c)  Electric lines are perpendicular to equipotential surface

(d)  Work done is zero

Answer: (b)

20. A closed organ pipe of length 20 cm is sounded with tuning fork in resonance. What is the frequency of tuning fork? (v = 332 m/s)

(a)  300 Hz

(b)  350 Hz

(c)  375 Hz

(d)  415 Hz

Answer: (d)

21. The displacement of particle is given by

What is its acceleration?

(a)  2a2/3

(b)  −2a2/3

(c)  a2

(d)  Zero

Answer: (b)

22. The area of the acceleration-displacement curve of a body gives

(a)  impulse

(b)  change in momentum per unit mass

(c)  change in KE per unit mass

(d)  total change in energy

Answer: (c)

23. The speed of earth’s rotation about its axis is ω. Its speed is increased to x times to make the effective acceleration due to gravity equal to zero at the equator. Then x is

(a)  1

(b)  8.5

(c)  17

(d)  34

Answer: (c)

24. Hard X-rays for the study of fractures in bones should have a minimum wavelength of 10−11 The accelerating voltage for electrons in X-ray machine should be

(a)  < 124 kV

(b)  > 124 kV

(c)  between 60 kV and 70 kV

(d)  = 100 kV

Answer: (a)

25. In photoelectric effect, the electrons are ejected from metals if the incident light has a certain minimum

(a)  wavelength

(b)  frequency

(c)  amplitude

(d)  angle of incidence

Answer: (b)

26. A capacitor having capacitance 1 μF with air, is filled with two dielectrics as shown. How m any times capacitance will increase?

(a)  12

(b)  6

(c)  8/3

(d)  3

Answer: (b)

27. A leaf which contains only green pigments, is illuminated by a laser light of wavelength 0.6328 μm. It would appear to be

(a)  brown

(b)  black

(c)  red

(d)  green

Answer: (b)

28. The operation of a nuclear reactor is said to be critical, if the multiplication factor (k) has a value

(a)  1

(b)  1.5

(c)  2.1

(d)  2.5

Answer: (a)

29. A galvanometer having a resistance of 8 Ω is shunted by a wire of resistance 2 Ω. If the total currents is 1 A, the part of it passing through the shunt will be

(a)  0.25 A

(b)  0.8 A

(c)  0.2 A

(d)  0.5 A

Answer: (b)

30. Flash light equipped with a new set of batteries, produces bright white light. As the batteries wear out

(a)  the light intensity gets reduced with no change in its colour

(b)  light colour changes first to yellow and then red with no change in intensity

(c)  it stops working suddenly while giving white light

(d)  colour changes to red and also intensity gets reduced

Answer: (d)

31. If alpha, beta and gamma rays carry same momentum, which has the longest wavelength?

(a)  Alpha rays

(b)  Beta rays

(c)  Gamma rays

(d)  None, all have same wavelength

Answer: (d)

32. When you make ice cubes, the entropy of water

(a)  does not change

(b)  increases

(c)  decreases

(d)  may either increase or decrease depending on the process used

Answer: (c)

33. Half-lives of two radioactive substances A and B are respectively 20 min and 40 min. Initially the samples of A and B have equal number of nuclei. After 80 min the ratio of remaining number of A and B nuclei is

(a)  1 : 16

(b)  4 : 1

(c)  1 : 4

(d)  1 : 1

Answer: (c)

34. Given that  where y and x are measured in metre. Which of the following statements is true?

(a)  The unit of λ is same as that of x and A

(b)  The unit of λ is same as that of x but not of A

(c)  The unit of c is same as that of 2π/λ

(d)  The unit(ct – x) is same as that of 2π/λ

Answer: (a)

35. A projectile is thrown in the upward direction making an angle of 60° with the horizontal direction with a velocity of 147 ms−1. Then the time after which its inclination with the horizontal is 45°, is

(a)  15 s

(b)  10.98 s

(c)  5.49 s

(d)  2.745 s

Answer: (c)

36. A motor cycle is going on an overbridge of radius R. The driver maintains a constant speed. As the motor cycle is ascending on the overbridge, the normal force on it

(a)  increases

(b)  decreases

(c)  remains the same

(d)  fluctuates erratically

Answer: (a)

37. If we throw a body upwards with velocity of 4 m/s, at what height does its kinetic energy reduce to half of the initial value?

(Take g = 10 ms−2)

(a)  4 m

(b)  2 m

(c)  1 m

(d)  0.4 m

Answer: (d)

38. Two glass plates are separated by water. If surface tension of water is 75 dyne/cm and area of each plate wetted by water is 8 cm2 and the distance between the plates is 0.12 mm, then the force applied to separated the two plates is

(a)  102 dyne

(b)  104 dyne

(c)  105 dyne

(d)  106 dyne

Answer: (c)

39. Which of the following is true for rays coming from infinity?

(a)  Two images are formed

(b)  Continuous image is formed between focal points of upper and lower lens

(c)  One image is formed

(d)  None of the above

Answer: (a)

40. The north pole of a long horizontal bar magnet is being brought closer to a vertical conducting plane along the perpendicular direction. The direction of the induced current in the conducting plane will be

(a)  horizontal

(b)  vertical

(c)  clockwise

(d)  anticlockwise

Answer: (c)

JIPMER MBBS Entrance Exam – 2008

Chemistry

1. Plaster of paris is

(a) 

(b)  CaSO4 ∙ 2H2O

(c)  CaSO4 ∙ H2O

(d)  CaSO4 ∙ 4H2O

Answer: (a)

2. The most stable compound is

(a)  LiF

(b)  LiCl

(c)  LiBr

(d)  LiI

Answer: (a)

3. Heavy water is

(a)  CaSO4

(b)  water contain CaSO4, MgSO4

(c)  D2O

(d)  water contain CaCO3

Answer: (c)

4. When copper reacts with hot and conc.H2SO4, gives

(a)  H2

(b)  N2

(c)  O2

(d)  SO2

Answer: (d)

5. BaO2 and ozone react to produce

(a)  Ba

(b)  BaO3

(c)  BaO

(d)  Ba(OH)3

Answer: (c)

6. Heisenberg uncertainty principle can be explained as

(a) 

(b) 

(c) 

(d) 

Answer: (b)

7. A gas mixture contains O2 and N2 in the ratio of 1 : 4by weight. The ratio of their number of molecules is

(a)  1 : 8

(b)  1 : 4

(c)  3 : 16

(d)  7 : 32

Answer: (d)

8. Bleaching powder is obtained by treating Cl2 with

(a)  Ca(OH)2

(b)  CaO

(c)  CaCO3

(d)  CaCl2

Answer: (a)

9. The de-Broglie wavelength of a particle with mass 1 kg and velocity 100 m/s is

(a)  6.6 × 10−33 m

(b)  6.6 × 10−36 m

(c)  3.3 × 10+33 m

(d)  3.3 × 10−36 m

Answer: (b)

10. The volume of a gas measured at 27°C and 1 atm pressure is 10 L. To reduce the volume to 2 L at 1 atm pressure, the temperature required is

(a)  60 K

(b)  75 K

(c)  150 K

(d)  225 K

Answer: (a)

11. The number of moles of oxygen obtained by the electrolytic decomposition of 108 g water is

(a)  2.5

(b)  3

(c)  5

(d)  7.5

Answer: (b)

12. The change in entropy for the fusion of 1 mole of ice [mp of ice = 273 K, molar enthalpy of fusion for ice = 6.0 kJ mol−1]

(a)  11.73 JK−1 mol−1

(b)  18.84 JK−1 mol−1

(c)  21.97 JK−1 mol−1

(d)  24.47 JK−1 mol−1

Answer: (c)

13. Which does not give a precipitate with AgNO3 solution ?

(a)  [Co(NH3)6]Cl3

(b)  [Co(NH3)5Cl]Cl2

(c)  [Co(NH3)4Cl2]Cl

(d)  [Co(NH3)­3Cl3]

Answer: (d)

14. Total volume of atoms present in a face centred cubic unit cell of a metal is (r is atomic radius)

(a) 

(b) 

(c) 

(d) 

Answer: (a)

15. Pure silicon doped with phosphorus is a metallic conductor

(a)  metallic conductor

(b)  insulator

(c)  n-type semiconductor

(d)  p-type semiconductor

Answer: (c)

16. Neutron is discovered by

(a)  Chadwick

(b)  Rutherford

(c)  Yukawa

(d)  Dalton

Answer: (a)

17. What is X in the following nuclear reaction?

7N14 + 1H18O15 + X

(a)  0n1

(b)  −1e0

(c)  +1e0

(d)  γ

Answer: (d)

18. Solubility product PbCl2 at 298 K is 1 × 10−6. At this temperature solubility of PbCl2 in mol/L is

(a)  (1 × 10−6)1/2

(b)  (1 × 10−6)1/3

(c)  (0.25 × 10−6)1/3

(d)  (2.5 × 10−6)1/2

Answer: (c)

19. The pH of a 0.001 M solution of HCl is

(a)  0

(b)  3

(c)  5

(d)  10

Answer: (b)

20. Gold number is associated with

(a)  amount of gold

(b)  protective colloids

(c)  purple cassius

(d)  electrophoresis

Answer: (b)

21. Noble gases are used in discharge tubes to give different colours. Reddish-orange glow is due to

(a)  Ar

(b)  Ne

(c)  Xe

(d)  Kr

Answer: (b)

22. The set representing the correct order for first ionization potential is

(a)  K > Na > Li

(b)  Be > Mg > Ca

(c)  B > C > N

(d)  Ge > Si > C

Answer: (b)

23. Dry ice is

(a)  solid CO2

(b)  solid camphor

(c)  solid SO2

(d)  solid NO2

Answer: (a)

24. Methanol and ethanol are miscible in water due to

(a)  covalent character

(b)  hydrogen bonding character

(c)  oxygen bonding character

(d)  None of the above

Answer: (b)

25. Stereoisomers differ in

(a)  configuration

(b)  conformation

(c)  they do not differ

(d)  None of the above

Answer: (a)

26. IUPAC name of the following compound

(a)  N, N-dimethylcyclopropane caboxamide

(b)  N-methylcyclopropanamide

(c)  cyclopropionamide

(d)  None of the above

Answer: (a)

27. The product of the following reaction is

2CH3­MgBr + CH2O →

(a)  CH3OH

(b)  C2H5OH

(c)  CH4

(d)  C2H6

Answer: (b)

28. Freon used as refrigerant is

(a)  CF2 = CF2

(b)  CH2F2

(c)  CH4

(d)  C2H6

Answer: (c)

29. Lucas reagent is

(a)  anhy. ZnCl2 and NH3

(b)  anhy. ZnCl2 and CaCl2

(c)  anhy. ZnCl2 and conc. HCl

(d)  anhy. ZnCl2 and HCl gas

Answer: (c)

30. The enzyme which can catalyze the conversion of glucose to ethanol is

(a)  zymase

(b)  invertase

(c)  maltase

(d)  diastase

Answer: (a)

31. When dihydroxy acetone reacts with HIO4, the product is/are

(a)  HCHO

(b)  HCOOH

(c)  HCHO and HCOOH

(d)  HCHO and CO2

Answer: (d)

32. Which of the following does not reduce Fehling’s solution?

(a)  Benzaldehyde

(b)  Formic acid

(c)  Glucose

(d)  Fructose

Answer: (d)

33. Sodium formate on heating gives

(a)  oxalic acid and H2

(b)  sodium oxalate and H2

(c)  sodium oxalate

(d)  CO2 and caustic soda

Answer: (b)

34. Reaction of ethyl formate with excess of CH3MgI followed by hydrolysis gives

(a)  n-propyl alcohol

(b)  isopropyl alcohol

(c)  acetaldehyde

(d)  acetone

Answer: (c)

35. Hydrolysis of phenyl isocyanide forms

(a)  benzoic acid

(b)  formic acid

(c)  acetic acid

(d)  None of these

Answer: (b)

36. Styrene can be purified by

(a)  simple distillation

(b)  fractional distillation

(c)  steam distillation

(d)  vacuum distillation

Answer: (d)

37. Which of the following is not reducing sugar?

(a)  Glucose

(b)  Fructose

(c)  Lactose

(d)  Sucrose

Answer: (d)

38. The monomer of Teflon is

(a)  CHF = CH2

(b)  CF2 = CF2

(c)  CHCl  = CHCl

(d)  CHF = CHCl

Answer: (b)

39. The hybridization state of carbon in fullerene is

(a)  sp

(b)  sp2

(c)  sp3

(d)  sp3d

Answer: (b)

40. A fruity smell is produced by the reaction of C2H5OH with

(a)  CH3COCH3

(b)  CH3COOH

(c)  PCl5

(d)  CH3CHO

Answer: (b)

JIPMER MBBS Entrance Exam – 2008

Zoology

1. Moulting hormone is secreted by

(a)  corpora cardiacum

(b)  corpora allata

(c)  neurosecretory hormone

(d)  prothoracic gland

Answer: (d)

2. Movements by pseudopodia of Amoeba are due to change in

(a)  Pressure

(b)  atmosphere

(c)  temperature

(d)  viscosity

Answer: (d)

3. From outer to inside the sequence of three bones present in the middle ear of mammals is

(a)  malleus, stapes and incus

(b)  stapes, malleus and incus

(c)  malleus, incus and stapes

(d)  incus, malleus and stapes

Answer: (c)

4. The reabsorption of water in the kidney is under the control of

(a)  LH

(b)  ADH

(c)  STH

(d)  ACTH

Answer: (b)

5. Yersinia pestis is responsible for

(a)  syphilis

(b)  whooping cough

(c)  plague

(d)  leprosy

Answer: (c)

6. Protein present in silk fibre is

(a)  casein

(b)  keratin

(c)  elastin

(d)  fibroin

Answer: (d)

7. Ovulation takes place in a month between

(a)  11-14 day

(b)  14-16 day

(c)  15-28 day

(d)  21-26 day

Answer: (b)

8. In sea anemone, the symmetry is

(a)  radial

(b)  bilateral

(c)  spherical

(d)  absent

Answer: (a)

9. Compound squamous epithelium is found in

(a)  stomach

(b)  intestine

(c)  trachea

(d)  pharynx

Answer: (d)

10. Solenocytes are associated with

(a)  respiration

(b)  digestion

(c)  excretion

(d)  nutrition

Answer: (c)

11. The cavity of diencephalon is known as

(a)  I ventricle

(b)  II ventricle

(c)  III ventricle

(d)  iter

Answer: (c)

12. Which vitamin should not be stored?

(a)  Calciferol

(b)  Retinol

(c)  Niacin

(d)  Ascorbic acid

Answer: (d)

13. Connecting link between annelids and molluscs is

(a)  Neophilina

(b)  Peripatus

(c)  Periplaneta

(d)  Limulus

Answer: (a)

14. Shell of mollusks is derived from

(a)  foot

(b)  mantle

(c)  ctenidia

(d)  placoid

Answer: (b)

15. Pneumatic bones are expected to be found in

(a)  house lizard

(b)  flying fish

(c)  pigeon

(d)  tadpole of frog

Answer: (c)

16. A man of ‘A’ blood group marries a woman of ‘AB’ blood group. Which type of progeny would indicate that man is heterozygous?

(a)  O

(b)  B

(c)  A

(d)  AB

Answer: (b)

17. Balbiani rings are the structural features of

(a)  allosomes

(b)  polytene chromosomes

(c)  autosomes

(d)  Lampbrush chromosomes

Answer: (b)

18. Presence of tail in a child is an example of

(a)  atavism

(b)  divergent evolution

(c)  convergent evolution

(d)  mutation

Answer: (a)

19. Action potential is generated by

(a)  Na+

(b)  K+

(c)  Ca+

(d)  Cl

Answer: (a)

20. Aqueous and vitreous humour are divided by

(a)  lens

(b)  iris

(c)  retina

(d)  optic nerve

Answer: (a)

21. Indian rhinoceros are protected in

(a)  Gir Forest

(b)  Kaziranga National Park

(c)  Bandipur National Park

(d)  Ranthambore National Park

Answer: (b)

22. Mitotic spindle have main protein

(a)  tubulin

(b)  myosin

(c)  tropomyocin

(d)  dynein

Answer: (a)

23. Accessory sexual character in female is promoted by

(a)  androgen

(b)  progesterone

(c)  estrogen

(d)  testosterone

Answer: (c)

24. X-linked recessive gene is

(a)  always expressed in male

(b)  always expressed in female

(c)  lethal

(d)  sub lethal

Answer: (a)

25. Ligament is mainly made up of

(a)  reticulin

(b)  elastin

(c)  myosin

(d)  collagen

Answer: (d)

26. Secretion of pancreatic juice is stimulated by

(a)  gastrin

(b)  secretin

(c)  enterogasteron

(d)  enterokinase

Answer: (b)

27. Crossing over occurs in

(a)  zygotene

(b)  leptotene

(c)  pachytene

(d)  diplotene

Answer: (c)

28. Mesozoic era is Golden period of

(a)  reptiles

(b)  Mollusca

(c)  fishes

(d)  amphibians

Answer: (a)

29. Hamburger shift is also known as

(a)  bicarbonate shift

(b)  chloride shift

(c)  potassium shift

(d)  All of the above

Answer: (b)

30. “Bundle of His” are

(a)  nervous tissue supplied to ventricles

(b)  nervous tissue supplied to heart

(c)  muscular tissue supplied to ventricle

(d)  muscular tissue supplied to heart

Answer: (c)

31. Cell theory was proposed by

(a)  Virchow

(b)  Schleiden and Schwann

(c)  Robert Hooke

(d)  B McClintock

Answer: (b)

32. Who for the first time developed electron microscope?

(a)  Knoll and Ruska

(b)  Rudolf and Kolliker

(c)  Robert Hooke

(d)  Swanson

Answer: (a)

33. Repressor protein is produced by

(a)  regulator gene

(b)  operator gene

(c)  structural gene

(d)  promoter gene

Answer: (a)

34. Autonomic nervous system affects

(a)  reflex actions

(b)  sensory organs

(c)  internal organ

(d)  None of these

Answer: (c)

35. Schizogony occurs in

(a)  RBC of human

(b)  intestine of parasite

(c)  liver of human

(d)  liver of parasite

Answer: (c)

36. Gases found in primitive atmosphere are

(a)  CH4, NH3, H2, H2O (vapour form)

(b)  CH4, NH3, CO2, H2O

(c)  CH4, H2O, CO2

(d)  CH4, O2, CO2

Answer: (a)

37. Sertoli cells are found in testis. These cells are

(a)  nurse cell

(b)  reproductive cell

(c)  receptor cell

(d)  None of the above

Answer: (a)

38. After a deep inspiration and maximum expiration, the capacity of lungs is known as

(a)  vital capacity

(b)  tidal volume

(c)  IRV

(d)  ERV

Answer: (a)

39. Which one is the sweetest sugar ?

(a)  Glucose

(b)  Fructose

(c)  Sucrose

(d)  Maltose

Answer: (b)

40. Species can be identified on the basis of

(a)  interbreed

(b)  species diversity

(c)  reproductive isolation

(d)  None of the above

Answer: (c)

JIPMER MBBS Entrance Exam – 2008

Botany

1. In Whittaker’s system of classification, prokaryotes belong to the kingdom

(a)  Monera

(b)  Protista

(c)  Animalia

(d)  Fungi

Answer: (a)

2. Water bloom is generally caused by

(a)  green algae

(b)  blue-green algae

(c)  bacteria

(d)  Hydrilla

Answer: (b)

3. Which one of the following is a saprophytic bryophyte?

(a)  Riccia fluitans

(b)  Buxbaumia aphylla

(c)  Funaria hygrometrica

(d)  Sphagnum

Answer: (b)

4. A genophore is made up of

(a)  a single double stranded DNA

(b)  a single stranded DNA

(c)  RNA and histones

(d)  histones and non-histones

Answer: (a)

5. Iodine is obtained from

(a)  Laminaria

(b)  Chlorella

(c)  Polysiphonia

(d)  Porphyra

Answer: (a)

6. Covered smut of barley is caused by

(a)  Ustilago hordei

(b)  Tilletia caries

(c)  Ustilago nuda

(d)  Colletotrichum falcatum

Answer: (a)

7. Root hair absorbs water from soil through

(a)  turgor pressure

(b)  ion exchange

(c)  osmosis

(d)  DPD

Answer: (c)

8. In CAM plants, CO2 acceptor is

(a)  RuBP

(b)  PEP

(c)  OAA

(d)  PGA

Answer: (b)

9. Which one of the following is a natural growth inhibitor?

(a)  NAA

(b)  ABA

(c)  IAA

(d)  GA

Answer: (b)

10. Apical dominance is caused by

(a)  auxin

(b)  gibberellin

(c)  kinetin

(d)  ABA

Answer: (a)

11. Growth of pollen tube towards emryo sac is

(a)  geotropism

(b)  thigmotaxis

(c)  chemotaxis

(d)  All of these

Answer: (c)

12. Ozone depletion in stratosphere would result in

(a)  forest fires

(b)  global warming

(c)  skin cancer incidence

(d)  None of the above

Answer: (c)

13. Which of the following is a test cross ?

(a)  Tt × tt

(b)  TT × tt

(c)  Tt × Tt

(d)  tt × tt

Answer: (a)

14. 1 : 2 : 1 phenotypic and genotypic ratio is found in

(a)  complementary genes

(b)  blending inheritance

(c)  multiple alleles

(d)  pseudo alleles

Answer: (b)

15. The back bone of RNA consists of which of the following sugar?

(a)  Deoxyribose

(b)  Ribose

(c)  Sucrose

(d)  Maltose

Answer: (b)

16. Stop codons are

(a)  AUG, GUG

(b)  UAA, UGA, UAG

(c)  UAC, UGG

(d)  AGU, AGA, UAC

Answer: (b)

17. A lake with nutrients is called

(a)  trophic

(b)  euphotic

(c)  oligotrophic

(d)  eutrophic

Answer: (d)

18. Which of the following is not caused by deficiency of mineral?

(a)  Chlorosis

(b)  Etiolation

(c)  Shortening of internodes

(d)  Necrosis

Answer: (b)

19. Retrovirus have genetic material

(a)  DNA only

(b)  RNA only

(c)  DNA or RNA only

(d)  either DNA or RNA only

Answer: (b)

20. DCMU

(a)  inhibits PS-I

(b)  inhibits PS-II

(c)  destroy chloroplast

(d)  inhibits oxidative phosphorylation

Answer: (b)

21. According to Kyoto protocol the major nations abide to reduce concentration of green house gases by

(a)  2008

(b)  2010

(c)  2012

(d)  2018

Answer: (c)

22. Ginger is an underground stem. It is distinguished from root because

(a)  it lacks chlorophyll

(b)  it stores food

(c)  it has nodes and internodes

(d)  it has xylem and vessels

Answer: (c)

23. During DNA replication in prokaryotes DNA is anchored to

(a)  chromosome

(b)  mesosome

(c)  nucleolus

(d)  ribosome

Answer: (b)

24. The mineral present in cell wall is

(a)  Na

(b)  Ca

(c)  K

(d)  Mg

Answer: (b)

25. Male gamete in angiosperm is produced by

(a)  generative cell

(b)  microspore cell

(c)  vegetative cell

(d)  tube cell

Answer: (a)

26. Meristematic tissue in vascular bundle is

(a)  phellem

(b)  procambium

(c)  interfascicular cambium

(d)  fascicular cambium

Answer: (d)

27. Study of Ecology of population is called

(a)  Autecology

(b)  Synecology

(c)  Ecotype

(d)  Demecology

Answer: (d)

28. Which of the following plant is LDP?

(a)  Xanthium

(b)  Soybean

(c)  Wheat

(d)  Tobacco

Answer: (c)

29. Which of the following is used as a best genetic vector in plants?

(a)  Bacillus thurengiensis

(b)  Agrobacterium tumifaciens

(c)  Pseudomonas putida

(d)  All of the above

Answer: (b)

30. One gene-one enzyme hypothesis of Beadle and Tatum was experimentally proved on

(a)  Saccharomyces

(b)  Neurospora crassa

(c)  Lathyrus odoratus

(d)  Claviceps

Answer: (b)

31. Which of the following is not a pyrimidine?

(a)  Thymine

(b)  Uracil

(c)  Guanine

(d)  Cytosine

Answer: (c)

32. Which organelle is present in higher number in secretory cells?

(a)  Dictyosome

(b)  ER

(c)  Lysosome

(d)  Vacuole

Answer: (a)

33. Bio-indicators are used for

(a)  oxygen demand

(b)  air pollution

(c)  mineral present

(d)  All of the above

Answer: (d)

34. What happened when we inoculated Rhizobium in wheat field?

(a)  No increase in production (nitrogen content of soil remains same)

(b)  A lot of increase in production (nitrogen content of soil increases)

(c)  Fertility of soil decreases

(d)  Fertility of soil increases

Answer: (a)

35. Nitrifying bacteria are able to

(a)  convert atmospheric nitrogen into soluble forms

(b)  convert ammonia to nitrate

(c)  ammonia to nitrogen

(d)  nitrate to nitrogen

Answer: (b)

36. RNA is not found in

(a)  chromosome

(b)  plasmalemma

(c)  nucleolus

(d)  ribosome

Answer: (b)

37. Systema Naturae was written by

(a)  Linnaeus

(b)  Aristotle

(c)  Hippocrates

(d)  Darwin

Answer: (a)

38. Number of chromosomes in an angiospermic plant is 14, then the number of chromosomes in synergid cells will be

(a)  14

(b)  7

(c)  28

(d)  21

Answer: (b)

39. Phytochrome is present in

(a)  vascular cryptogam

(b)  prokaryotes

(c)  flowering plants

(d)  algae

Answer: (c)

40. Vessels and companion cells are found in

(a)  angiosperm

(b)  pteridophytes

(c)  bryophytes

(d)  thallophytes

Answer: (a)

JIPMER MBBS Entrance Exam – 2008

General English

Directions (Q. 1-5) In each of the following questions, choose the alternative which best expresses the meaning of the given idiom/phrase.

1. To throw dust in one’s eyes

(a)  To show false things

(b)  To make blind

(c)  To harm someone

(d)  To deceive

Answer: (d)

2. To be rolling in money

(a)  Wasting a lot of money

(b)  Very rich

(c)  Borrowing money liberally

(d)  Spending more than his income

Answer: (b)

3. To get into hot waters

(a)  To be impatient

(b)  To suffer a huge financial loss

(c)  To get into trouble

(d)  To be in a confused state of mind

Answer: (c)

4. To save one’s face

(a)  To hide oneself

(b)  To oppose

(c)  To evade disgrace

(d)  To say plainly

Answer: (c)

5. To wash one’s dirty linen in public

(a)  To criticize one’s nature in public

(b)  To quarrel in the open

(c)  To do some ugly work in public

(d)  To discuss dirty and scandalous matters of personal nature in the presence of strangers

Answer: (d)

Directions (Q. 6-10) : Choose the correct alternative to fill in the blank.

6. There was so much ………. material in the speech that it was difficult to know what the speaker wanted to say.

(a)  banal

(b)  extraneous

(c)  superficial

(d)  variegated

Answer: (c)

7. Our flight was ……….. from Jaipur to Agra airport.

(a)  deflected

(b)  shifted

(c)  diverted

(d)  reverted

Answer: (c)

8. His ……….. handling resulted in all that destruction and damage.

(a)  inept

(b)  skilful

(c)  sophisticated

(d)  uncouth

Answer: (a)

9. Rabindranath Tagore was a ……… writer of his times.

(a)  true

(b)  fantastic

(c)  profound

(d)  prolific

Answer: (c)

10. The manager tried hard to ………….. his men to return to work before declaring a lockout.

(a)  motivate

(b)  persuade

(c)  encourage

(d)  permit

Answer: (b)

Directions (Q. 11-15) : In each of the following questions, rearrange the parts P, Q, R and S to make a proper sentence.

11. Little

(P) that he had been let down

(Q) stood by all these years

(R) did he realize

(S) by a colleague whom he had

(a)  QSPR

(b)  QPSR

(c)  RPSQ

(d)  RSQP

Answer: (c)

12. It has been established that

(P) Einstein was

(Q) although a great scientist

(R) Weak in arithmetic

(S) right from his school days

(a)  QPRS

(b)  QPSR

(c)  RQPS

(d)  SRPQ

Answer: (a)

13. The statement

(P) therefore you must listen carefully

(Q) what the speaker has said

(R) in order to understand

(S) will be made just once

(a)  RSPQ

(b)  SPQR

(c)  SPRQ

(d)  SRQP

Answer: (c)

14. Towards midnight

(P) so that the sky was lighted with

(Q) and the clouds drifted away

(R) the rain ceased

(S) the incredible lamp of stars

(a)  RPQS

(b)  RQPS

(c)  SPQR

(d)  SQPR

Answer: (b)

15. Without books

(P) no cultured society is possible

(Q) no fresh ideas are possible

(R) and

(S) without fresh ideas

(a)  PRSQ

(b)  QRSP

(c)  RSPQ

(d)  RSQP

Answer: (b)

Directions (Q. 16-20) : In each of the following questions, a part of the sentence is italicized. Below each sentence, three possible substitutions for the italicized part are given. Choose the one which improves the italicized part. If none of the substitutions improves the italicized part, your answer is (d)

16. Tell your leader that I grant him permission of stay in my kingdom.

(a)  for stay

(b)  about staying

(c)  to stay

(d)  No improvement

Answer: (c)

17. I was delighted to see him fully recover.

(a)  he full recovered

(b)  him fully recovered

(c)  his fully recovery

(d)  No improvement

Answer: (b)

18. While we would like that all Indian children to go to school, we need to ponder why they do not.

(a)  all Indian children

(b)  that all the Indian children

(c)  if all the children of India

(d)  No improvement

Answer: (a)

19. Scarcely had he left when his friend came.

(a)  He had scarcely left

(b)  He had left scarcely

(c)  He scarcely had left

(d)  No improvement

Answer: (d)

20. Dewan has driven almost thrice so far as Mr. Gupta has.

(a)  as far

(b)  as far as

(c)  as much as

(d)  No improvement

Answer: (c)

Directions (Q. 21-25) : In each of the following questions, choose the alternative which is most  nearly the same in meaning to the word given in italics in the sentence.

21. The prisoner has been languishing in the jail for the last many years.

(a)  enjoying

(b)  avoiding

(c)  suffering

(d)  convicted

Answer: (c)

22. Some people adopt excellent manners only to camouflage their real character.

(a)  project

(b)  hide

(c)  misrepresent

(d)  reveal

Answer: (b)

23. The venerated the old teacher.

(a)  humiliated

(b)  teased

(c)  respected

(d)  let go

Answer: (c)

24. Nobody has yet seen the peregrination of heavenly beings on earth.

(a)  visit

(b)  arrival

(c)  travelling

(d)  casting glance on

Answer: (c)

25. He did not make any attempt to placate

(a)  please

(b)  pacify

(c)  serve

(d)  satisfy

Answer: (b)

Directions (Q. 26-30) : In each of the following questions, choose the alternative which is opposite in meaning to the word given in italics in the sentence.

26. My uncle is very wealthy, but rather parsimonious in his habits.

(a)  extravagant

(b)  generous

(c)  careless

(d)  strict

Answer: (a)

27. I am somewhat skeptical about his claim.

(a)  optimistic

(b)  convinced

(c)  credulous

(d)  hopeful

Answer: (b)

28. His writings have been much eulogised by Indian scholars.

(a)  condemned

(b)  flouted

(c)  disapproved

(d)  disparaged

Answer: (a)

29. The boy was accused of theft.

(a)  liberated

(b)  impeached

(c)  exonerated

(d)  sentenced

Answer: (c)

30. This is not ideology but pragmatic language teaching.

(a)  impractical

(b)  improper

(c)  imperfect

(d)  impossible

Answer: (a)

Directions (Q. 31-35) : In each of the following questions, choose the alternative which can be substituted for the given words/sentence.

31. A man who is having the qualities of woman

(a)  Loquacious

(b)  Celibate

(c)  Effeminate

(d)  Epicurean

Answer: (c)

32. A word no longer in use

(a)  Exotic

(b)  Primitive

(c)  Obsolete

(d)  Ancient

Answer: (c)

33. A person who enters without any invitation

(a)  Burglar

(b)  Intruder

(c)  Thief

(d)  Vandal

Answer: (b)

34. The period between two reigns

(a)  Lapse

(b)  Interregnum

(c)  Anachronism

(d)  Intermission

Answer: (b)

35. Poem in short stanzas narrating a popular story

(a)  Ballet

(b)  Epic

(c)  Ballad

(d)  Sonnet

Answer: (c)

Directions (Q. 36-40) : Read the following passage carefully and answer the questions given below it.

Journalism combines writing with news gathering and interpretation. While the journalist’s work obviously varies from newspaper to newspaper and from magazine to magazine, all journalists are as much research workers, as they are writers. They cannot write their news or feature stories, unless they locate it and research them first. They must be able to read the in-between lines of the main source-news and interpret that. A great many stories hunt the journalist, who rejects most of them. he carefully sorts and sifts those, taking only a relatively very small proportion of news, may be, coming from an unexpected source. A successful journalist may gather news to the tune of 100 percent but he can use them for his profession only 3-5 percent. He must be able to see or forecast to himself, the news of tomorrow or the day after, from the news of tomorrow or the day after, from the news of today; because newspapers want advance news or advance warning to give a good, exclusive and exhaustive coverage to anything of interest. But a real ‘scoop’ is a very rare event. And bogus scoops also bring disrepute to the newspaper.

36. According to the passage, a journalist’s work includes

(a)  writing

(b)  news gathering

(c)  sorting out the news

(d)  All of the above

Answer: (d)

37. What, according to the passage, is the common characteristic of all journalists?

(a)  They all write for the newspapers

(b)  They all have to first probe into the news they gather

(c)  They all make an adventure to find a worthy news

(d)  They all can forecast future news

Answer: (b)

38. What is most important for a journalist?

(a)  He must be able to interpret the news correctly

(b)  He must know how to sort out the news

(c)  He must be able to write effectively

(d)  He must know how to gather the right news

Answer: (a)

39. What does the passage say about successful journalists?

(a)  They reject a major portion of the gathered news

(b)  They can use a relatively small portion of the news

(c)  Their news come from unexpected sources

(d)  They present advance news

Answer: (b)

40. What is the basic requisite for exclusive and exhaustive coverage of newspaper?

(a)  Forecast of tomorrow’s news from today’s news

(b)  Collecting the news of a ‘real scoop’

(c)  Selection of a small portion of the gathered news

(d)  Effective style of writing

Answer: (a)

JIPMER Medical Entrance Exam Previous Year Question Paper 2016 With Answer Key

JIPMER MBBS Entrance Exam – 2016

Physics

1. The displacement x of a particle varies with time t as x = ae−αt + beβt, where a, a, b, α and β are positive constants. The velocity of the particle will

(a)   decrease with time

(b)   be independent of α and β

(c)   drop to zero when α − β

(d)   increase with time

Answer: (d)

2. A transformer having efficiency of 75% is working on 220 V and 4.4 kW power supply. If the current in the secondary coil is 5A. What will be the voltage across secondary coil and the current in primary coil?

(a)   VS = 220 V, iP = 20 A

(b)   VS = 660 V, iP = 15 A

(c)   VS = 660 V, iP = 20 A

(d)   VS = 220 V, iP = 15 A

Answer: (c)

3. A fish rising vertically to the surface of water in a lake uniformly at the rate of 2 m/s observes a kingfisher diving vertically towards the water at a rate of 10 m/s. If refractive index of water n = 4/3, what will be the actual velocity o f the kingfisher

(a)   10 m/s

(b)   8 m/s

(c)   6 m/s

(d)   9 m/s

Answer: (c)

4. One mole of a monoatomic ideal gas undergoes the process A → B in the given p-V diagram. The molar heat capacity for this process is

(a)   3R/2

(b)   13R/6

(c)   5R/2

(d)   2R

Answer: (b)

5. A pan with a set of weights is attached to a light spring. The period of vertical oscillation is 0.5s. When some additional weights are put in pan, then the period of oscillations increases by 0.1s. The extension caused by the additional weight is

(a)   5.5 cm

(b)   3.8 cm

(c)   2.7 cm

(d)   1.3 cm

Answer: (c)

6. A cylinder of radius R and length L is placed in a uniform electric field E parallel to the cylinder axis. The total flux for the surface of the cylinder is given by

(a)   2πR2E

(b)   πR2/E

(c)   R/E

(d)   zero

Answer: (d)

7. If A + B = C and that C is perpendicular to What is the angle between A and B, if |A| = |C|

(a)  

(b) 

(c) 

(d)   π rad

Answer: (c)

8. A beam of light travelling along X-axis is described by the electric field  the maximum magnetic force on a charge q = 2e, moving along Y-axis with the speed of 3 × 108 m/s is

(a)   19.2 × 10−17 N

(b)   1.92 × 10−17 N

(c)   0.192 N

(d)   None of these

Answer: (b)

9. A body is whirled in a horizontal circle of radius 25 cm. It has an angular velocity of 13 rad/s. What is its linear velocity at any point on circular path?

(a)   2 m/s

(b)   3 m/s

(c)   3.25 m/s

(d)   4.25 m/s

Answer: (c)

10. In the given figure, potential difference between A and B is

(a)   0

(b)   5 V

(c)   10 V

(d)   15 V

Answer: (c)

11. A block having 12g of element is placed in a room. This element is a radioactive element with half-life of 15 yr. After how many years will there be just 1.5 g of the element in the box?

(a)   40 yr

(b)   45 yr

(c)   20 yr

(d)   15 yr

Answer: (b)

12. In the given figure what will be the coefficient of mutual inductance

(a) 

(b) 

(c) 

(d) 

Answer: (c)

13. An L-C-R series circuit with a resistance of 100 Ω is connected to 200 V (AC source) and angular frequency 300 rad/s. When only the capacitor is removed, then the current lags behind the voltage by 60°. The average power dissipated in original L-C-R circuit is

(a)   50 W

(b)   100 W

(c)   200 W

(d)   400 W

Answer: (d)

14. The intensity of each of the two slits in Young’s double slit experiment is I0. Calculate the minimum separation between the points on the screen, where intensities are 2I0 and I0. If fringe width is b

(a)   b/5

(b)   b/8

(c)   b/12

(d)   b/4

Answer: (c)

15. A metal rod at a temperature of 145°C, radiates energy at a rate of 17 W. If its temperature is increased to 273°C, then it will radiate at the rate of

(a)   49.6 W

(b)   17.5 W

(c)   50.3 W

(d)   67.5 W

Answer: (a)

16. Two wires are stretched through same distance. The force constant of second wire is half as that of the first wire. The ratio of work done to stretch first wire and second wire will be

(a)   2 : 1

(b)   1 : 2

(c)   3 : 1

(d)   1 : 3

Answer: (a)

17. A square wire of side 2.0 c is placed 20 cm in front of a concave mirror of focal length 10 cm with its centre on the axis of the mirror and its plane normal to the axis. The area enclosed by the image of wire is

(a)   7.5 cm2

(b)   6 cm2

(c)   2 cm2

(d)   4 cm2

Answer: (d)

18. A lead bullet penetrates into a solid object and melts. Assuming that 50% of kinetic energy was used to heat it, the initial speed of the bullet is (the initial temperature of the bullet is 25°C and its melting point is 300°C). Latent heat of fusion of lead = 2.5 × 104 J/kg and specific heat capacity of lead = 125 J/kg-K

(a)   100 m/s

(b)   ≈ 490 m/s

(c)   520 m/s

(d)   360 m/s

Answer: (b)

19. A quarter cylinder of radius R and refractive index 1.5 is placed on a table. A point object P is kept at a distance of mR from it as shown in figure. For water value of m for which a ray from P will emerge parallel to the table?

(a)   2/3

(b)   3/2

(c)   4/3

(d)   3/4

Answer: (c)

20. The graph 1/λ and stopping potential (V) of three metals having work function ϕ1, ϕ2 and ϕ3 in an experiment of photoelectric effect is plotted as shown in figure. Which one of the following statement is/are correct? [Here λ is the wavelength of the incident ray]

(i) Ratio of work functions ϕ1 : ϕ2 : ϕ3 = 1: 2 : 4

(ii) Ratio of work functions ϕ1 : ϕ2 : ϕ3 = 4 : 2 : 1

(iii)  , where h = Planck’s constant, c = speed of light

(iv) The violet colour-light can eject photoelectrons from metals 2 and 3

(a)   (i), (iii)

(b)   (i), (iv)

(c)   (ii), (iii)

(d)   (i), (ii) and (iv)

Answer: (a)

21. The concentric, conducting spherical shells X, Y and Z with radii r, 2r and 3r, respectively. X and Z are connected by a conducting wire and Y is uniformly charged to charge Q as shown in figure. Charges on shells X and Z will be

(a) 

(b) 

(c) 

(d) 

Answer: (b)

22. The temperature of gas is raised from 27°C to 927°C. The root mean square speed

(a)   gets halved

(b)   gets doubled

(c)   is  the earlier value

(d)   remains the same

Answer: (b)

23. A particles slides down on a smooth incline of inclination 30°, fixed in an elevator going up with an acceleration 2 m/s2. The box of incline has a length 4m. The time taken by the particle to reach the bottom will be

(a) 

(b) 

(c)  

(d) 

Answer: (c)

24. A stone projected with a velocity u at an angle θ with the horizontal reaches maximum height H1. When it is projected with velocity u at an angle  with the horizontal, it reaches maximum height H2. The relation between the horizontal range R of the projectile, H1 and H2 is

(a) 

(b)   R = 4(H1 – H2)

(c)   R = 4(H1 + H2)

(d) 

Answer: (a)

25. Two batteries of emf 3V and 6V with internal resistances 2Ω and 4Ω are connected in a circuit with resistance of 10Ω as shown in figure. The current and potential difference between the points P and Q are

(a) 

(b) 

(c) 

(d)  

Answer: (d)

26. A light string passes over a frictionless pulley. To one of its ends a mass of 8 kg is attached. To its other end two masses of 7 kg each are attached. The acceleration of the system will be

(a)   10.2 g

(b)   5.10 g

(c)   20.36 g

(d)   0.27 g

Answer: (d)

27. A capillary tube of length L and radius r is connected with another capillary tube of the same length but half the radius in series. The rate of steady volume flow of water through first capillary tube under a pressure difference of p is V. The rate of steady volume flow through the combination will be (the pressure difference across the combination is p)

(a) 17 V

(b) 

(c)  

(d) 

Answer: (c)

28. A system consist of a cylinder surrounded by a cylindrical shell. A cylinder is a radius R and is made of material of thermal conductivity K, whereas a cylindrical shell has inner radius R and outer radius 2R and is made of material of thermal conductivity twice as that of cylinder. Assuming the system in steady state and negligible heat loss across the cylindrical surface, find the effective thermal conductivity of the system, if the two ends of the combined system are maintained at two different temperatures.

(a)   3K

(b) 

(c)   7K/4

(d)   5K/4

Answer: (c)

29. An isotropic point source emits light with wavelength 500 nm. The radiation power of source is P = 10 W. Find the number of photons passing through unit area per second at a distance of 3 m from the source.

(a)   5.92 × 1017/m2 s

(b)   2.23 × 1017/m2 s

(c)   2.23 × 1018/m2 s

(d)   5.92 × 1018/m2 s

Answer: (b)

30. If a proton and anti-proton come close to each other and annihilate, how much energy will be released?

(a)   1.5 × 10−10 J

(b)   3 × 10−10 J

(c)   4.5 × 10−10 J

(d)   2 × 10−10 J

Answer: (b)

31. A body a rest slides down a 30° inclined plane. The time taken by it to slide down is twice the time it takes when it slides down the same distance in the absence of friction. The coefficient of friction between the body and the inclined plane is

(a)   0.43

(b)   0.37

(c)   0.64

(d)   0.75

Answer: (a)

32. A bat emitting an ultrasonic wave of frequency 4.5 × 104 Hz at speed of 6 m/s between two parallel walls. The two frequencies heared by the bat will be

(a)   4.67 × 104 Hz, 4.34 × 104 Hz

(b)   4.34 × 104 Hz, 4.67 × 104 Hz

(c)   4.5 × 104 Hz, 5.4 × 104 Hz

(d)   4.67 × 103 Hz, 4.34 × 104 Hz

Answer: (b)

33. The charges on two spheres are +7 μC and −5 μC respectively. They experience of force F. If each of them is given an additional charge of −2μC, then the new force of attraction will be

(a)   F

(b)   F/2

(c)   F/√3

(d)   2F

Answer: (a)

34. A rod made up of metal is 1.2 m long and 0.8 cm diameter. Its resistance is 3.5 × 10−3 Ω. Another disc made of the same metal is 2.0 cm in diameter and 1.25 mm thick. What is the resistance between the round faces of the disc?

(a)   1.35 × 10−8 Ω

(b)   2.70 × 10−7 Ω

(c)   5.82 × 10−7 Ω

(d)   8.10 × 10−5 Ω

Answer: (c)

35. A body is moving along a rough horizontal surface with an initial velocity of 10 ms−1. If the body comes to rest after travelling a distance of 12 m, then the coefficient of sliding friction will be

(a)   0.5

(b)   0.2

(c)   0.4

(d)   0.6

Answer: (c)

36. The magnification produced by a astronomical telescope for normal adjustment is 10 and the length of the telescope is 1.1 m. The magnification, when the image is formed atleast distance of distinct vision is

(a)   6

(b)   18

(c)   16

(d)   14

Answer: (d)

37. A circular current carrying coil has a radius R. The distance from the centre of the coil, on the axis, where B will be 1/8 of its value at the centre of the coil is

(a)   R/√3

(b)   √3 R

(c)   2√3 R

(d)   2R/√3

Answer: (b)

38. Angular width of central maximum in the Fraunhoffer diffraction pattern of a slit is measured. The slit is illuminated by light of wavelength 6000 Å. When the slit is illuminated by light of another wavelength, then the angular width decreases by 30%. The same decrease in angular width of central maximum is obtained when the original apparatus is immersed in a liquid. The refractive index of the liquid will be

(a)   1.25

(b)   1.42

(c)   1.67

(d)   1.5

Answer: (b)

39. An energy of 68.0 eV is required to excite a hydrogen-like atom in its second Bohr energy level to third energy level the charge of nucleus is Ze. The wavelength of a radiation required to eject the electron from first orbit to infinity is

(a)   2.2 nm

(b)   2.85 nm

(c)   3.2 nm

(d)   2.5 nm

Answer: (d)

40. A current carrying loop is placed in a uniform magnetic field in four different orientations I, II, III and IV as shown in figure. Arrange them in decreasing order of potential energy.

(a)   I > III > II > IV

(b)   I > II > III > IV

(c)   I > IV > II > III

(d)   III > IV > I > II

Answer: (c)

41. Two different isotherms representing the relationship between pressure p and volume V at a given temperature of the same ideal gas are shown for masses m1 and m2, then

(a)   Nothing can be predicted

(b)   m1 < m2

(c)   m1 = m2

(d)   m1 > m2

Answer: (b)

42. A7 Li target is bombarded with a proton beam current of 10−4 A for one hour to produce 7Be of activity 1.8 × 108 disintegrations per second. Assuming that one 7Be radioactive nuclei is produced by bombarding 1000 protons, its half-life is

(a)   0.87 × 107 s

(b)   0.2 × 107 s

(c)   0.67 × 108 s

(d)   0.87 × 106 s

Answer: (a)

43. In the given figure, the capacitors C1, C3, C4, C5 have a capacitance 4 μF each. If the capacitor C2 has a capacitance 10 μF, then effective capacitance between A and B will be

(a)   2 μF

(b)   6 μF

(c)   4 μF

(d)   8 μF

Answer: (c)

44. The truth table for the following logic circuit is

(a) 

(b) 

(c) 

(d) 

Answer: (a)

45. A sphere of mass m moving with velocity v hits inelastically with another stationary sphere of same mass. The ratio of their final velocities will be (in terms of e)

(a) 

(b) 

(c) 

(d) 

Answer: (b)

46. A small spherical drop fall from rest in viscous liquid. Due to friction, heat is produced. The correct relation between the rate of production of heat and the radius of the spherical drop at terminal velocity will be

(a) 

(b) 

(c) 

(d) 

Answer: (d)

47. A galvanometer of resistance 25 Ω shows a deflection of 5 divisions when a current of 2 mA is passed through it. If a shunt of 4Ω is connected and there are 20 divisions on the scale, then the range of the galvanometer is

(a)   1 A

(b)   58 A

(c)   58 mA

(d)   30 mA

Answer: (c)

48. The total charge induced in a conducting loop when it is moved in magnetic field depends on

(a)   the rate of change of magnetic flux

(b)   initial magnetic flux only

(c)   the total change in magnetic flux

(d)   final magnetic flux only

Answer: (c)

49. Particles of masses m, 2,, 3, …, nm are placed on the same line at distances L, 2L, 3L, …, nL from O. The distance of centre of mass from O is

(a) 

(b) 

(c)  

(d) 

Answer: (d)

50. The length of a given cylindrical wire is increased by 150%. Due to consequent decrease in diameter the change in the resistance of the wire will be

(a)   200%

(b)   525%

(c)   300%

(d)   400%

Answer: (b)

51. An ideal solenoid having 5000 turns/m has an aluminium core and carries a current of 5A. If χAl = 2.3 × 10−5, then the magnetic field developed at centre will be

(a)   0.031 T

(b)   0.048 T

(c)   0.027 T

(d)   0.050 T

Answer: (a)

52. A ball of radius R rolls without slipping. Find the fraction of total energy associated with its rotational energy, if the radius of the gyration of the ball about an axis passing through its centre of mass is K.

(a) 

(b) 

(c)  

(d) 

Answer: (a)

53. A capacitor is charged and then made to discharge through a resistance. The time constant is τ. In what time will be potential difference across the capacitor decrease by 10%?

(a)   τ ln 0.1

(b)   τ ln 0.9

(c)  

(d)  

Answer: (c)

54. A body of mass 2 m is placed on earth’s surface. Calculate the change in gravitational potential energy, if this body is taken from earth’s surface to a height of h, where h = 4R.

(a) 

(b)  

(c) 

(d) 

Answer: (c)

55. The slope of isothermal and adiabatic curves are related as

(a)   isothermal curve slope = adiabatic curve slope

(b)   isothermal curve slope = γ × adiabatic curve slope

(c)   adiabatic curve slope = γ × isothermal curve slope

(d)   adiabatic curve slope = 1/2 × isotherma curve slope

Answer: (c)

56. A solid sphere of mass M and radius 2R rolls down an inclined plane of height h without slipping. The speed of its centre of mass when it reaches the bottom is

(a)  

(b)  

(c) 

(d) 

Answer: (c)

57. A liquid of density 800 kg/m3 is filled in a cylindrical vessel upto a height of 3 m. This cylindrical vessel stands on a horizontal plane. There is circular hole on the side of the vessel. What should be the minimum diameter of the hole to move the vessel on the floor, if plug is removed. Take the coefficient of friction between the bottom of the vessel and the p lane as 0.5 and total mass of vessel plus vessel as 95 kg.

(a)   0.107 m

(b)   0.053 m

(c)   0.206 m

(d)   0.535 m

Answer: (a)

58. The transfer ratio β of a transistor is 50. The input resistance of the transistor when used in the common emitter configuration is 2 kΩ. The peak value of the collector AC current for and AC input voltage of 0.02 V peak is

(a)   200 μA

(b)   0.01 mA

(c)   0.25 mA

(d)   500 μA

Answer: (d)

59. The volume of an ideal gas is doubled in an isothermal process. Then, which of the following is true?

(a)   Work done by the gas is positive

(b)   Work done by the gas in negative

(c)   Internal energy of the system decreases

(d)   Internal energy of the system increases

Answer: (a)

60. A prism of a certain angle deviates the red and blue rays by 8 and 12, respectively. Another prism of the same angle deviates the red and blue rays by 10 and 14, respectively. The prism are small angled and made of different materials. The dispersive power of the materials of the prisms are in the ratio

(a)   5 : 6

(b)   9 : 11

(c)   6 : 5

(d)   11 : 9

Answer: (c)

JIPMER MBBS Entrance Exam – 2016

Chemistry

1. Temperature of a gas is t K. What would be the temperature at which volume and pressure, both will reduced to half of the initial values?

(a)   t/2

(b)   t/4

(c)   t/3

(d)   t/8

Answer: (b)

2. What will be the number of waves formed by a Bohr electron in one complete revolution in its second orbit?

(a)   Three

(b)   Two

(c)   One

(d)   Zero

Answer: (b)

3. When 2-methyl butyl bromide is treated with sodium ethoxide in ethanol, what will be the major product?

(a)   2-methyl but-2-ene

(b)   3-methyl but-1-ene

(c)   2-methyl but-1-ene

(d)   2-methyl sodium-butoxide

Answer: (a)

4. Solid NaHCO3 will be neutralized by 40.0 mL of 0.1 M H2SO4 What would be the weight of solid NaHCO3 in gram?

(a)   0.672 g

(b)   6.07 g

(c)   17 g

(d)   20 g

Answer: (a)

5. The pair of boiling point and compound are given as,

Which will show lowest vapor pressure at room temperature?

(a)   C6H6

(b)   CH3OH

(c)   C6H5NO2

(d)   C6H5NH2

Answer: (c)

6. In the following conversion,

Identify ‘A’ from the following option

(a)   NaBD4

(b)   LiAlD4

(c)   Mg, ether/D2O

(d)   BH3, D2O

Answer: (c)

7. Ethers like ROR can be cleaved by concentrated HI but not by HCl because

(a)   I is a weaker nucleophile than Cl

(b)   I is stronger nucleophile than Cl

(c)   SN1 mechanism carried out in this reaction is rapidly in presence of HI.

(d)   None of the above

Answer: (b)

8. In the reaction,

How many electron donating groups are attached with the carbon atom of unsaturated part of the product ‘B’?

(a)   Two

(b)   Three

(c)   Four

(d)   None of these

Answer: (a)

9. At 27°C one mole of an ideal gas is compressed isothermally and reversibly from a pressure of 2 atm to 10 atm.

Choose the correct option from the following.

(a)   Change in internal energy is positive

(b)   Heat is negative

(c)   Work done is −965.84 cal

(d)   All are incorrect

Answer: (b)

10. Which of the following hydrocarbons is the most reactive towards addition of H2SO4?

(a)   Ethene

(b)   Propylene

(c)   3-methyl but-1-ene

(d)   1-butene

Answer: (c)

11. Choose the incorrect statement about noble gas?

(a)   Boiling point increases with increasing atomic mass

(b)   Helium has least tendency to form compound

(c)   Noble gases have some value of electron affinity

(d)   Xenon has maximum number of compounds

Answer: (c)

12. For the cell reaction

Pb + Sn2+ → Pb2+ + Sn

Given that, Pb → Pb2+, E° = 0.13 V

Sn2+ + 2e → Sn; E° = −0.14 V

What would be the ratio of cation concentration for which E = O?

(a)   1/4

(b)   1/2

(c)   1/3

(d)   1/1

Answer: (b)

13. [CuCl4­]2− exists while [Cul4]2− does not exist, because

(a)   I is stronger reductant than Cl

(b)   I is weaker reductant than Cl

(c)   I is stronger oxidant than Cl

(d)   None of the above

Answer: (a)

14. In the reaction,

X + I2 + 2HCl → SnCl4 + 2HI

The correct option regarding X is /are

(a)   It is an strong reducing agent

(b)   It is an angular molecule

(c)   It is used as a reagent in test of Hg2+ radical

(d)   All of the above

Answer: (d)

15. If oxide ions are arranged in hcp and the aluminium ions occupy two thirds of octahedral voids. What will be the formula of the compound?

(a)   AlO3

(b)   Al2O3

(c)   (Al2O3)

(d)   Al3O4

Answer: (b)

16. Actinides exhibit larger number of oxidation state than that of corresponding lanthanides. The reason behind this aspect is

(a)   lesser energy difference between 5f and 6d-orbitals than between 4f and 5d-orbitals

(b)   larger atomic size of actinides than the lanthanides

(c)   more energy difference between 5f and 6d orbitals than between 4f and 5d-orbitals

(d)   greater reactive nature of the actinides than the lanthanides

Answer: (a)

17. Aqueous solution of AlCl3 is acidic towards litmus while of NaCl is not. The correct reason behind this is

(a)   AlCl3 furnishes OH ion in the solution

(b)   AlCl3 furnishes H+ ion in the solution

(c)   AlCl3 furnishes both H+ well as OH ion in the solution

(d)   AlCl3 is the salt of strong base and strong acid

Answer: (b)

18. The melting point of solid substances is directly proportional to pressure acting on them. However, ice-melts at a temperature lower than its usual melting point, when the pressure increases. This is because

(a)   ice is not a true solid

(b)   the bonds break under pressure

(c)   ice is loss dense than water

(d)   pressure generates heat

Answer: (c)

19. The E° values for Mn and Zn are more negative than expected because negative than expected because

(a)   they have either half-filled and fully-filled configurations

(b)   they can easily donate electrons

(c)   it is Quite easy to remove electrons from their orbitals

(d)   None of the above

Answer: (a)

20. For the formation of Cr2O3 and Al2O3, values of ∆fG° are 540 kJ mol−1 and −827 kJ mol−1 What will be the possibility for reaction of Cr2O3 by Al?

(a)   Reduction of Cr2O3 by Al will take place

(b)   Oxidation of Cr2O3 by Al will take place

(c)   Neither oxidation nor reduction will take place

(d)   Reaction is not feasible

Answer: (a)

21. Work is being performed, when a weight lifter lifts a base ball off a weight rack. This is due to

(a)   magnetic attraction

(b)   gravity

(c)   electrostatic repulsion

(d)   mechanical force

Answer: (b)

22. Magnetic moment of [Ti(H2­O)6]4+, [Mn(H2O)6]2+ and [Cr(H2O)6]3+ can be represented as X, Y and Z.

They are in order of

(a)   X < Z < Y

(b)   Z < Y < X

(c)   X < Y < Z

(d)   Z < Y < X

Answer: (a)

23. Beryllium differs in properties from other elements of its own group but shows resemblance with aluminium because of

(a)   relatively bigger ionic radius and high polarizing power of Be

(b)   relatively smaller ionic radius and high polarizing power of Be

(c)   relatively bigger ionic radius is the only reason behind this

(d)   None of the above

Answer: (b)

24. Two similar reactions have the same rate constant at 25°C but at 35°C, one of the reaction has a higher rate constant than the other. The appropriate reason for this is

(a)   due to effective collisions

(b)   due to different activation energies

(c)   due to different threshold energies

(d)   due to higher population of molecules

Answer: (b)

25. In [CO(H2O)6]2+, there are three unpaired electrons present. The μexperimental of 4.40 BM.

This is because of

(a)   increase in number of unpaired electrons

(b)   some contribution of the orbital motion of the electron to the magnetic moment

(c)   change in orbital spin of the electron

(d)   d-d transition

Answer: (b)

26. When excess of NaOH solution is added to aqueous solution of iodine, the colour of solution becomes

(a)   blue

(b)   yellow

(c)   colourless

(d)   pale green

Answer: (c)

27. Look at the graph,

Choose the correct equation from the following which best suited to the above graph

(a)   [At] = [A]0 – Kt

(b)   [At] = [A]0 + Kt

(c)   [At] = [A0]e−Kt

(d)   [At] = Kt2 + [A0]

Answer: (b)

28. Choose the correct statement from the following

(a)   NH4+ and CH4 are not isoelectronic species

(b)   BF3 does not have dipole moment

(c)   O – Cl – O obeys octet rule

(d)   O in O3 is sp3-hybridized

Answer: (b)

29. Lauryl alcohol on treatment with H2SO4 followed by neutralization forms a product which is

(a)   cationic detergent

(b)   anionic detergent

(c)   neutral detergent

(d)   antidepressant

Answer: (b)

30. When an iron object is plated with tin, tin does not act as sacrificial anode in protecting against corrosion, because

(a)   tin is more reactive than iron

(b)   tin is less reactive than iron

(c)   reactivity of tin and iron is same

(d)   tin is oxidizing agent while iron is not so

Answer: (b)

31. Which of the following pair of compounds will have three sp2-hybride orbitals?

(a)   SO2, CH4

(b)   SO3, C2H­4

(c)   BF3, SF4

(d)   I3, SF4

Answer: (b)

32. Consider the reaction,

H2 + X2 → 2HX, where X is halogen. The reactivity of halogens decrease in the order of

(a)   Cl2 > F2 > Br2 > I2

(b)   I2 > Br2 > F2 > Cl2

(c)   F2 > Cl2 > Br2 > I2

(d)   I2 > Br2 > Cl2 > F2

Answer: (c)

33. Choose the correct statement from the following.

(a)   The greater positive value of E°OP indicates greater reactivity of metal

(b)   F is strong oxidant while Cu2+ is weak reductant

(c)   The metals placed above Mg in the electrochemical series do not decompose water at ordinary temperature

(d)   Oxides of Hg do not decompose on heating

Answer: (a)

34. 100 mL of a solution contains 2 g of acetic acid and 3g of sodium acetate providing Ka = 1.8 × 10−5, then choose the correct option.

(a)   This solution is basic in nature

(b)   This solution is acidic in nature

(c)   This solution is amphoteric in nature

(d)   This solution is neutral in nature

Answer: (b)

35. Consider the following structures.

Choose the correct statement regarding the above structures.

(a)   Dipole moment varies as II > III > I

(b)   II is more stable than I

(c)   I is the most reactive among three

(d)   All of the above

Answer: (d)

36. On the basis of Langmuir adsorption isotherm the amount of gas adsorbed at very high pressure.

(a)   Reaches a constant limiting value

(b)   Goes on increasing with pressure

(c)   Goes on decreasing with pressure

(d)   First increasing and then decreasing with pressure

Answer: (a)

37. Both Mg and Fe metal can reduce copper from a solution having Cu2+ ion, according to equilibria.

Mg(s) + Cu2+ ⇌ Mg2+ + Cu(s); K1 = 6 × 1090

Fe(s) + Cu2+ ⇌ Fe2+ + Cu­(s); K2 = 3 × 1026

Choose the correct option regarding above equilibrium

(a)   Mg removes more Cu2+ from solution

(b)   Fe removes more Cu2+ from solution

(c)   Both will equally remove Cu2+ from solution

(d)   Both metals cannot remove Cu2+ from solution

Answer: (a)

38. Consider the following reaction sequence

Choose the correct option regarding the different products obtained in the above reaction sequence.

(a)   ‘D’ is an alcohol consisting one carbon more than the starting alcohol

(b)   Product ‘B’ is formed via SN2-pathway

(c)   ‘C’ is an amine

(d)   All of the above

Answer: (d)

39. Which of the following is crossed aldol product in the reaction?

(a) 

(b) 

(c) 

(d) 

Answer: (c)

40. Which of the following statements is incorrect regarding the reaction?

(a)   The equivalent weight of CH3CHO is 2

(b)   Three moles of OH are required in the above reaction

(c)   CH3CHO is an oxidizing agent

(d)   Reduction of [Ag(NH3)2] occurs

Answer: (c)

41. Consider the following compounds.

Cl3C – COOH   Br3C – COOH  I3C – COOH

         (I)                     (II)               (III)

The decreasing order of decarboxylation is

(a)   I > II > III

(b)   III > II > I

(c)   III > I > II

(d)   II > I > III

Answer: (a)

42. Which of the following is an example of carbylamines reaction?

(a) 

(b) 

(c) 

(d)   Ph2 – NH + HONO → Ph2 – N – N = O + H2O

Answer: (c)

43. C3H9N reacts with Hinsberg reagent and the product is insoluble in alkali but soluble in ether. This nitrogen containing compound is

(a)   primary amine

(b)   secondary amine

(c)   tertiary amine

(d)   methyl isocyanide

Answer: (b)

44. 

Product ‘Q’ is

(a)   an amide

(b)   an amine

(c)   nitro compound

(d)   nitrile compound

Answer: (a)

45. Which of the following statements is incorrect?

(a)   Polypropylene is a thermoplastic polymer

(b)   Melamine-formaldehyde is a thermosetting polymer

(c)   Mixture of styrene and methyl methacrylate can form ionic addition polymer

(d)   Low-density polythene is a poor conductor of electricity

Answer: (c)

46. Match the particle with its characteristic.

(a)   A – p; B – r; C – q; D – s

(b)   A – p; B – q; C – r; D – s

(c)   A – r; B – s; C – p; D – q

(d)   A – s; B – r; C – p; D – q

Answer: (a)

47. The magnitude of screening effect depends upon the number of

(a)   inner electrons

(b)   outer electrons

(c)   bond order

(d)   Both (a) and (b)

Answer: (a)

48. Highest energy will be absorbed to eject out the electron in the configuration.

(a)   1s2 2s2 2p1

(b)   1s2 2s2 2p3

(c)   1s2 2s2 2p2

(d)   1s2 2s2 2p4

Answer: (b)

49. Sugars are separated by using the solvent BAW (n-butanol acetic acid – H2O) and detected by spraying the plate with

(a)   aniline hydrogen phthalate solution

(b)   hydrogen peroxide solution

(c)   crystals of I2

(d)   cupric oxide

Answer: (a)

50. Duma’s method involves the determination of nitrogen content in the organic compound in the form of

(a)   NH3

(b)   N2

(c)   NaCN

(d)   (NH4)2SO4

Answer: (b)

51. Which of the following statement(s) is not correct?

(a)   Suspended particulate matter is an important pollutant released by diesel vehicles

(b)   Soot particles (size < 5μ) cause fibrosis of the lung living

(c)   H2SO4 particulates have size of 500 – 1000 nm

(d)   Photochemical smog is formed by oxides of sulphur, smoke and dust particles

Answer: (d)

52. At radioactive equilibrium, the ratio between two atoms a radioactive elements A and B is 3.1 ×109 : 1. If the half-life period of A is 2 × 1010 yrs, then the half-life of B is

(a)   9.54 yrs

(b)   2.14 yrs

(c)   3.29 yrs

(d)   6.45 yrs

Answer: (d)

53. The effective neutron capture radius of a nucleus having a cross-section of 1.0 barr is [ Given, 1 barr = 1.0 × 10−24 cm2]

(a)   5.6 × 10−13 cm

(b)   4.3 × 10−13 cm

(c)   2.3 × 10−11 cm

(d)   5.6 × 10−24 cm

Answer: (a)

54. Lime stone is added in the extraction of lead from galena why?

(a)   It prevents the formati Con of PbSO4

(b)   It remove the impurity of silica as fusible slag

(c)   It converts lead silicate to lead oxide

(d)   All of the above

Answer: (d)

55. A hypothetical reaction A → 2B proceeds through the following sequence of steps

(I) A → C; ∆rH = q1

(II) C → D; ∆rH = q2

The heat of hypothetical reaction is

(a)   q1  + q2 – 2q3

(b)   q1 + q2 + 2q3

(c)   q1 + 2q2 – 2q3

(d)   q1 – q2 + 2q3

Answer: (b)

56. Anhydrous mixture of KF and HF contain which types of ions?

(a)   K+, H+, F

(b)   {KF+, (HF)}

(c)   KH+, F

(d)   K+, HF2

Answer: (d)

57. Polypropylene can be obtained by polymerization of

(a)   CH2 = CH – CH2 – Cl

(b)   CH2 = CH – Cl

(c)   CH3 – Cl

(d)   CH3 – CHCl2

Answer: (c)

58. (X), (X) formed in the above reaction is

(a) 

(b)   (C6H5)2NOH

(c) 

(d) 

Answer: (c)

59. 2 L of gas at STP Weight 14g.

The gas would be

(a)   H2

(b)   CO

(c)   B2H6

(d)   All of these

Answer: (d)

60. The value of n in

MnO4 + 8H+ + ne → Mn2+ + 4H2O is

(a)   5

(b)   4

(c)   2

(d)   3

Answer: (a)

JIPMER MBBS Entrance Exam – 2016

Biology

1. ‘GIFT’ refers to

(a)   Gamete ln vitro Fertilization Technique

(b)   Gamete Inter Fallopian Tube

(c)   Gamete Intra Fallopian Transfer

(d)   Gamete ln vitro Fertilization Transfer

Answer: (c)

2. Consider the following statements and Choose correct ones from given options.

(I) Shark do not have any bone in its body.

(II) Water snake and salamander belongs to same class and have largest RBC.

(III) Silver fish is a true fish while cuttle and star fishes are mollusks and echinoderms respectively.

(IV) Ornithorhynchus is a connecting link between reptiles and mammals.

(a)   I, II and IV

(b)   I and IV

(c)   I, II and III

(d)   III and IV

Answer: (b)

3. Which one of the following is not correctly matched?

(a)   Diphtheria – Corynebacerium

(b)   Elephantiasis – Wuchereria

(c)   Plague – Paramyxo

(d)   Lockjaw – Clostridium

Answer: (c)

4. The largest muscle in the human is

(a)   biceps

(b)   gluteus maximus

(c)   stapedius

(d)   masseter

Answer: (b)

5. The best description on natural selection is

(a)   the reproductive success of the members of a population best adapted to environment

(b)   it acts when the resources are unlimited

(c)   a change in the proportion of variations within a population

(d)   it follows Hardy-Weinberg principle

Answer: (a)

6. The chromosome of cell duplicate during the

(a)   S-phase of cell cycle

(b)   G1-phase of cell cycle

(c)   G2-phase of cell cycle

(d)   prophase of cell division

Answer: (a)

7. Adenosine triphosphate was discovered by

(a)   Jack Lipman

(b)   A Bloor

(c)   Karl Lohmann

(d)   Emil Fisher

Answer: (c)

8. Lens of eyes is derived from

(a)   ectoderm

(b)   mesoderm

(c)   endoderm

(d)   Both (b) and (c)

Answer: (a)

9. Schneiderian and tympanic membrane respectively belongs to

(a)   ear and nose

(b)   ear and eye

(c)   ear and ear

(d)   nose and ear

Answer: (d)

10. Due to alcohol abuse brain related disease Korsakkoff’s syndrome occurs. Which nutrient is responsible for this syndrome?

(a)   Vitamin-B12

(b)   Fat

(c)   Protein

(d)   Vitamin-B1

Answer: (d)

11. Who received Nobel Prize in 2008 for the discovery of HIV?

(a)   Harald Zur Hausen

(b)   Luc Montagnier

(c)   Jack Szostak

(d)   Carol Greider

Answer: (b)

12. Which one of the following is a reorganization site for restriction enzyme Bam HI?

(a)   5ʹ– GAATTC – 3ʹ 3ʹ – CTTAAG – 5ʹ

(b)   5ʹ – GGATCC – 3ʹ 5ʹ – CCATGG – 3ʹ

(c)   5ʹ – GGATCC -– 3ʹ 3ʹ – CCTAGG – 5ʹ

(d)   5ʹ– GAATTC – 3ʹ 5ʹ– GTTAAC – 3ʹ

Answer: (c)

13. Golden ages of reptiles and fishes are respectively

(a)   Mesozoic and Devonian

(b)   Jurassic and Permian

(c)   Triassic and Silurian

(d)   Palaeozoic and Mesozoic

Answer: (a)

14. Match the following Columns.

(a)   A – 3; B – 4; C – 1; D – 2

(b)   A – 2; B – 3; C – 4; D – 1

(c)   A – 3; B – 4; C – 2; D – 1

(d)   A – 3; B – 1; C – 4; D – 2

Answer: (d)

15. Which one of the following pair is not correctly matched?

(a)   Almond-Drupe

(b)   Brinjal-Berry

(c)   Guava-Pepo

(d)   Loquat-Pome

Answer: (c)

16.The powerhouse of cells is first discovered by

(a)   C Benda in 1897

(b)   Kolliker in 1850

(c)   Claude in 1880

(d)   Kingsburg in 1882

Answer: (b)

17. During chloride shift or Hamburger phenomenon, when the whole blood is saturated with CO2, following changes occurs. Which one of them is not correct?

(a)   Bicarbonate content of plasma and corpuscles increase

(b)   Chloride content of plasma diminished and that of the cell is increased

(c)   Total base of blood remain unchanged

(d)   Water content and volume of corpuscles decrease

Answer: (d)

18. Which one of the following elements is activator for both ribulose biphosphate, carboxylase oxygenase and phosphoenol pyruvate carboxylase in photosynthetic carbon fixation?

(a)   Mg2+

(b)   Zn2+

(c)   Mn2+

(d)   Cl and K+

Answer: (a)

19. Common phase in aerobic and anaerobic respiration is

(a)   Kreb’s cycle

(b)   glycolysis

(c)   glycogenolysis

(d)   ETS

Answer: (b)

20. ‘Treponema pallidum’ pathogen is a cause of

(a)   leprosy

(b)   plague

(c)   syphilis

(d)   pertussis

Answer: (c)

21. Which one of the following pair is not correct?

(a)   Mangolian idiocy – 21st chromosome

(b)   Patau syndrome – 13th chromosome

(c)   Cri-Du-Chat – 11th chromosome

(d)   Edward syndrome – 18th chromosome

Answer: (c)

22. Which one of the following pair is not correct?

(a)   single-stranded DNA

(b)   single-stranded RNA

(c)   double-stranded DNA

(d)   None of the above

Answer: (a)

23. An irregular mode of reproduction resulting in the development of an embryo without fertilization is called

(I) parthenogenesis

(II) apogamy

(III) sporophytic budding

Select the correct answer  using the code given below.

(a)   Only I

(b)   Only II

(c)   II and III

(d)   I, II and III

Answer: (d)

24. In which one of the family formation of endosperm doesn’t take place?

(a)   Orchidaceae

(b)   Cactaceae

(c)   Ranunculaceae

(d)   Malvaceae

Answer: (a)

25. In cardiac cycle maximum time is taken by

(a)   atria systole

(b)   atria diastole

(c)   ventricle systole

(d)   ventricle diastole

Answer: (b)

26. A horse and a donkey can breed to produce mule which is an infertile animal. The infertility is because horse and donkey belong to different

(a)   class

(b)   order

(c)   species

(d)   genus

Answer: (c)

27. Source of commercial chewing gum latex is

(a)   Hevea brasiliensis

(b)   Carica papaya

(c)   Ficus elastica

(d)   Achras sapota

Answer: (d)

28. Compare the statement I and II and choose the correct option.

Statement I In the flowering plants due to higher accumulation of auxins dormancy of lateral buds occurs.

Statement II In Maryland Mammoth (a tobacco variety) flowering occurred at different time at different latitude due to gibberellins concentration.

(a)   Statement I is true, but II is false

(b)   Statement I is false, but II is true

(c)   Both statements are true

(d)   Both statements are false

Answer: (a)

29. Which one of the given pollination technique/adaptation is different than others?

(a)   Herkogamy

(b)   Geitonogamy

(c)   Dichogamy

(d)   Heterostyly

Answer: (b)

30. The phenotypic ratio of trihybrid cross in F2-generation is

(a)   27 : 9 : 9 : 9 : 3 : 3 : 3 : 1

(b)   9 : 3 : 3 : 1

(c)   1 : 4 : 6 : 4 : 1

(d)   27 : 9 : 3 : 3 : 9 : 1 : 2 : 1

Answer: (a)

31. A point mutation where guanine is replaced by cytosine is also called

(a)   frame shift mutation

(b)   transition mutation

(c)   translocation mutation

(d)   transversion mutation

Answer: (d)

32. Match the following Columns.

(a)   A – 4; B – 3; C – 2; D – 1

(b)   A – 2; B – 3; C – 1; D – 4

(c)   A – 3; B – 1; C – 2; D – 4

(d)   A – 4; B – 2; C – 1; D – 3

Answer: (b)

33. Match the following Columns.

(a)   A – 1; B – 4; C – 2; D – 3

(b)   A – 3; B – 4; C – 1; D – 2

(c)   A – 1; B – 3; C – 2; D – 4

(d)   A – 3; B – 1; C – 4; D – 2

Answer: (d)

34. Which one of the following antibodies plays an important role as mediator in allergic response?

(a)   IgE

(b)   IgG

(c)   IgD

(d)   IgA

Answer: (a)

35. Which one of the following human ancestors is known as tool maker?

(a)   Homo erectus

(b)   Java man

(c)   Homo habilis

(d)   Heidelberg man

Answer: (c)

36. Which one of the following is a matching pair of vector and the disease?

(a)   Culex – Filariasis

(b)   Housefly – Leprosy

(c)   Aedes aegypti – Chickenpox

(d)   Sandfly – Cholera

Answer: (a)

37. Consider these following sentences and choose the correct ones.

(I) Each gene contains a specific promoter region and a leader sequence for guiding the beginning of transcription.

(II) Only one strand of DNA, called template strand is copied by RNA polymerase this strand runs in 3ʹ → 5ʹ direction.

(III) RNA polymerase adds complementary nucleotides forming single strand mRNA in 3ʹ → 5ʹ direction.

(IV) Section of DNA that has been transcribed is rewound into its original configuration.

(a)   II, III and IV

(b)   II and IV

(c)   I, II and IV

(d)   I, II, and IV

Answer: (d)

38. Which one of the following is not correct pair of type of cancer and origin place?

(a)   Benign tumour – Non-cancerous tumour

(b)   Carcinomas – Cancer of epithelial tissues

(c)   Lymphomas – Haematopoietic cells tumour

(d)   Sarcomas – Cancer of glands (secretory tissues)

Answer: (d)

39. In animals normally which organism has maximum number of chromosomes?

(a)   Butterfly

(b)   Elephant

(c)   Hermit crab

(d)   Chimpanzee

Answer: (a)

40. During meiosis-I, the bivalent chromosomes clearly appear as tetrads during

(a)   diakinesis

(b)   diplotene

(c)   pachytene

(d)   zygotene

Answer: (c)

41. Snapdragon flower is an exception of Mendel’s laws. It is a good example of

(a)   law of dominance

(b)   complementary gene

(c)   codominance

(d)   incomplete dominance

Answer: (d)

42. Consider these sentences regarding to the structure and nature of DNA.

(I) DNA has tow pyrmidine bases which contain single ring structure.

(II) In DNA, composition of bases should be 

(III) Each base pair of DNA is 3.4Å apart from othes base pair.

(IV) The nucleosome model DNA packaging was proposed by Kornberg and Thomas.

Choose the correct statements from given options.

(a)   I and III

(b)   I, II and III

(c)   II, III and IV

(d)   I, III and IV

Answer: (d)

43. The term ‘prebiotic’ soup’ for organic water containing mixture of simple organic compounds was given by

(a)   Richter

(b)   Haldane

(c)   Arrhenius

(d)   Miller

Answer: (b)

44. Number of Barr body which will found in case of Turner’s syndrome will be

(a)   1

(b)   2

(c)   0

(d)   Can’t be determine by given data

Answer: (c)

45. Which one of the following options is not a sexually transmitted disease?

(a)   AIDS

(b)   Hepatitis-B

(c)   Pertussis

(d)   Syphilis

Answer: (c)

46. The Sub-units of 80S ribosome will be

(a)   40S, 40S

(b)   60S, 40S

(c)   60S, 20S

(d)   55S, 25S

Answer: (b)

47. Pyloric sphincter guards the opening between

(a)   stomach and duodenum

(b)   cardia and fundus

(c)   oesophagus and stomach

(d)   fundus and pylorus

Answer: (a)

48. Vegetative propagation of Bryophyllus takes place through

(a)   bulbil

(b)   corms

(c)   leaf buds

(d)   eyes

Answer: (c)

49. Pappus helps in dispersal of pollen in

(a)   Asteraceae

(b)   Brassicaceae

(c)   Malvaceae

(d)   Solanaceae

Answer: (a)

50. Diphtheria is caused by

(a)   poison released by living bacterial cell into the host

(b)   poison released from dead bacterial cell into the host

(c)   poison released by virus into the host

(d)   excessive immune response by the body of host

Answer: (a)

51. Which one of the following microbes is the source for vitamin-C?

(a)   Pseudomonas sp.

(b)   Acetobacter sp.

(c)   Aspergillus sp.

(d)   Chlorella

Answer: (b)

52. The fact that DNA is a genetic material was established by the experiment of

(a)   Meselson and Stahl

(b)   Hershey and Chase

(c)   Avery, Macleod and McCarty

(d)   Rosalind Franklin and Kornberg

Answer: (b)

53. Match the following Columns.

(a)   A – 3; B – 2; C – 1; D – 4

(b)   A – 2; B – 4; C – 1; D – 3

(c)   A – 4; B – 3; C – 2; D – 1

(d)   A – 1; B – 3; C – 2; D – 4

Answer: (b)

54. Which one of the following hormones is released by posterior lobe of pituitary gland?

(a)   FSH

(b)   ADH

(c)   ACTH

(d)   MSH

Answer: (b)

55. During the transmission of nerve impulse through a nerve fibre, the potential on the innerside of the plasma membrane has which type of electric charge?

(a)   First negative then positive and again back to negative

(b)   First positive then negative and continue to be negative

(c)   First negative then positive and continue to be positive

(d)   First positive then negative and again back to positive

Answer: (a)

56. Uric acid is the excretory waste of

(a)   adult amphibians

(b)   birds

(c)   amphibians larvae

(d)   mammals

Answer: (b)

57. In case of pregnancy the heartbeat of embryo starts at

(a)   4th week

(b)   7th week

(c)   6th week

(d)   5th week

Answer: (d)

58. PPLO are smallest cell in the living world. The extend form of PPLO is

(a)   Pseudo Pneumonia Length Orge

(b)   Pseudo Plank Leg Organelle

(c)   Penumonia Plank Like Organism

(d)   Pleuro Penumonia Like Organism

Answer: (d)

59. Slime-mould belongs to

(a)   kingdom-Protista

(b)   kingdom-Monera

(c)   kingdom-Fungi

(d)   kingdom-Plantae

Answer: (a)

60. Which one of the following is not a flower?

(a)   Shoe-flower

(b)   Sunflower

(c)   Larkspur

(d)   Water lily

Answer: (b)

JIPMER MBBS Entrance Exam – 2016

English & Quantitative Reasoning

Directions (Q. Nos. 1-5) Fill up blanks in the passage given below with the most appropriate word from the options given for each blank.

The …….(1)….. age is the age of machines. From the ….(2)… the industrial Revolution began in Europe. Man’s life has been changing …(3)… many ways. At first the change was …(4)… Now machines have become …(5)… of our daily lives.

1.

(a)   modern

(b)   new

(c)   civilized

(d)   present

Answer: (a)

2.

(a)   birth

(b)   time

(c)   beginning

(d)   start

Answer: (b)

3.

(a)   into

(b)   to

(c)   in

(d)   with

Answer: (c)

4. 

(a)   slow

(b)   steady

(c)   fast

(d)   stagnant

Answer: (a)

5.

(a)   component

(b)   part

(c)   necessity

(d)   support

Answer: (b)

Directions (Q. Nos. 6-8) Read each sentence to find out whether there is any error in it. The error, if any, will be in one part of the sentence. The alphabet number of that part is the answer. If there is no error, then the answer is (e).

6. We (a) / saw (b)/ a elephant (c)/ In the zoo. (d)/ No error (e)

Answer: (c)

7. It is (a) / a most (b)/ beautiful (c)/ painting of the gallery. (d)/ No error (e)

Answer: (b)

8. Mr Gaurav Sharma (a)/ is (b)/ coming to (c)/ dinner. (d)/ No error (e)

Answer: (e)

Directions (Q. Nos. 9-10) Select the word which is most nearly the same in meaning as the given words.

9. DEMISE

(a)   Result

(b)   Default

(c)   Death

(d)   Apprehension

Answer: (c)

10. DISPARITY

(a)   Distaste

(b)   Dissimilarity

(c)   Criticism

(d)   Distinction

Answer: (b)

Directions (Q. Nos. 11-12) Find the missing number/letter from the given alternatives.

11. BMO, EOQ, HQS, ?

(a)   SOW

(b)   LMN

(c)   KSU

(d)   SOV

Answer: (c)

12. 4117, 5138, 6159, 71710, ?

(a)   71382

(b)   76599

(c)   81911

(d)   81798

Answer: (c)

Directions (Q. Nos. 13-14) In each of the following questions, select the missing number from the given alternatives.

13. 

(a)   8

(b)   3

(c)   6

(d)   36

Answer: (c)

14. 

(a)   60

(b)   39

(c)   55

(d)   65

Answer: (d)

15. If the Ist of November falls on Monday, what day will the 25th of November will be?

(a)   Tuesday

(b)   Thursday

(c)   Wednesday

(d)   Friday

Answer: (b)

16. Which one set of letters when sequentially placed at the gaps in given letter series shall complete it?

m_st_u_t_ _st

(a)   ummmu

(b)   umsmu

(c)   muumm

(d)   ttssuu

Answer: (b)

17. A and Bare standing at a distance of 20 km from each other on a straight East-West road. A and B start walking simultaneously Eastwards and Westwards respectively and both cover a distance of 5 km. Then, A turns to his left and walks 10 km. B turns to his right and walks 10 km at the same speed. What will be the distance between them?

(a)   10 km

(b)   30 km

(c)   20 km

(d)   25 km

Answer: (a)

18. There are three baskets of fruits. 1st basket has twice the number of fruits in the 2nd basket. 3rd basket has three-fourth of the fruits in the first. The average of the fruits in all the baskets is 30. What is the number of fruits in the first basket?

(a)   20

(b)   30

(c)   35

(d)   40

Answer: (d)

19. Find the word which cannot be formed from the letter used in the given word.

GERMINATION

(a)   ORNAMENT

(b)   TERMINAL

(c)   IGNITE

(d)   GERMAN

Answer: (b)

20. In a certain code, MODEL is written as 513# 2 and DEAR is written as 3 # % 8. How is LOAD written in that code?

(a)   23%1

(b)   21%3

(c)   25%3

(d)   21#3

Answer: (b)

JIPMER Medical Entrance Exam Previous Year Question Paper 2017 With Answer Key

JIPMER MBBS Entrance Question Paper – 2017

PHYSICS

1. Two 20 g flatworms climb over a very thin wall, 10 cm high. One of the worm is 20 cm long, the other is wider and only 10 cm long. Which of the following statement is correct regarding them?

(a)  20 cm worm has done more work against gravity

(b)  10 cm worm has done more work against gravity

(c)  Both worms have done equal work against gravity

(d)  Ratio of work done by both the worms is 4 : 5

Answer: (b)

2. A rocket is intended to leave the Earth’s gravitational field. The fuel in tis main engine is a little less than the amount that is necessary and an auxiliary engine, (only capable of operating for a short time) has to be used as well. When is it best to switch on the auxiliary engine?

(a)  at take-off

(b)  when the rocket has nearly stopped with respect to the Earth.

(c)  It doesn’t matter.

(d)  Can’t say

Answer: (a)

3. The turns of a solenoid, designed to provide a given magnetic flux density along its axis, are wound to fill the space between two concentric cylinders of fixed radii. How should the diameter d of the wire used be chosen so as to minimize the heat dissipated in the windings?

(a)  Wire should be multiple of 5d

(b)  Wire should be multiple of d/3

(c)  Wire is independent of d

(d)  Can’t say

Answer: (c)

4. A cylindrical tube of uniform cross-sectional area A is fitted with two air tight frictionless pistons. The pistons are connected to each other by a metallic wire. Initially, the pressure of the gas is p0 and temperature is {0, atmospheric pressure is also p0. Now, the temperature of the gas is increased to 2I0, the tension of wire will be

(a)  2ρ0A

(b)  ρ0A

(c)  ρ0A/2

(d)  4ρ0A

Answer: (b)

5. A particle of mass m is executing oscillation about the origin on X-axis. Its potential energy is V(x) = K|x|3. Where K is a positive constant. If the amplitude of oscillation is a, then its time period T is proportional to

(a)  1/√a

(b)  a

(c)  √a

(d)  a3/2

Answer: (a)

6. A body is projected vertically upwards. The times corresponding to height h while ascending and while descending are t­1 and t2, respectively.

Then, the velocity of projection will be (take g as acceleration due to gravity)

(a)   

(b)    

(c)   

(d)   

Answer: (b)

7. A long straight wire is carrying current I in +z direction. The x – y plane contains a closed circular loop carrying current I2 and not encircling the straight wire. The force on the loop will be

(a)  μ0l1l0/2π

(b)  μ0l1l0/4π

(c)  zero

(d)  depends on the distance of the centre of the loop from the wire

Answer: (d)

8. When the radioactive isotope 88Ra226 decays in a series by emission of three alpha (α) and a beta (β) particle, the isotope X which remains undecay is

(a)  83X214

(b)  84X218

(c)  84X220

(d)  87X223

Answer: (a)

9. A solid cylinder is attached to a horizontal massless spring as shown in figure. If the cylinder rolls without slipping, the time period of oscillation of the cylinder is

(a)   

(b)   

(c)   

(d)   

Answer: (c)

10. N lamps each of resistance r, are fed by a machine of resistance R. If light emitted by any lamp is proportional to the square of the heat produced, prove that the most efficient way of arranging them is to place them in parallel arcs, each containing n lamps, where n is the integer nearest to

(a)   

(b) 

(c)  (NRr)3/2

(d)  (NRr)1/2

Answer: (b)

11. Radioactive decay will occur as follows

If a certain mass of radon (Rn = 220) is allowed to decay in a certain container; then after 5 minutes the element with the greater mass will be

(a)  radon

(b)  polonium

(c)  lead

(d)  bismuth

Answer: (c)

12. A stream of a liquid of density ρ flowing horizontally with speed v rushes out of a tube of radius r and hits a vertical wall nearly normally. Assuming that the liquid does not rebound from the wall, the force exerted on the wall by the impact of the liquid is given by

(a)  πrρv

(b)  πrρv2

(c)  πr2ρv

(d)  πr2ρv2

Answer: (d)

13. The x and y coordinates of a particle moving in a plane are given by x(t) = a cos (pt) and y(t) = b sin (pt) where a, b (<a) and pare positive constants of appropriate dimensions and t is time. Then, which of the following is not true?

(a)  The path  of the particle is an ellipse.

(b)  Velocity and acceleration of the particle are perpendicular to each other at  t = π/2ρ

(c)  Acceleration of the particle is always directed towards a fixed point.

(d)  Distance travelled by the particle in time interval between t = 0 and t = π/2P is a

Answer: (d)

14. White light is used to illuminate two slits in Young’s double slit experiment. The separation between the slits is b and the screen is at a distance d (>>b) from the slits. At a point on the screen directly in front of one of the slits, which wavelengths are missing?

(a)   

(b)   

(c)   

(d)   

Answer: (c)

15. A skier starts from rest at point A and slides down the hill without turning or breaking. The friction coefficient is μ. When he stops at point B, his horizontal displacement is S. What is the height difference between points A and B?

(The velocity of the skier is small so that the additional pressure on the snow due to the curvature can be neglected. Neglect also the friction of air and the dependence of μ on the velocity of the skier.)

(a)  h = μS

(b)  h = μ/S

(c)  h = 2μS

(d)  h = μS2

Answer: (a)

16. A bicycle wheel rolls without slipping on a horizontal floor. Which one of the following is true about the motion of points on the rim of the wheel, relative to the axis at the wheel’s centre?

(a)  Points near the top move faster than points near the bottom

(b)  Points near the  bottom move faster than points near the top

(c)  All points on the rim move with the same speed

(d)  All points have the velocity vectors that are pointing in the radial direction towards the centre of the wheel

Answer: (a)

17. The planets with radii R1 and R2 have densities ρ1, ρ2 Their atmospheric pressures are p1 and p2 respectively. Therefore, the ratio of masses of their atmosphere, neglecting variation of g within the limits of atmosphere is

(a)  ρ1R2p1/ ρ2R1p2

(b)  p1R2ρ2/pP2R1ρ1

(c)  p1R1ρ1/p2R2ρ2

(d)  p1R1ρ2/p2R2ρ1

Answer: (d)

18. A thin symmetrical double convex lens of refractive index μ2 = 1.5 is placed between a medium of refractive index μ1 = 1.4 to the left and another medium of refractive index μ3 = 1.6 to the right. Then, the system behaves as

(a)  a convex lens

(b)  a concave lens

(c)  a glass plate

(d)  a convexo concave lens

Answer: (c)

19. A wide hose pipe is held horizontally by a fireman. It delivers water through a nozzle at one litre per second. On increasing the pressure, this increases to two litres per second. The fireman has now to

(a)  push forward twice as hard

(b)  push forward four times as hard

(c)  push backward four times as hard

(d)  push backward twice as hard.

Answer: (b)

20. The wavelength λ of a photon and the de-Broglie wavelength of an electron have the same value. Find the ratio of energy of photon to the kinetic energy of electron in terms of mass m, speed of light c and planck constant.

(a)   

(b)   

(c)   

(d)   

Answer: (d)

21. A non-conducting ring of radius 0.5 m carries a total charge of 1.11 × 1010 C distributed non-uniformly on its circumference producing on its circumference on electric field E, everywhere in space.

The value of line integral   (l = 0 being centre of ring) in volts is

(a)  +2

(b)  −1

(c)  −2

(d)  Zero

Answer: (a)

22. The upper half of an inclined p lane of inclination θ is perfectly smooth while the lower half rough. A block starting from rest at the top of the plane will again come to rest at the bottom if the coefficient of friction between the block and the lower half of the plane is given  by

(a)  μ = 2 tanθ

(b)  μ = tanθ

(c)  μ = 2 /(tanθ)

(d)  μ = 1/ tanθ

Answer: (a)

23. Two masses 10 kg and 20 kg respectively are connected by a massless spring as shown in figure. A force of 200 N acts on the 20 kg mass. At the instant shown is figure the 10 kg mass has acceleration of 12 m/s2. The value of acceleration of 20 kg mass is

(a)  4 m/s2

(b)  10 m/s2

(c)  20 m/s2

(d)  30 m/s2

Answer: (a)

24. A cylinder rolls up an inclined plane, reaches some height and then rolls down (without slipping throughout these motions). The directions of the frictional force acting on the cylinder are

(a)  up the incline while ascending and down the incline while descending.

(b)  up the inline while ascending as well as descending.

(c)  down the incline while ascending and up the incline while descending.

(d)  down the incline while ascending as well as descending.

Answer: (b)

25. A liquid is allowed into a tube of truncated cone shape. Identify the correct statement from the following.

(a)  The speed is high at the wider end and low at the narrow end.

(b)  The speed is low at the wider end and high at the narrow end.

(c)  The speed is same at both ends in an streamline flow.

(d)  The liquid flows with uniform velocity in the tube.

Answer: (b)

26. Two soap bubble coalesce. It is noticed that, whilst joined together, the radii of the two bubbles are a and b where a > b. Then the radius of curvature of interface between the two bubbles will be

(a)  a – b

(b)  a + b

(c)  ab/(a – b)

(d)  ab / (a + b)

Answer: (c)

27. The displacement of a particle along the x-axis is given by x = a sin2 ω The motion of the particle corresponds to

(a)  simple harmonic motion of frequency ω/π

(b)  simple harmonic motion of frequency 3ω/2π

(c)  non simple harmonic motion

(d)  simple harmonic motion of frequency ω/2π

Answer: (c)

28. Mercury boils at 367° However, mercury thermometers are made such that they can measure temperature upto 500°C. This is done by

(a)  maintaining vacuum aboves mercury column in the stem of the thermometer

(b)  filling nitrogen gas at high pressure above the mercury column.

(c)  filling oxygen gas at high pressure above the mercury column.

(d)  filling nitrogen gas at low pressure above the mercury column.

Answer: (b)

29. Two identical glass spheres filled with air are connected by a horizontal glass tube. The glass tube contains a pellet of mercury at its mid-points. Air in one sphere is at 0°C and the other is at 20° If both the vessels are heated through 10°C, then neglecting the expansions of the bulbs and the tube

(a)  the mercury pellet gets displaced towards the sphere at lower temperature.

(b)  the mercury pellet gets displaced towards the sphere at higher temperature.

(c)  the mercury pellet does not get displaced at all

(d)  the temperature rise causes the pellet to expand without any displacement.

Answer: (c)

30. A nucleus  has mass represented by m(A, Z). If mp and mn denote the mass of proton and neutron respectively and BE the binding energy (in MeV) then,

(a)  BE = [m(A, Z) – Zmp – (A – Z)mn]C2

(b)  BE = [Zmp + (A – Z)mn – m(A, Z)]C2

(c)  BE = [Zmp + Amn – m(A, Z)] C2

(d)  BE = m(A, Z) – Zmp – (A – Z) mN

Answer: (b)

31. A graph between pressure P(along y-axis) and absolute temperature, T(along x-axis) for equal moles of two gases has been drawn. Given that volume of second gas is more than volume of first gas. Which of the following statement is correct?

(a)  Slope of gas 1 is less than gas 2

(b)  Slope of gas 1 is more than gas 2

(c)  Both have some slopes

(d)  None of the above

Answer: (b)

32. A piece of blue glass heated to a high temperature and a piece of red glass at room temperature are taken inside a dimly-lit room. Then,

(a)  the blue piece will look blue and the red piece will look red as usual.

(b)  the red piece will look brighter red and the blue piece will look ordinary blue.

(c)  the blue will look brighter as compared to the red piece.

(d)  both the pieces will look equal red.

Answer: (c)

33. A certain charge Q is divided into two parts q and Q – q. How the charge Q and q must be related so that when q and (Q – q) is placed at a certain distance apart experience maximum electrostatic repulsion?

(a)  Q = 2q

(b)  Q = 3q

(c)  Q = 4q

(d)  Q = 4q + c

Answer: (a)

34. A charged particle ‘q’ is shot with speed v towards another fixed charged particle Q. It approaches Q upto a closest distance r and then returns. If q were given a speed 2v, the closest distance of approach would be

(a)  r

(b)  2r

(c)  r/2

(d)  r/4

Answer: (d)

35. A long block A of mass M is at rest on a smooth horizontal surface. A small block B of mass M/2 is placed on A at one end and projected along A with some velocity v. The coefficient of friction between the block is μ. Then, the accelerations of blocks A and B before reaching a common velocity will be respectively

(a)  μg/2, (towards right), μg/2(towards left)

(b)  μg(towards right), μg(towards left)

(c)  μg/2(towards right), μg(towards left)

(d)  μg(toward right), μg/2(towards left)

Answer: (c)

36. A beam of light composed of red and green rays is incident obliquely at a point on the face of a rectangular glass slab. When coming out on the opposite parallel face, the red and green rays emerge from

(a)  two points propagating in two different non-parallel directions.

(b)  two points propagating in two different parallel directions.

(c)  one point propagating in two different directions.

(d)  one point propagating in the same direction.

Answer: (b)

37. The plane face of a plano convex lens is silvered. If μ be the refractive index and R, the radius of curvature of curved surface, then system will behave like a concave mirror of curvature

(a)  μR

(b)  R2/ μ

(c)  R / (μ – 1)

(d)  [(μ + 1)/( μ – 1)]R

Answer: (c)

38. The maximum numbers of possible interference maxima for slit separation equal to twice the wavelength in Young’s double slit experiment is

(a)  infinite

(b)  five

(c)  three

(d)  zero

Answer: (b)

39. An isotropic point source of light is suspended h metre vertically above the centre of circular table of radius r metre. Then, the ratio of illumenances at the centre to that at the edge of the table is

(a)   

(b)   

(c)   

(d)   

Answer: (c)

40. In the given figure, what is the magnetic field induction at point O.

(a)   

(b)   

(c)   

(d)   

Answer: (c)

41. p-V plots for two gases during adiabatic process as shown in figure plots 1 and 2 should correspond respectively to

(a)  He and O2

(b)  O2 and He

(c)  He and Ar

(d)  O2 and N2

Answer: (b)

42. The half-life period of a radioactive element X is same as the mean life of another radioactive element Y. Initially, both of them have the same numbers of atoms then,

(a)  X and Y have the same decay rate initially.

(b)  X and Y decay at the same rate always

(c)  Y will decay at a faster rate than X

(d)  X will decay at a faster rate than Y

Answer: (c)

43. A source emits electromagnetic waves of wavelength 3m. One beam reaches the observer directly and other after reflection from a water surface, travelling 1.5m extra distance and with intensity reduced to 1/4 as compared to intensity due to the direct beam alone. The resultant intensity will be

(a)  (1/4) fold

(b)  (3/4) fold

(c)  (5/4) fold

(d)  (9/4) fold

Answer: (d)

44. The following circuit represents

(a)  OR gate

(b)  XOR gate

(c)  AND gate

(d)  NAND gate

Answer: (b)

45. Two identical conducting balls A and B have positive charges q1 and q2 But q1 ≠ q2/ The balls are brought together so that they touch each other and then kept in their original positions. The force between them is

(a)  less than that before the balls touched

(b)  greater than that before the balls touched

(c)  same as that before the balls touched

(d)  zero

Answer: (b)

46. A positively charged ball hangs from a silk thread. We put a positive test charges q0 at a point and measure F/q0, then it can be predicted that the electric field strength E

(a)  > F/q0

(b)  = F/q

(c)  < F/q0

(d)  cannot be estimated

Answer: (a)

47. Capacitor C1 of capacitance 1 μF and capacitor C2 of capacitance 2 μF are separately charged fully by a common battery. The two capacitors are then separately allowed to discharged through equal resistors at time t = 0

(a)  the current in each of the two discharging circuits is zero at t = 0

(b)  the currents in the two discharging circuits at t = 0 are equal but non-zero

(c)  the currents in the two discharging circuits at t = 0

(d)  Capacitor C1 loses 40% of its initial charge sooner than C2 loses 40% of initial charge.

Answer: (b)

48. A uniform electric field and a uniform magnetic field acting along the same direction in a certain region. If an electron is projected along the direction of the fields with a certain velocity, then

(a)  it will turn towards left of direction of motion.

(b)  it will turn towards right of direction of motion.

(c)  its velocity will increase.

(d)  its velocity will decrease.

Answer: (d)

49. To reduce the range of voltmeter, its resistance need to be reduced. A voltmeter has resistance R0 and range V. Which of the following resistances when connected in parallel will convert it into a voltmeter of range V/n?

(a)  nR0

(b)  (n + 1)R0

(c)  (n – 1)R

(d)  None of these

Answer: (d)

50. A uniform rod of length l is free to rotate in a vertical plane about a fixed horizontal axis through B. The rod begins rotating from rest from its unstable equilibrium position. When, it has turned through an angle θ, its angular velocity ω is given by

(a)    

(b)   

(c)   

(d)   

Answer: (a)

51. A stick of length L and mass M lies on a frictionless horizontal surface on which it is free to move in any way. A ball of mass m moving with speed V collides elastically with the stick as shown in fig below. If after the collision, the ball comes to rest, then what should be the mass of the ball?

(a)  m = 2M

(b)  m = M

(c)  m = M/2

(d)  m = M/4

Answer: (d)

52. The mass of a proton is 1847 times that of an electron. A electron and a proton are injected into a uniform electric field at right angle to the direction of the filed with the same initial K.E.

(a)  the electron trajectory will be less curved than the proton trajectory.

(b)  both the trajectories will be straight.

(c)  the proton trajectory will be less curved than the electron trajectory.

(d)  both the trajectories will be equally curved.

Answer: (d)

53. Two condensers, one of capacity C and the other of capacity C/2, are connected to a V-volt battery, as shown.

The work done in charging fully both the condensers is

(a)  CV2

(b)    

(c)   

(d)   

Answer: (c)

54. A capacitor of capacitance 5μF is connected as shown in the figure. The internal resistance of the cell is 0.5Ω. The amount of charge on the capacitor plates is

(a)  80 μC

(b)  40 μC

(c)  20 μC

(d)  10 μC

Answer: (d)

55. A photo cell is illuminated by a small bright source placed 1m away. When the same source of light is placed 2m away, the electrons emitted by photo cathode

(a)  carry one quarter of their previous energy

(b)  carry one quarter of their previous momenta

(c)  are half as numerous

(d)  are one quarter as numerous

Answer: (d)

56. ABC is right angled triangular plane of uniform thickness. The sides are such that AB > BC as shown in figure. I1, I2, I3 are momenta of inertia about AB, BC and AC, respectively. Then which of the following relations is correct?

(a)  I1 = I2 = I3

(b)  I2 > I1 > I3

(c)  I3 < I2 < I1

(d)  I3 > I1 > I2

Answer: (b)

57. An ice-berg of density 900 kgm3 is floating in water of density 1000 kgm3. The percentage of volume of ice-berg outside the water is

(a)  20%

(b)  35%

(c)  10%

(d)  11%

Answer: (c)

58. The potential of an atom is given by V = V0 loge(r/r0) where r0 is a constant and r is the radius of the orbit. Assuming Bohr’s model to be applicable, which variation of rn with n is possible (n being principal quantum number)?

(a)  rn ∝ n

(b)  rn ∝ 1/n

(c)  rn ∝ n2

(d)  rn ∝ 1/n2

Answer: (a)

59. The temperature of source and sink of a heat engine are 127°C and 27°C, respectively. An inventor claims its efficiency to be 26%, then

(a)  it is impossible

(b)  it is possible with high probability        

(c)  it is possible with low probability

(d)  Data are insufficient

Answer: (a)

60. You are given resistance wire of length 50 cm and a battery of negligible resistance. In which of the following cases is largest amount of heat generated?

(a)  When the wire is connected to the battery directly

(b)  When the wire is divided into two parts and both the parts are connected to the battery in parallel.

(c)  When the wire is divided into four parts and all the four parts are connected to the battery in parallel.

(d)  When only half of the wire is connected to the battery.

Answer: (c)

JIPMER MBBS Entrance Question Paper – 2017

CHEMISTRY

61. The compressibility of a gas is less than unity at STP, therefore

(a)  Vm > 22.4 L

(b)  Vm­ < 22.4 L

(c)  Vm = 22.4 L

(d)  Vm = 44.8 L

Answer: (b)

62. Among the following set of quantum numbers, the impossible set is

(a)  n : 3; l : 2; m : 3; s : −1/2

(b)  n : 4; l : 0; m : 0; s : 1/2

(c)  n : 5; l : 3; m : 0; s : −1/2

(d)  n : 3; l : 2; m : −2; s : 1/2

Answer: (a)

63. 

In the above reaction, end product ‘C’ is

(a)  salicylaldehyde

(b)  salicylic acid

(c)  phenyl acetate

(d)  aspirin

Answer: (d)

64. The normality of 10% (w/v) of acetic acid is

(a)  1 N

(b)  1.3 N

(c)  1.7 N

(d)  1.9 N

Answer: (c)

65. Which of the following is least soluble in water?

(a)  C2H6

(b)  CH3OH

(c)  CH3NH2

(d)  C6H5OH

Answer: (a)

66. The compound (A) is

(a)  C2H5Cl

(b)  C2H5ONa

(c)  CH2N2

(d)  CH3ONa

Answer: (b)

67. 

The product (P) is

(a) 

(b) 

(c) 

(d) 

Answer: (a)

68. 1-butyne on oxidation with hot alkaline KMnO­4 would yield. Which of the following as end product?

(a)  CH3CH2CH2COOH

(b)  CH3CH2COOH

(c)  CH3CH2COOH + CO2 + H2O

(d)  CH3CH2COOH + HCOOH

Answer: (c)

69. A cubic unit cell of a metal with molar mass of 63.55 g mol1 has an edge length of 362 pm. Its density is 8.92 g cm3. The type of unit cell is

(a)  primitive

(b)  face centred

(c)  end centred

(d)  body centred

Answer: (b)

70. Which of the following is the major product in the reaction of HOBr with propene?

(a)  2-bromo, 1-propanol

(b)  3-bromo, 1-propanol

(c)  2-bromo, 2-propanol

(d)  1-bromo, 2-propanol

Answer: (d)

71. Which of the following is the correct IUPAC name?

(a)  3, 4-dimethyl pentanoyl chloride

(b)  1-chloro-1-oxo-2, 3-dimethyl pentane

(c)  2-ethyl-3-methyl butanoyl chloride

(d)  2, 3-dimethyl pentanoyl chloride

Answer: (d)

72. Consider the following solutions,

A = 0.1 glucose, B = 0.05 M NaCl,

C = 0.05 M BaCl2, D = 0.1 M AlCl­3

Which of the following pairs is isotonic?

(a)  A and B

(b)  A and D

(c)  A and C

(d)  B and C

Answer: (a)

73. Which of the following compound is not coloured?

(a)  Na2CuCl

(b)  Na2Cd ∙ Cl4

(c)  FeSO4

(d)  Vl3

Answer: (b)

74. Which of the following oxide is most acidic?

(a)  As2O3

(b)  P2O5

(c)  Sb2O3

(d)  Bi2O3

Answer: (b)

75. When CO2 is bubbled through a solution of barium peroxide in water then

(a)  carbonic acid is formed

(b)  O2 is released

(c)  H2O2 is formed

(d)  no reaction occurs

Answer: (c)

76. The correct increasing order of ionic radii of the following Ce3+, La3+, Pm3+ and Yb3+ is

(a)  Yb3+ < Pm3+ < Ce3+ < La3+

(b)  Ce3+ < Yb3+ < Pm3+ < Ce3+

(c)  Yb3+ < Pm3+ < La3+ < Ce3+

(d)  Pm3+ < La3+ < Ce3+ < Yb3+

Answer: (a)

77. The shape of gaseous SnCl2 is

(a)  tetrahedral

(b)  linear

(c)  angular

(d)  T-shape

Answer: (c)

78. Which of the following aqueous solution should have highest boiling point?

(a)  1.0 M NaOH

(b)  1.0 M Na2SO4

(c)  1.0 M NH4NO3

(d)  1.0 MKNO3

Answer: (b)

79. When a lead storage battery is discharged;

(a)  SO2 is evolved

(b)  lead sulphate is consumed

(c)  lead is formed

(d)  sulphuric acid is consumed

Answer: (d)

80. Combustion of glucose takes place according to the equation,

C6H12O6 + 6O2 → 6CO2 + 6H2O, ∆H = −72 k-cal

The energy required for combustion of 1.6 g of glucose is

(a)  0.064 k-cal

(b)  0.64 k-cal

(c)  6.4 k-cal

(d)  64 k-cal

Answer: (b)

81. When the heat of reaction at constant pressure is −5 × 103 cal and entropy change is 7.4 cal deg1 at 25°C, the reaction is predicted as

(a)  reversible

(b)  spontaneous

(c)  non-spontaneous

(d)  irreversible

Answer: (b)

82. Which of the following is not expected to show paramagnetism?

(a)  [Ni(H2O)]2+

(b)  Ni(CO)4

(c)  [Ni(NH3)4]2+

(d)  [Co(NH3)6]2+

Answer: (b)

83. In which of the following compounds, sulphur show maximum oxidation number?

(a)  H2SO4

(b)  SO3

(c)  H2S2O7

(d)  All have same oxidation number for sulphur

Answer: (d)

84. For the chemical reaction, 2O3 ⇌ 3O2

The reaction proceed as follows

O3 ⇌ O2 + O (fast)

O + O3 → 2O2 (slow)

The rate law expression will be

(a)  r = k’[O3]2

(b)  r = k’[O3]2[O2]1

(c)  r = k’[O3][O2]

(d)  Unpredictable

Answer: (b)

85. Which of the following is the correct order of stability for the given superoxides?

(a)  KO2 > RbO2 < CsO2

(b)  CsO2 < RbO2 < KO2

(c)  RbO2 < CsO2 < KO2

(d)  KO2 < CsO2 < RbO2

Answer: (a)

86. Among the following compounds, which will produce POCl3 with PCl5.

(a)  Only O2

(b)  O2 and CO2

(c)  CO2, O2 and P4O10

(d)  SO2, H2­O, H2SO4 and P4O10

Answer: (d)

87. In the following graph.

The slope of line AB gives the information of

(a)  

(b)   

(c)   

(d)   

Answer: (c)

88. Among the following compounds, which compound is polar as well as exhibit sp2-hyrbidization by the central atom

(a)  H2CO3

(b)  SiF4

(c)  BF3

(d)  HClO3

Answer: (a)

89. Which one among the following is added to soap to impart antiseptic property?

(a)  Sodium lauryl sulphate

(b)  Sodium dodecyl benzene sulphonates

(c)  Rosin

(d)  Bithional

Answer: (d)

90. How many Faradays are required to reduce 1 mol of 

(a)  3

(b)  5

(c)  6

(d)  4

Answer: (c)

91. The energy released when 6 moles of octane is burnt in air will be [Given, ∆Hf for CO2(g), H2O(g) and C8H8(l), respectively are −490, −240 and +160 J/mol]

(a)  −37.4 kJ

(b)  −20 kJ

(c)  −6.2 kJ

(d)  −35.5 kJ

Answer: (d)

92. When I is oxidized by  in alkaline medium, I converts into

(a)    

(b)  I2

(c)   

(d)  IO

Answer: (a)

93. The value of reaction quotient (Q), for the following cell

Zn(s) |Zn2+(0.01 M)| | Ag+ (1.25 M) | Ag (s) is

(a)  156

(b)  125         

(c)  1.25 × 102

(d)  6.4 × 103

Answer: (d)

94. When 750 mL of 0.5 M HCl is mixed with 250 mL of 2M NaOH solution, the value of pH will be

(a)  pH = 7

(b)  pH > 7

(c)  pH < 7

(d)  pH = 0

Answer: (b)

95. The most stable carbonium ion among the following is

(a)   

(b)   

(c)   

(d)   

Answer: (c)

96. Which of the following is not applicable to the phenomenon of adsorption?

(a)  ∆H > 0

(b)  ∆G < 0

(c)  ∆S < 0

(d)  ∆H < 0

Answer: (d)

97. In cyamide extraction process of silver from argentite ore, the oxidizing and reducing agents are respectively.

(a)  O2 and CO2

(b)  O2 and Zn dust

(c)  HNO3 and Zn dust

(d)  HNO3 and CO

Answer: (b)

98. The product of acid hydrolysis of (P) and (Q) can be distinguished by

(a)  Lucas reagent

(b)  2, 4-DNP

(c)  Fehling’s solution

(d)  NaHSO3

Answer: (c)

99. Clemmensen reaction of ketone is carried out in the presence of

(a)  LiAlH4

(b)  Zn-Hg with HCl

(c)  glycol with KOH

(d)  H2 with Pt as catalyst

Answer: (b)

100. The standard reduction potential for Zn2+/Zn, Ni2+/Ni and Fe2+/Fe are −76, −0.23 and −0.44 V, respectively.

The reaction X + Y2+ → X2+ + Y will be spontaneous when

(a)  X = Ni, Y = Fe

(b)  X = Ni, Y = Zn

(c)  X = Fe, Y = Zn

(d)  X = Zn, Y = Ni

Answer: (d)

101. Cannizzaro reaction is not shown by

(a) 

(b) 

(c)  CH3CHO

(d)  HCHO

Answer: (c)

102. The main product formed in the following reaction is

(a) 

(b)  

(c)  

(d) 

Answer: (a)

103. The correct order of basic strength of the following are

(a)  I > II > III > IV

(b)  IV > II > III > I

(c)  III > IV > II > I

(d)  III > II > IV > I

Answer: (d)

104. In the reaction,

In the given reaction (B) is

(a)

(b)  

(c) 

(d)  

Answer: (a)

105. Polymer formation from monomers starts by

(a)  condensation reaction between monomers

(b)  coordination reaction between monomers

(c)  conversion of one monomer into other monomer

(d)  hydrolysis of monomers

Answer: (a)

106. Match the type of series given in Column I with the wavelength range given in Column II and choose the correct option.

(a)  A – 1; B – 2; C – 4; D – 3

(b)  A – 4; B – 3; C – 1; D – 2

(c)  A – 3; B – 1; C – 2; D – 4

(d)  A – 4; B – 3; C – 2; D – 1

Answer: (a)

107. Which of the following coordination compounds would exhibit optical isomerism?

(a)  Pentamminenitrocobalt (III) iodide

(b)  Tris-(ethylenediamine) cobalt (III) bromide

(c)  Trans-dicyanobis (ethylenediamine)

(d)  Diamminedinitroplatisum (II)

Answer: (b)

108. The electrons identified by quantum numbers n and l, are as follows

(I) n = 4, l = 1

(II) n = 4, l = 0

(III) n = 3, l = 2

(IV) n = 3, l = 1

If we arrange them in order of increasing energy, i.e. from lowest to highest, the correct order is

(a)  IV < II < III < I

(b)  II < IV < I < III

(c)  I < III < II < IV

(d)  III < I < IV < II

Answer: (a)

109. On hydrolysis of starch, we finally get

(a)  glucose

(b)  fructose

(c)  Both (a) and (b)

(d)  sucrose

Answer: (a)

110. Which of the following sodium compound/compound(s) are formed when an organic compound containing both nitrogen and sulphur is fused with sodium?

(a)  Cyanide and sulphide

(b)  Thiocyanate

(c)  Sulphite and cyanide

(d)  Nitrate and sulphide

Answer: (b)

111. Which of the following region is coldest?

(a)  Stratosphere

(b)  Troposphere

(c)  Mesosphere

(d)  Thermosphere

Answer: (c)

112. A solid AB has NaCl structure. If the radius of cation A+ is 170 pm, then the maximum possible radius of the anion B is

(a)  397.4 pm

(b)  347.9 pm

(c)  210.9 pm

(d)  410.6 pm

Answer: (d)

113. A first order reaction is 50% completed in 1.26 × 1014 How much time would it takes for 100% completion?

(a)  1.26 × 1015 s

(b)  2.52 × 1014 s

(c)  2.52 × 1028 s

(d)  Infinite

Answer: (d)

114. Flux is used to

(a)  remove all type of impurities

(b)  reduce metal oxide

(c)  remove carbonate and sulphate

(d)  remove silica and undesirable metal oxides

Answer: (d)

115. Sulphur reacts with chlorine in 1 : 2 ratio and forms X. Hydrolysis of X gives a sulphur compound Y. The hybridization of central atom in the anion Y is

(a)  sp3

(b)  sp2

(c)  sp3d

(d)  sp

Answer: (a)

116. Aqueous 10% NaHCO3 solution is used as a reagent for identifying ‘A’. Which of the following compounds yield ‘A’ on hydrolysis?

(a)  CH3COOC2H5

(b)  C2H5 – COO – C2H5

(c)  CH3CHO

(d)  CH3CH2OH

Answer: (a)

117. Which of the following fibres is made of polyamides?

(a)  Dacron

(b)  Orion

(c)  Nylon

(d)  Rayon

Answer: (c)

118. Consider the following reaction,

X is

(a)  benzoic acid

(b)  salicylic acid

(c)  phenol

(d)  aniline

Answer: (d)

119. The volume of water to be added to 100 cm3 of 0.5 N H2SO4 to get decinormal concentration is

(a)  100 cm3

(b)  450 cm3

(c)  500 cm3

(d)  400 cm3

Answer: (d)

120. In which of the following the oxidation number of oxygen has been arranged in increasing order?

(a)  BaO2 < KO2 < O3 < OF2

(b)  OF2 < KO2 < BaO2 < O3

(c)  BaO2 < O3 < OF2 < KO2

(d)  KO2 < OF2 < O3 < BaO2

Answer: (a)

JIPMER MBBS Entrance Question Paper – 2017

BIOLOGY

121. One of the breeding techniques useful to eliminate harmful recessive genes by selection is

(a)  artificial insemination

(b)  outbreeding

(c)  inbreeding

(d)  MOET

Answer: (c)

122. Herbicide that blocks electron transport from PS-II to PS-I by inhibiting electron flow between plastoquinone → cytochrome is

(a)  DCMU

(b)  Paraquat

(c)  DCPIP

(d)  None of these

Answer: (a)

123. Statement I Microtubules are formed only in animals cells.

Statement II Microtubules are made up of a protein called myosin.

Choose the correct option

(a)  Statement I is correct and statement II is incorrect.

(b)  Statement II is correct and statement I is incorrect.

(c)  Both statements are correct.

(d)  Both statements are incorrect.

Answer: (d)

124. Some functions of nutrient element are given below

(I) Important constituent of proteins involved in ETS.

(II) Activator of catalase.

(III) Important constituent of cytochrome..

(IV) Essential for chlorophyll synthesis.

The concerned nutrient is

(a)  Cu

(b)  Fe

(c)  Ca

(d)  Mo

Answer: (b)

125. Torsion of visceral mass is seen in animals belonging to class

(a)  Cephalopoda

(b)  Scaphopoda

(c)  Amphineura

(d)  Gastropoda

Answer: (d)

126. A plant is provided with ideal conditions for photosynthesis and supplied with isotope 14 CO2. When the products of the process are analyzed carefully, what would be the nature of products?

(a)  Glucose and oxygen are labelled

(b)  Oxygen is labelled, but glucose is normal

(c)  Glucose and oxygen are normal

(d)  Glucose is labelled, but oxygen is normal

Answer: (d)

127. Match the following columns,

(a)  A – 4; B – 1; C – 3; D – 2

(b)  A – 5; B – 3; C – 1; D – 2

(c)  A – 5; B – 4; C – 1; D – 2

(d)  A – 2; B – 5; C – 3; D – 1

Answer: (c)

128. Pick the hormone which is not secreted by human placenta.

(a)  hCG

(b)  hPL

(c)  Prolactin

(d)  Oestrogen

Answer: (c)

129. Fixation of one CO2 molecule through Calvin cycle requires.

(a)  1 ATP and 2NADPH2

(b)  2 ATP and 2NADPH2

(c)  3ATP and 2NADH2

(d)  2ATP and 1NADPH2

Answer: (c)

130. Oxygen dissociation curve of haemoglobin is

(a)  sigmoid

(b)  hyperbolic

(c)  linear

(d)  hypobolic

Answer: (a)

131. A hormone, secreted by the endocrinal cells of duodenal mucosa which influences the release of pancreatic juice is

(a)  relaxin

(b)  cholecystokinin

(c)  secretin

(d)  progesterone

Answer: (b)

132. Cotyledons and testa are edible parts of

(a)  ground nut and pomegranate

(b)  walnut and tamarind

(c)  french bean and coconut

(d)  cashew nut and litchi

Answer: (a)

133. Intrinsic and extrinsic pathways of blood clotting are interlinked at the activation steps of which of the following factors?

(a)  Factor IX

(b)  Factor IV

(c)  Factor X

(d)  Factor XIII-a

Answer: (c)

134. Match the storage products listed under column I with the organism given under column II, choose the appropriate option from the given options

(a)  A – 3; B – 4; C – 1; D – 5

(b)  A – 4; B – 3; C – 5; D – 2

(c)  A – 5; B – 4; C – 1; D – 3

(d)  A – 2; B – 1; C – 4; D – 3

Answer: (c)

135. With respect to angiosperms, identify the incorrect pair from the following

(a)  antipodal-2n

(b)  vegetative all of male gametophyte – n

(c)  primary endosperm nucleus – 3n

(d)  cell of nucellus of ovule – 2n

Answer: (a)

136. The globular head of myosin contains

(a)  calcium ions in large quantities

(b)  troponin

(c)  ATPase enzyme

(d)  ATP

Answer: (c)

137. Parbhani Kranti, a variety of bhindi (lady finger) is resistant to

(a)  bacterial blight

(b)  yellow mosaic virus

(c)  black rot

(d)  leaf curl

Answer: (b)

138. Gastrula is the embryonic stage in which

(a)  cleavage occurs

(b)  blastocoels forms

(c)  germinal layers form

(d)  villi form

Answer: (c)

139. Dense regular connective tissue is present in

(a)  ligament and tendons

(b)  joint capsule and Wharton’s Jelly

(c)  periosteum and endosteum

(d)  pericardium and heart valves

Answer: (a)

140. Minisatellites or VNTR’s are used in

(a)  DNA fingerprinting

(b)  Polymerase Chain Reaction, (PCR)

(c)  gene therapy

(d)  gene mapping

Answer: (a)

141. Note the following features and choose the ones applicable to Wuchereria bancrofti.

(I) Coelozoic parasite

(II) Histozoic parasite

(III) Monogenetic parasite

(IV) Digenetic parasite

(V) Monomorphic, acoelomate parasite

(VI) Dimorphic, pseudocoelomate paraiste

(a)  II, III, V

(b)  II, III, VI

(c)  II, IV, VI

(d)  I, III, VI

Answer: (c)

142. hnRNA undergoes two additional process. Out of them in one process an unusual nucleotide (methyl GPT) is added to the 5’ end of What would you called this?

(a)  Tailing

(b)  Splicing

(c)  Termination

(d)  Capping

Answer: (d)

143. Which type of immunoglobin is/are abundantly found in foetus?

(a)  IgE

(b)  IgG

(c)  IgM

(d)  IgD

Answer: (b)

144. IUCN stands for

(a)  Indian Union for Conservation of Nature

(b)  International Union for Conservation of Nature

(c)  International Union for Chemical Nomenclature

(d)  International Union for Conservation for Nutrients

Answer: (b)

145. The secretory phase in the human menstrual cycle is also called as

(a)  luteal phase and last for about 6 days

(b)  follicular phase lasting for about 6 days

(c)  luteal phase and last for about 13 days

(d)  follicular phase and last for about 13 days

Answer: (c)

146. Which one of the following statements is correct?

(a)  Hard outer layer of pollen is called intine

(b)  Sporogenous tissue is haploid

(c)  Endothelium produces the microspores

(d)  Tapetum nourishes the developing pollen

Answer: (d)

147. Variation in gene frequencies within population can occur by chance rather than by natural selection. This is reffered to as

(a)  genetic flow

(b)  genetic drift

(c)  random mating

(d)  genetic load

Answer: (b)

148. The first stable product of fixation of atmospheric nitrogen in leguminous plant is

(a)   

(b)  ammonia

(c)   

(d)  glutamate

Answer: (b)

149. If two persons with ‘AB’ blood group marry and have sufficiently large number of children, these children could be classified as ‘A’ blood group. ‘AB blood group ‘B’ blood group in 1 : 2 : 1 ratio.

Modern technique of protein electrophoresis reveals presence of both ‘A’ and ‘B’ type proteins in ‘AB’ blood group individuals. This is an example of

(a)  codominance

(b)  incomplete dominance

(c)  partial dominance

(d)  complete dominance

Answer: (a)

150. Which of the following DNA sequences qualifies to be designated as a palindrome?

(a)  5’-GACCAG-3’ in one strand

(b)  3’-GACCAG-5’ in one strand

(c)  5’-GACGAG-3’, 3’-CIGGIC-5’

(d)  5’-AGCGCT-3’, 3’-TCGCGA-5’

Answer: (d)

151. Which one of the following pairs is not correctly matched?

(a)  Vitamin-B­12 : Pernicious anaemia

(b)  Vitamin-B6 : Loss of appetite

(c)  Vitamin-B1 : Beri-beri

(d)  Vitamin-B2: Pellagra

Answer: (d)

152. Humoral immunity is mediated by

(a)  R-cells

(b)  T-cells

(c)  NK-cells

(d)  plasma cells

Answer: (d)

153. In the lac operon model, lactose molecules function as

(a)  inducers, which bind with the operator gene

(b)  repressors, which bind with the operator gene

(c)  inducers, which bind with the repressor protein

(d)  corepressors, which bind with repressor protein

Answer: (c)

154. Which one of the following generally acts as an antagonist to gibberellins?

(a)  Zeatin

(b)  Ethylene

(c)  ABA

(d)  IAA

Answer: (c)

155. The ornithine cycle removes two waste products from the blood in liver. These products are

(a)  CO2 and urea

(b)  ammonia and urea

(c)  CO2 and ammonia

(d)  ammonia and uric acid

Answer: (b)

156. Macromolecule chitin is

(a)  nitrogen containing polysaccharide

(b)  phosphorous containing polysaccharide

(c)  sulphur containing polysaccharide

(d)  simple polysaccharide

Answer: (a)

157. Which of the following statement is correct in relation to the endocrine system?

(a)  Adenohypophysis is under diriect neural regulation of the hypothalamus

(b)  Organs in the body like gastro-intestinal tract, heart, kidney and liver do not produce any hormones

(c)  Non-nutrient chemical produced by the body in trace amount that act as inter-cellular messanger are known as hormones

(d)  Releasing and inhibitory hormones are produced by the pituitary gland

Answer: (c)

158. Following are the two statements regarding the origin of life.

(I) The earliest organisms that appeared on the earth were non-green and presumably an aerobes.

(II) The first autotrophic organisms were the chemoautotrophs that never released oxygen of the above statements which one of the following options is correct?

(a)  II is correct, but I is false

(b)  Both I and II are correct

(c)  Both I and II are false

(d)  I is correct, but II is false

Answer: (b)

159. Taxonomic key is one of the taxonomic tools in the identification and classification of plants and animals. It is used in the preparation of

(a)  monographs

(b)  flora

(c)  Both (a) and (b)

(d)  None of these

Answer: (b)

160. Match the following columns.

(a)  A – 3; B – 2; C – 4; D – 1

(b)  A – 2; B – 3; C – 4; D – 1

(c)  A – 4; B – 3; C – 1; D – 2

(d)  A – 1; B – 4; C – 2; D – 3

Answer: (a)

161. Phellogen and phellem respectively denote

(a)  cork and cork cambium

(b)  cork cambium and cork

(c)  secondary cortex and cork

(d)  cork and secondary cortex

Answer: (b)

162. Which one of the following combinations is incorrect?

(a)  Rio convention : Air pollution

(b)  Kyoto protocol : Climate change

(c)  Montreal protocol : Ozone depletion

(d)  Ramsar convention : Wetland conservation

Answer: (a)

163. ‘Organ of Jacobson’ helps in

(a)  touch

(b)  vision

(c)  smell

(d)  hearing

Answer: (c)

164. Which one correctly describe reproduction and life cycle of fern?

(a)  Spore → Gamete → Prothallus → Sporophyte

(b)  Gamete → Spore → Prothallus → Plant

(c)  Prothallus → Sporophyte → Gamete → Fern

(d)  Sporangia → Spore → Prothallus → Sporophyte → Plant

Answer: (d)

165. Monoclonal antibodies and polyclonal antibodies are produced by

(a)  T-memory cells

(b)  NK-cells

(c)  plasma cells of B-lymphocytes

(d)  memory cells of B-lymphocytes

Answer: (c)

166. All monerons

(a)  contain DNA and RNA

(b)  demonstrate a long circular strand of DNA, not formed enclosed in a nuclear membrane

(c)  are bacteria

(d)  All of the above

Answer: (d)

167. Match the following columns

(a)  A – 1; B – 2; C – 3

(b)  A – 2; B – 3; C – 1

(c)  A – 3; B – 2; C – 1

(d)  A – 2; B – 1; C – 3

Answer: (b)

168. The chromosome in which centromere is situated close to one end are

(a)  metacentric

(b)  acrocentric

(c)  telocentric

(d)  sub-metacentric

Answer: (b)

169. Sliding filament theory can be best explained as

(a)  when myofilaments slide pass each other actin filaments shorten while myosin filament donot shorten

(b)  actin myosin filament shorten and slide pass each other

(c)  actin and myosin filaments do not shorten, but rather slide pass each other

(d)  when myofilament slide pass each other myosin filament shorten while actin filament do not shorten

Answer: (b)

170. Select the correct combination of statements regarding Myasthenia gravis

(I) It is an auto immune disorder.

(II) It causes insufficient acetylecholine binding that effects muscular contraction.

(III) Antibodies are developed against acetylcholine.

(IV) Antibodies are developed against acetylcholine receptors.

(V) It causes drooping of eyelids.

(a)  I, III, IV, VI

(b)  I, III, V, II

(c)  I, II, IV, VI

(d)  II, III, IV, V

Answer: (c)

171. Study the following statements and select the option with correct statements.

(I) Pulvinus leaf base is present in some leguminous plants.

(II) In Eichhornia  the petioles expand, becomes green and synthesise food.

(III) Opposite phyllotaxy is seen in guava.

(a)  I and II

(b)  I and III

(c)  II and III

(d)  I, II and III

Answer: (b)

172. The method of directly injecting a sperm into ovum in assisted reproductive technology is called

(a)  GIFT

(b)  ZIFT

(c)  ICSI

(d)  ET

Answer: (c)

173. Appearance of antibiotic resistant bacteria is an example of

(a)  adaptive radiation

(b)  transduction

(c)  pre-existing variation

(d)  divergent evolution in the population

Answer: (c)

174. Post mitotic gap phase is characterized by all, except

(a)  synthesis of RNA and nucleotides

(b)  no change in DNA content

(c)  synthesis of histone proteins

(d)  growth phase of the cell

Answer: (c)

175. Munch hypothesis is based on

(a)  translocation of food due to Turgor Pressure (TP) gradient and imbibitions force

(b)  translocation of food due to Turgor Pressure (TP) gradient

(c)  translocation of food due to imbition force

(d)  None of the above

Answer: (b)

176. Match the following columns

Column I                                   Column II

(A) Molecular oxygen                (1) α-ketoglutaric acid

(B) Electron acceptor                 (2) Hydrogen acceptor

(C) Pyruvate dehydrogenase      (3) Cytochrome-C

(D) Decarboxylation                    (4) Acetyle Co-A

(a)  A – 2; B – 3; C – 4; D – 1

(b)  A – 3; B – 4; C – 2; D – 1

(c)  A – 2; B – 1; C – 3; D – 4

(d)  A – 4; B – 3; C – 1; D – 2

Answer: (a)

177. Roquefort cheese is ripended by using a

(a)  type of yeast

(b)  fungus

(c)  bacterium

(d)  cyanobacteria

Answer: (b)

178. Identify the wrong combination

(a)  Dryopteris : Rhizome

(b)  Cycas : Corralloid roots

(c)  Volvox : Colonial form

(d)  Marchantia : Pseudoelaters

Answer: (d)

179. Foramen ovale

(a)  connects the two atria in the foetal heart.

(b)  is a condition in which the heart valves do not completely close

(c)  is a shallow depression in the inter ventricular septum

(d)  is a connection between the pulmonary trunk and the aorta in the foetus

Answer: (a)

180. Which one of the following graphs correctly describes disruptive selection? When studying fitness level associated with body size?

(a) 

(b)  

(c) 

(d)

Answer: (d)

JIPMER MBBS Entrance Question Paper – 2017

ENGLISH & QUANTITATIVE REASONING

Directions (Q. Nos. 181-183) Choose the word which best expresses the meaning of the underlined word in the sentence

181. Decay is an immutable factor of human life.

(a)  important

(b)  unique

(c)  unchangeable

(d)  awful

Answer: (c)

182. It was an ignominious defeat for the team.

(a)  shameful

(b)  admirable

(c)  unaccountable

(d)  worthy

Answer: (a)

183. His conjecture was better than mine.

(a)  guess

(b)  fact

(c)  surprise

(d)  doubt

Answer: (a)

Directions (Q. Nos. 184-186) Fill in the blank.

184. Freedom and equality are the …………… rights of every human.

(a)  inalienable

(b)  inscrutable

(c)  incalculable

(d)  incredible

Answer: (d)

185. Pradeep’s face spoke …………. for the happiness he was feeling.

(a)  elegantly

(b)  tons

(c)  volumes

(d)  much

Answer: (c)

186. His speech was disappointing : it ……………. all the major issues.

(a)  projected

(b)  revealed

(c)  skirted

(d)  analyzed

Answer: (c)

Directions (Q. Nos. 187-189) Choose the word which is closest to the opposite in meaning of the following sentence.

187. Hydra is biologically believed to be

(a)  undying

(b)  perishable

(c)  ancient

(d)  eternal

Answer: (b)

188. The Gupta rulers advocated all cultural activities and thus Gupta period was called the golden era in Indian History.

(a)  fostered

(b)  enriched

(c)  opposed

(d)  spurned

Answer: (c)

189. This is a barbarous

(a)  bad

(b)  good

(c)  civilised

(d)  exemplary

Answer: (c)

190. Though novice in art shows great promise.

(a)  tyro

(b)  inexperienced

(c)  veteran

(d)  green horn

Answer: (c)

191. If 15th August 2011 was Tuesday, then what day of the week was it on 17th September, 2011?

(a)  Thursday

(b)  Friday

(c)  Saturday

(d)  Sunday

Answer: (d)

192. In the following question five figures are given. Out of them, find the three figures that can be joined to form a square.

(a)  1

(b)  2

(c)  4

(d)  5

Answer: (a)

193. In a certain code language, ‘DOME’ is written as ‘8943’ and ‘MEAL’ is written as ‘4321’. What group of letters can be formed for the code ‘38249’?

(a)  EOADM

(b)  MEDOA

(c)  EDAMO

(d)  EMDAO

Answer: (d)

194. Replace the question mark (?) in the series given below with the correct option.

4, 5, 7, 11, 19, 35, ?

(a)  67

(b)  76

(c)  55

(d)  45

Answer: (b)

195. Complete the series by replacing ‘?’ mark.

G4T, J9R, M20P, P43N, S90L, ?

(a)  S90L

(b)  V185J

(c)  M20P

(d)  P43N

Answer: (d)

196. Neeraj starts walking towards South. After walking 15 m, he turns towards North. After walking 20 m, he turn towards East and walks 10 m. He then turns towards South and walks 5 m. How far is he from his original position and in which direction?

(a)  10 m, East

(b)  10 m, South-East

(c)  10 m, West

(d)  10 m, North-East

Answer: (b)

197. Shikha is mother-in-law of Ekta who is sister-in-law of Ankit. Pankaj is father of Sanjay, the only brother of Ankit. How is Shikha related to Ankit?

(a)  Mother-in-law

(b)  Aunt

(c)  Wife

(d)  Mother

Answer: (c)

198. In a row of forty children, P is thirteenth from the left end and Q is ninth from the right end. How many children are there between P and R, if R is fourth to the left of Q?

(a)  12

(b)  13

(c)  14

(d)  15

Answer: (a)

199. From the given four positions of a single dice, find the colour at the face opposite to the face having red colour.

(a)  Yellow

(b)  Pink

(c)  Green

(d)  Black

Answer: (a)

200. Choose the answer figure which completes the problem figure matrix.

Answer: (c)

JIPMER Medical Entrance Exam Previous Year Question Paper 2013 With Answer Key

JIPMER

Medical Entrance Exam Previous Year-2013

Physics

1. A car of mass 1000 kg moves on a circular track of radius 40 m. If the coefficient of friction is 1.28. The maximum velocity with which the car can be moved, is

(a)   22.4 m/s

(b)   112 m/s

(c)    

(d)   1000 m/s

Ans: (a)

2. The escape velocity for the earth is 11.2 km/s. The mass of another planet 100 times mass of earth and its radius is 4 times radius of the earth. The escape velocity for the planet is

(a)   280 km/s

(b)   56.0 km/s

(c)   112 km/s

(d)   56 km/s

Ans: (b)

3. Light ravels faster in air than that in glass. This is accordance with

(a)   wave theory of light

(b)   corpuscular theory of light

(c)   neither (a) nor (b)

(d)   Both (a) and (b)

Ans: (a)

4. The speed of air flow on the upper and lower surface of a wing of an aeroplane are υ1 If A is the cross section area of the wing and ρ is the density of air, then the upward life is

(a)    

(b)    

(c)    

(d)    

Ans: (c)

5. A body is thrown with a velocity of 8.8 m/s making an angle of 30° with the horizontal. It will hit the ground after a time

(a)   1.5s

(b)   1s

(c)   3s

(d)   2s

Ans: (b)

6. A radioactive element 90X238 decays into 83Y222. The number of β-particles emitted are

(a)   1

(b)   2

(c)   4

(d)   6

Ans: (a)

7. Minimum excitation potential of Bohr’s first orbit in hydrogen atom is

(a)   3.6 V

(b)   10.2 V

(c)   13.6 V

(d)   3.4 V

Ans: (b)

8. A gas expands 0.25 m3 at constant pressure 103 N/m2, the work done is

(a)   250 N

(b)   250 W

(c)   250 J

(d)   2.5 erg

Ans: (c)

9. The work done in increasing the size of soap film for 10 cm × 6 m to 10 cm × 11cm is 3 × 10−4 The surface tension of the film is

(a)   1.0 × 102 N/m

(b)   6.0 × 102 N/m

(c)   3.0 × 102 N/m

(d)   1.5 × 102 N/m

Ans: (c)

10. A parallel plate condenser is filled with two dielectrics as shown in figure. Area of each pate is A m2 and the separation is d metre. The dielectric constants are K1 and K2 respectively. Its capacitance in farad will be 

(a)    

(b)   

(c)    

(d)    

Ans: (b)

11. A luminous object is placed at a distance of 30 cm from the convex lens of focal length 20 cm. On the other side of the lens, at what distance from the lens a convex mirror of radius of curvature 10 cm be placed in order to have an upright image of the object coincident with it

(a)   30 cm

(b)   60 cm

(c)   50 cm

(d)   12 cm

Ans: (c)

12. A battery of emf 10 V and internal resistance of 0.5 ohm is connected across a variable resistance R. The maximum value of R is given by

(a)   0.5Ω

(b)   1.00Ω

(c)   2.0Ω

(d)   0.25Ω

Ans: (a)

13. For a gas  This gas is made up of molecules which are

(a)   mono atomic

(b)   poly atomic

(c)   mixture of diatomic and poly atomic molecules

(d)   diatomic

Ans: (a)

14. A point source of light is placed 4 m below the surface of water of refractive index 5/3. The minimum diameter of a disc which should be placed over the source on the surface of water to cut-off all light coming out of water is

(a)   6 m

(b)   3 m

(c)   4 m

(d)   2 m

Ans: (a)

15. A moving body of mass m and velocity 3 km/h collides with a rest body of mass 2 m and stick to it. Now the combined mass starts to move. What will be the combined velocity?

(a)   2 km/h

(b)   1 km/h

(c)   2 km/h

(d)   3 km/h

Ans: (b)

16. A transverse wave is represented by the equation

For what value of λ is the particle velocity equal to two times the wave velocity

(a)    

(b)    

(c)   

(d)    

Ans: (a)

17. Ionisation potential of hydrogen atom is 13.6 eV. Hydrogen atom on the ground state rarely excited by monochromatic radiation of photon 12.1 eV. The special line mitted by a hydrogen atom according to Bohr’s theory will be

(a)   one

(b)   two

(c)   three

(d)   four

Ans: (c)

18. The internal resistance of a primary cell is 4Ω. It generates a current of 0.2 A in an external resistance of 21Ω. The rate at which chemical energy to consumed in providing current is

(a)   1 J/s

(b)   5 J/s

(c)   0.42 J/s

(d)   0.8 J/s

Ans: (a)

19. The binding energy per nucleon is maximum in the case

(a)    

(b)   

(c)   

(d)    

Ans: (c)

20. Two rigid bodies A and B rotate with rotational kinetic energies EA and EB respectively. The moments of inertia of A and B about the axis of rotation are IA and IB respectively.

If  and EA = 100 = EB, the ratio of angular momentum (LA) of A to the angular momentum (LB) of B is

(a)   25

(b)   5/4

(c)   5

(d)   1/4

Ans: (c)

21. The working principle of a ball point pen is

(a)   Bernoulli’s theorem

(b)   surface tension

(c)   gravity

(d)   viscosity

Ans: (b)

22. Progressive waves are represented by the equation

y1 = a sin(ωt – x)

and      y2 = b cos (ωt – x)

The phase difference between waves is

(a)   0°

(b)   45°

(c)   90°

(d)   180°

Ans: (c)

23. Two simple pendulums of length 0.5 m and 20 m respectively are given small linear displacement in one direction at the same time. They will gain be in the phase when the pendulum of shorter length has completed x oscillations, where k is

(a)   1

(b)   3

(c)   2

(d)   5

Ans: (c)

24. A balloon contains 500 m3 of helium at 27℃ and 1 atmosphere pressure. The volume of the helium at −3℃ temperature and 0.5 atmosphere pressure will be

(a)   1000 m3

(b)   900 m3

(c)   700 m3

(d)   500 m3

Ans: (b)

25. 220 V, 50 Hz, AC source is connected to an inductance of 0.2 H and a resistance of 20 Ω in series. What is the current in the circuit?

(a)   3.33 A

(b)   33.3 A

(c)   5 A

(d)   10 A

Ans: (a)

26. In 0.2 s, the current in a coil increases from 2.0 A to 3.0 A. If inductance of coil is 60 mH, then induced current in external resistance of 3 Ω will be

(a)   1 A

(b)   0.5 A

(c)   0.2 A

(d)   0.1 A

Ans: (d)

27. Two coherent light beams of intensities I and 4I are superposed. The maximum and minimum possible intensities in the resulting beam are

(a)   5I and I

(b)   5I and 3I

(c)   9I and I

(d)   9I and 3I

Ans: (c)

28. A galvanometer acting as a voltmeter should have

(a)   low resistance in series with its coil

(b)   low resistance in parallel with its coil

(c)   high resistance in series with its coil

(d)   high resistance in parallel with its coil

Ans: (c)

29. The equivalent resistance across A and B is

(a)   2 Ω

(b)   3 Ω

(c)   4 Ω

(d)   5 Ω

Ans: (c)

30. A black body has a wavelength of λ at temperature 2000 K. Its corresponding wavelength at temperature 3000 K will be

(a)    

(b)   

(c)    

(d)    

Ans: (a)

31. At room temperature, copper has free electron density of 8.4 × 1028 m−3. The electron drift velocity in a copper conductor of cross-sectional area of 10−6 m2 and carrying a current of 5.4 A, will be

(a)   4 ms1

(b)   0.4 ms1

(c)   4 cm ms1

(d)   0.4 mm s1

Ans: (d)

32. A uniform wire of resistance R and length L is cut into four equal parts, each of length L/4 which are then connected in parallel combination. The effective resistance of the combination will be

(a)   R

(b)   4 R

(c)   R/4

(d)   R/16

Ans: (d)

33. The half-life of radio isotope is 4 h. If initial mass of the isotope was 200 g, then mass remaining after 24 h will be

(a)   1.042 g

(b)   2.084 g

(c)   3.125 g

(d)   4.167 g

Ans: (c)

34. Which logic gate is represented by the following combination of logic gates?

(a)   OR

(b)   NOR

(c)   AND

(d)   NAND

Ans: (c)

35. The work function for metals A, B and C are respectively 1.92 eV, 2.0 eV and 5eV. According to Einstein’s equation the metals which will emit photo, electrons for a radiation of wavelength 4100 Å is/are

(a)   none

(b)   A only

(c)   A and B only

(d)   All the three metals

Ans: (c)

36. Two boys are standing at the ends A and B of a ground, where AB = a. The boy at B starts running in a direction perpendicular to AB with velocity υ1. The boy at A starts running simultaneously with velocity υ and catches the other boy in a time t, where t is

(a)    

(b)   

(c)    

(d)   

Ans: (b)

37. A 5 amp fuse wire can withstand a maximum power of 1 W in circuit. The resistance of the fuse wire is

(a)   0.2 Ω

(b)   5 Ω

(c)   0.4 Ω

(d)   0.04 Ω

Ans: (d)

38. A force F is given F = at + bt2, where, t is time. What are the dimensions of a and b?

(a)    

(b)    

(c)    

(d)     

Ans: (d)

39. Two equal negative charges –q are fixed at the point (0, α) and (0, −α) on the y-axis. A positive charge Q is released from rest at the point (2a, 0) on the x-axis. The charge will

(a)   execute SHM about the origin

(b)   move to the origin and remain at rest

(c)   move to infinity

(d)   execute oscillatory but not SHM

Ans: (d)

40. An ice-cube of density 900 kg/m3. The percentage of volume of ice-cube outside the water is

(a)   20%

(b)   35%

(c)   10%

(d)   25%

Ans: (c)

Chemistry

1. 

The above reaction is known as

(a)   aromatisation

(b)   pyrolysis

(c)   isomerisation

(d)   oxidation

Ans: (c)

2. Number of hydrogen-bonded water molecules associated in CuSO4 ∙ 5H2O is

(a)   one

(b)   two

(c)   three

(d)   All the live

Ans: (a)

3. Which of the following species do not show disproportionation on reaction?

(a)   

(b)  

(c)    

(d)  

Ans: (a)

4. Which one of the following acts as nucleophile?

(a)

(b)    

(c)

(d)    

Ans: (a)

5. During estimation of nitrogen in the organic compound by Kjeldahl’s method, the ammonia evolved from 0.5 g of the compound in Kjeldahl’s estimation of nitrogen, neutralized 10 mL of 1M H2SO4. Find out the percentage of nitrogen in the compound.

(a)   14%

(b)   28%

(c)   56%

(d)   68%

Ans: (c)

6. Which of the following compounds have highest melting point?

(a)   Only I

(b)   Only II

(c)   I and II

(d)   II and III

Ans: (b)

7. Identify the major product ‘X’ obtained in the following reaction.

(a) 

(b) 

(c) 

(d) 

Ans: (a)

8. Addition of water of alkynes occurs in acidic medium and in the presence Hg2+ ions as a catalyst. Which of the following products will be formed on addition of water to but-1-yne under these conditions?

(a)   CH3CH2CH2CHO

(b)   CH3CH2COCH3

(c)   CH3CH2COOH + CO2

(d)   CH3COOH + HCHO

Ans: (b)

9. The correct order of increasing acidic strength is

(a)   phenol < ethanol < chloroacetic acid < acetic acid

(b)   ethanol < phenol < chloroacetic acid < acetic acid

(c)   ethanol < phenol < acetic acid < chloroacetic acid

(d)   chloroacetic acid < acetic acid < phenol < ethanol

Ans: (c)

10. KF has ccp structure. How many F ions and octahedral voids are there in this unit cell respectively

(a)   4 and 4

(b)   4 and 8

(c)   8 and 4

(d)   6 and 6

Ans: (a)

11. The osmotic pressure of blood is 8.21 atm at 37℃. How much glucose would be used for injection that is at the same osmotic pressure as blood?

(a)   22.17 gL1

(b)   58.14 gL1

(c)   61.26 gL1

(d)   75.43 gL1

Ans: (b)

12. At equilibrium, the rate of dissolution of a solid solute in a volatile liquid solvent is

(a)   less than the rate of crystallisation

(b)   greater than the rate of crystallisation

(c)   equal to the rate of crystallisation

(d)   zero

Ans: (c)

13. A chelating agent has two or more than two donor atoms to bind a single metal ion. Which of the following is not a chelating agent?

(a)   Thiosulphato

(b)   Glycinato

(c)   Oxalato

(d)   Ethane-1, 2-diamine

Ans: (a)

14. On addition of small amount of KMnO4 to conc. H2SO4, a green oily compound is obtained which is highly explosive in nature. Identify the compound from the following.

(a)   Mn2O7

(b)   MnO2

(c)   MnSO4

(d)   Mn2O3

Ans: (a)

15. The magnetic nature of elements depends on the presence of unpaired electrons. Identify the configuration of transition element, which shows highest magnetic moment.

(a)   3d7

(b)   3d5

(c)   3d8

(d)   3d2

Ans: (b)

16. Which of the following elements can be involved in pπ – pπ bonding?

(a)   Carbon

(b)   Nitrogen

(c)   Phosphorus

(d)   Boron

Ans: (c)

17. On addition of conc. H2SO4 to a chloride salt, colourless fumes are evolved but in case o iodide slat, violet fumes come out. This is because.

(a)   H2SO4 reduces HI to I2

(b)   HI is of violet colour

(c)   HI gets oxidized to I2

(d)   HI changes to HIO3

Ans: (c)

18. Affinity for hydrogen decreases in the group from fluorine to iodine. Which of the halogen acids should have highest bond dissociation enthalpy?

(a)   HF

(b)   HCl

(c)   HBr

(d)   HI

Ans: (a)

19. Which of the following statement is not correct about an inert electrode in a cell?

(a)   It does not participate in the cell reaction.

(b)   It provides surface either for oxidation or for reduction reaction.

(c)   It provides surface for conduction of electrons.

(d)   It provides surface for redox reaction

Ans: (d)

20. Which of the following statement is correct?

(a)   Ecell and ∆rG of cell reaction both are extensive properties.

(b)   Ecell and ∆rG of cell reaction both are intensive properties.

(c)   Ecell in the intensive property while ∆rG of cell reaction is an extensive properly.

(d)   Ecell is an extensive property while ∆rG of cell reaction is an intensive property.

Ans: (c)

21. Which of the following curves is in accordance with Freundlich adsorption isotherm?

(a) 

(b) 

(c) 

(d) 

Ans: (c)

22. A number of elements available in earth’s crust but most abundant elements are

(a)   Al and Fe

(b)   Al and Cu

(c)   Fe and Cu

(d)   Cu and Ag

Ans: (a)

23. The element which forms oxides in all oxidation states +1 to +5 is

(a)   nitrogen

(b)   phosphorus

(c)   arsenic

(d)   antimony

Ans: (a)

24. Which of the following is the increasing order of enthalpy of vaporization?

(a)   NH3, PH3, AsH3

(b)   AsH3, PH3, NH3

(c)   NH3, AsH3, PH3

(d)   PH3, AsH3, NH3

Ans: (d)

25. When Br2 is treated with aqueous solutions of NaF, NaCl, NaI separately

(a)   F2, Cl2 and I2 are liberated

(b)   only F2 and Cl2 are liberated

(c)   only I2 is liberated

(d)   only Cl2 is liberated

Ans: (c)

26. In the presence of a catalyst, the heat evolved or absorbed during the reaction

(a)   increases

(b)   decreases

(c)   remains unchanged

(d)   may increase or decrease

Ans: (c)

27. The rate of gaseous reaction is given by the expression k[A][B]. If the volume of reaction vessel is suddenly reduced to 1/4th of the initial volume, the reaction rate relating to original rate will be

(a)   1/10

(b)   1/8

(c)   8

(d)   16

Ans: (d)

28. Which of the following is 3° amine?

(a)   1-methylcyclohexylamine

(b)   Triethyl amine

(c)   Tert-butylamine

(d)   N-methyl aniline

Ans: (b)

29. Which of the following enhances lathering property of soap?

(a)   Sodium carbonate

(b)   Sodium rosinate

(c)   Sodium stearate

(d)   Trisodium phosphate

Ans: (b)

30. The deficiency of vitamin C causes

(a)   scurvy

(b)   rickets

(c)   pyrrohea

(d)   pernicious anaemia

Ans: (a)

31. Excess fluoride (over 10 ppm) in drinking water can cause

(a)   harmful effect of bones and teeth

(b)   methemoglobinemia

(c)   kidney damage

(d)   laxative effect

Ans: (a)

32. For the process to occur under adiabatic conditions, the correct condition is

(a)   ∆T = 0

(b)   ∆p = 0

(c)   q = 0

(d)   W= 0

Ans: (c)

33.  Kp for this reaction is 2.47 × 10−29. At 298 K, ∆rG° for conversion of oxygen to ozone will be

(a)   100 kJ mol1

(b)   150 kJ mol1

(c)   163 kJ mol1

(d)   2303 kJ mol1

Ans: (c)

34. Which one of the following statements about C2 molecule is wrong?

(a)   The bond order of C2 is 2.

(b)   In vapour phase, C2 molecule is diamagnetic.

(c)   Double bond in C2 molecule consists of both π-bonds because of the presence of 4es in two π-molecular orbitals.

(d)   double bond in C2 molecule consists of one σ-bond and on π-bond.

Ans: (d)

35. The type of hybridization in SF6 molecule is

(a)   sp3d

(b)   dsp3

(c)   sp3d2

(d)   d2sp3

Ans: (c)

36. Among LiCl, BeCl2, BCl3 and CCl4, the covalent bond character follows the order

(a)   LiCl < BeCl2 < BCl3 < CCl4

(b)   BCl3 < CCl4 < BeCl2 < LiCl

(c)   BeCl2 < LiCl < CCl4 < BCl3

(d)   CCl4 < BCl3 < BeCl2 < LiCl

Ans: (a)

37. Maximum number of electrons in a subshell of an atom determined by the following?

(a)   4l + 2

(b)   2n2

(c)   4l – 2

(d)   2l + 1

Ans: (a)

38. The average kinetic energy of an ideal gas per molecule in SI units at 25℃ will be

(a)   6.17 × 1021 JK1

(b)   6.17 × 1021 kJK1

(c)   6.17 × 1020 JK1

(d)   7.16 × 1020 JK1

Ans: (a)

39. pKa of acetic acid and pKb of ammonium hydroxide are 4.76 and 4.75 respectively. Calculate the pH of ammonium acetate solution.

(a)   6.02

(b)   7.005

(c)   8

(d)   5.602

Ans: (b)

40. The value of Kc for the reaction,

2A ⇋ B + C is 2 × 10−3. At a given time, if the composition of reaction mixture is [A] = [B] = [C] = 3 × 10−3 M. Which is true?

(a)   The reaction will be proceed in forward direction

(b)   The reaction will proceed in backward direction

(c)   The reaction will proceed in any direction

(d)   None of the above

Ans: (b)

Zoology

1. Pellagra is caused by deficiency of vitamins

(a)   B5

(b)   B2

(c)   B6

(d)   B1

Ans: (a)

2. Notochord originates from

(a)   mesoderm

(b)   ectoderm

(c)   endoderm

(d)   None of these

Ans: (a)

3. Parthenogenesis is a term of

(a)   budding

(b)   asexual reproduction

(c)   sexual reproduction

(d)   regeneration

Ans: (c)

4. Bartholin’s gland is found in

(a)   penis

(b)   stomach

(c)   liver

(d)   vagina

Ans: (d)

5. Which one of the following statements best characterize the testis?

(a)   The seminiferous epithelium contains only proliferative cells

(b)   Functional compartmentalization of the seminiferous epithelium depends on tight junctions

(c)   The interstitial tissue contains few capillaries

(d)   The seminiferous epithelium contains numerous capillaries

Ans: (b)

6. Drugs that cause malformation in developing embryo during pregnancy are called

(a)   teratogens

(b)   nicotine

(c)   tranquillisers

(d)   alcoholic beverages

Ans: (a)

7. Which set is similar?

(a)   Corpus luteum          -Graafian follicles

(b)   Sebum                       -Sweat

(c)   Vitamin-B7               -Niacin

(d)   Bundle of His            -Pacemaker

Ans: (a)

8. Which one out of (a) to (d) given below correctly represents the structural formula of the basic amino acid?

(a) 

(b) 

(c) 

(d) 

Ans: (d)

9. Given below is a schematic break-up of the phases/stages of cell cycle. Which one of the following is the correct indication of the stage/phase in the cell cycle?

(a)   C-karyokinesis

(b)   S-synthetic phase

(c)   A-cytokinesis

(d)   B-metaphase

Ans: (b)

10. Which one of the following structural formula of two organic compounds is correctly identified along with its related function?

(a)   B-uracil –a component of DNA

(b)   A-triglyceride – major source of energy

(c)   A-lecithin – a component of cell membrane

(d)   B-adenine – nucleotide that makes up nucleic

Ans: (c)

11. Which is substitution of mitochondria in coli?

(a)   Golgi body

(b)   Mesosome

(c)   Ribosome

(d)   Glyoxysomes

Ans: (b)

12. Animal cell differ from plant cell in possessing

(a)   vacuoles

(b)   centrosomes

(c)   pastids

(d)   mitochondria

Ans: (b)

13. Which of the following organelles does not contain RNA?

(a)   Plasmalemma

(b)   Ribosome

(c)   Chromosome

(d)   Nucleolus

Ans: (a)

14. Dutrochet has given the concept about cell in

(a)   1834

(b)   1814

(c)   1822

(d)   1824

Ans: (d)

15. The scientific name of gharial is

(a)   Naja bungarus

(b)   Gavialis gangeticus

(c)   Hemidactylus flavivridis

(d)   None of the above

Ans: (b)

16. Which of the given option is correct regarding the statements?

Statement I  Cephalochordata bears notochord all along the body throughout life.

Statement II Urochordate bears vertebral column only in tail region throughout the life.

(a)   I wrong, II correct

(b)   I correct, II wrong

(c)   Both I and II are wrong

(d)   Both are correct

Ans: (b)

17. In which of the following haemocyanin pigment is found?

(a)   Lower invertebrates

(b)   Echinodermata

(c)   Insecta

(d)   Annelida

Ans: (a)

18. Which of the following cells in earthworm play a role similar to liver in vertebrates?

(a)   Amoebocytes

(b)   Mucocytes

(c)   Chloragogen cells

(d)   Epidermal cells

Ans: (c)

19. Match the following and select the correct option.

(a)   A-3, B-5, C-2, D-1, E-4

(b)   A-3, B-1, C-5, D-2, E-4

(c)   A-1, B-2, C-3, D-4, E-5

(d)   A-2, B-3, C-4, D-1, E-5

Ans: (a)

20. Chondrichthyes is characterized by

(a)   placoid scale

(b)   placoid scale and ventral mouth

(c)   ventral mouth

(d)   ctenoid scale and ventral mouth

Ans: (b)

21. Ichthylogy is study of

(a)   aves

(b)   amphibians

(c)   reptiles

(d)   fishes

Ans: (d)

22. What will happen if ligaments are torn?

(a)   Bone will become unfixed

(b)   Bone will become fixed

(c)   Bone less movable at joint and pain

(d)   Bone will move freely at joint and no pain

Ans: (c)

23. Achondroplasia is a disease related with the defect in the formation of

(a)   membrane

(b)   mucosa

(c)   bone

(d)   cartilage

Ans: (d)

24. Yellow bone marrow is found specially in the medullary cavity

(a)   long  bones

(b)   spongy bones

(c)   short bones

(d)   All of the above

Ans: (a)

25. Match the items of column I with column II and choose the correct option from the code given below.

(a)   A-4, B-1, C-2, D-3

(b)   A-2, B-1, C-4, D-3

(c)   A-3, B-4, C-1, D-2

(d)   A-2, B-3, C-4, D-1

Ans: (b)

26. Space in the jaw bone unoccupied by teeth is called

(a)   dentine

(b)   diastema

(c)   enamel

(d)   crown

Ans: (b)

27. Identify the correct set, which shows the name of the enzyme from where it is secreted and substrate upon which it acts.

(a)   Ptyalin – Intestine – Maltose

(b)   Ptyalin – Pancreas – Lipid

(c)   Pepsin – Stomach wall – Caesin

(d)   Chymotrypsin – Salivary gland – Lactose

Ans: (c)

28. Endemic goiter is a state of

(a)   normal thyroid function

(b)   moderate thyroid function

(c)   increased thyroid function

(d)   decreased thyroid function

Ans: (d)

29. Hormone responsible for the secretion of milk after parturition is

(a)   ACTH

(b)   LH

(c)   ICSH

(d)   Prolactin

Ans: (d)

30. What is another name for the wind pipe?

(a)   Trachea

(b)   Larynx

(c)   Oesophagus

(d)   Lungs

Ans: (a)

31. Soil salinity is measured by

(a)   Porometer

(b)   Calorimeter

(c)   Conductivity meter

(d)   Potometer

Ans: (c)

32. Predation and parasitism are which type of interactions.

(a)   (+, +)

(b)   (+, 0)

(c)   (−, −)

(d)   (+, −)

Ans: (d)

33. The ultimate source of energy for living being is

(a)   sunlight

(b)   ATP

(c)   fats

(d)   carbohydrates

Ans: (a)

34. Which of the following species are restricted to an area?

(a)   Sympatric species

(b)   Sibling species

(c)   Allopatric species

(d)   Endemic species

Ans: (d)

35. Select the incorrect statement.

(a)   Stellar’s sea cow and passenger pigeon got extinct due to over exploitation by men

(b)   The mitotic convention on biological diversity was held in 1992

(c)   Species diversity increase as we move away from the equator towards the poles

(d)   Lantana and Eichhornia are invasive weed species in India

Ans: (c)

36. The effect of cigarette smoking and radon in combination on lungs is

(a)   fatal

(b)   synergistic

(c)   mutualistic

(d)   antagonistic

Ans: (b)

37. The thermostable enzymes, Taq and pfu, isolated from thermophilic bacteria are

(a)   RNA polymerases

(b)   DNA ligases

(c)   DNA polymerases

(d)   restriction endonucleases

Ans: (c)

38. Biolistic technique is used in

(a)   gene transfer process

(b)   tissue culture process

(c)   hybridisation process

(d)   germplasm conservation process

Ans: (a)

39. The largest gene in man is

(a)   insulin gene

(b)   tumour suppressor gene

(c)   beta globin gene of haemoglobin

(d)   dystrophin

Ans: (d)

40. Herbicide resistant gene in plant is

(a)   Mt

(b)   Gt

(c)   Ct

(d)   Bt

Ans: (d)

Botany

1. In photosynthesis carbon dioxide is converted to carbohydrates. It is a …… process.

(a)   reductive

(b)   oxidative

(c)   catabolic and exergonic

(d)   None of the above

Ans: (d)

2. Which of the following is not an auxin?

(a)   IAA

(b)   IBA

(c)   Zeatin

(d)   NAA

Ans: (c)

3. Which of the following properties is shown by cytokinins?

(a)   Delay leaf senescence

(b)   Cause leaf abscission

(c)   Promote seed dormancy

(d)   Promote stomatal closing

Ans: (a)

4. Which of the following characteristics is are exhibited by C4-plants?

I. Kranz anatomy.

II. The first product of photosynthesis is oxaloacetic acid.

III. Both PEP carboxylates and ribulose biphosphate carboxylate act as carboxylating enzymes.

The correct option is

(a)   I and III, but not II

(b)   I and II, but not III

(c)   II and III, but not I

(d)   II and III

Ans: (d)

5. Which of the following plant keeps it stomata open during night and closed during the day?

(a)   Orchid

(b)   Cactus

(c)   Tea

(d)   Wheat

Ans: (b)

6. Genetic dwarfism can be overcome by

(a)   gibberellin

(b)   ethyelen

(c)   auxin

(d)   ABA

Ans: (a)

7. Hormone inducing fruit ripening is

(a)   cytokinin

(b)   ethylene

(c)   abscissic acid

(d)   gibberellic acid

Ans: (b)

8. The year 1900 AD is highly significant for geneticists due to

(a)   discovery of genes

(b)   principle of linkage

(c)   chromosome theory of heredity

(d)   rediscovery of Mendelism

Ans: (d)

9. F­1-generation means

(a)   first filial generation

(b)   first seed generation

(c)   first flowering generation

(d)   first fertile generation

Ans: (a)

10. Skin colour is controlled by

(a)   single gene

(b)   3 pair of genes

(c)   2 pairs of genes

(d)   2 pairs of genes with an intragene

Ans: (b)

11. Which of the following cross will produce terminal flower in garden pea?

(a)   AA × Aa

(b)   AA × aa

(c)   Aa × Aa

(d)   Aa × AA

Ans: (c)

12. Which one of the following pairs of plants are not seed producers?

(a)   Funaria and Ficus

(b)   Fern and Funaria

(c)   Funaria and Pinus

(d)   Ficus and Chlamydomonas

Ans: (b)

13. Which one of the following is heterosporous?

(a)   Equisetum

(b)   Dryopteris

(c)   Salvinia

(d)   Adiantum

Ans: (c)

14. Cycas revolute is popularly known as

(a)   sago palm

(b)   royal palm

(c)   data palm

(d)   sea palm

Ans: (a)

15. Match the following with correct combination.

(a)   A-3, B-1, C-5, D-4, E-2

(b)   A-2, B-3, C-1, D-5, E-4

(c)   A-4, B-3, C-1, D-5, E-2

(d)   A-4, B-5, C-1, D-2, C-3

Ans: (d)

16. Bacterial endotoxin is

(a)   a toxic protein that stays inside the bacterial cell

(b)   a toxic protein that is excreted into the medium

(c)   lipopolysaccharide located on the surface of the bacteria

(d)   None of the above

Ans: (c)

17. Endosperm of gymnosperms is

(a)   haploid

(b)   tetraploid

(c)   diploid

(d)   None of these

Ans: (a)

18. First vascular plant is

(a)   thallophyta

(b)   pteridophyta

(c)   bryophyta

(d)   spermatophyta

Ans: (b)

19. Diatomaceous earth is obtained from

(a)   Bacillarophyceae

(b)   Xanthophyceae

(c)   Rhodophyceae

(d)   Chrysophyceae

Ans: (a)

20. Which of the following is an epidermal cell containing chloroplast?

(a)   Stomata

(b)   Hydathode

(c)   Guard cell

(d)   None of these

Ans: (c)

21. The structures present in the roots to absorb water and minerals is

(a)   epidermal extensions

(b)   hypodermis

(c)   endodermis

(d)   epidermal appendages

Ans: (a)

22. Lady finger belongs to family

(a)   Malvaceae

(b)   Cucurbitaceae

(c)   Brassicaceae

(d)   Liliaceae

Ans: (a)

23. The interxylary phloem is found in the stem of

(a)   Cucurbita

(b)   Salvia

(c)   Calotropis

(d)   None of these

Ans: (c)

24. Wound healing is due to

(a)   ventral meristem

(b)   secondary meristem

(c)   primary meristem

(d)   All of these

Ans: (b)

25. Angular collenchymas occurs in

(a)   Salvia

(b)   Helianthus

(c)   Althaea

(d)   Cucurbita

Ans: (d)

26. In pteridophytes, phloem is without

(a)   bast fibers

(b)   sieve tubes

(c)   companion cells

(d)   sieve cells

Ans: (c)

27. Match the following entities of column I with their respective orders of column II and choose the correct combination form the option.

(a)   A-4, B-3, C-2, D-1

(b)   A-1, B-2, C-4, D-3

(c)   A-3, B-4, C-2, D-1

(d)   A-2, B-4, C-1, D-3

Ans: (a)

28. Agar-agar is produce by

(a)   fungi

(b)   algae

(c)   bacteria

(d)   blue-green algae

Ans: (b)

29. In DNA, when AGCT occurs, their association is as per which of the following pair

(a)   A-G, C-T

(b)   A-T, G-C

(c)   A-C, G-T, A-C, E-T

(d)   All of these

Ans: (b)

30. A segment of DNA has 120 adenine and 120 cytosine bases. The total number of nucleotides present in the segment is

(a)   60

(b)   240

(c)   120

(d)   480

Ans: (d)

31. Lactose is composed of

(a)   glucose + glucose

(b)   glucose + galactose

(c)   glucose + fructose

(d)   fructose + galactose

Ans: (b)

32. Meiosis is best observed in dividing

(a)   Cell of lateral meristem

(b)   Cells of apical meristem

(c)   microsporocytes

(d)   microspores and anther wall

Ans: (c)

33. Study the following statements and select the correct option.

I. Tapetum nourishes the developing pollen grains.

II. Hilum represents the junction between ovule and funicle.

III. In aquatic plants such as water hyacinth and water lily, pollination is by water.

IV. The primary endosperm nucleus is triploid.

(a)   I, II, and IV are correct, but III is incorrect

(b)   I and II are correct, but III and IV are incorrect

(c)   I and IV are correct, but II and III are incorrect

(d)   I, III and IV are correct, but I is incorrect

Ans: (a)

34. Masses of pollen grains, i.e., pollinia is found in

(a)   Graminease

(b)   Solanaceae

(c)   Orchidaceae

(d)   Malvaceae

Ans: (c)

35. Morphine, which is used as an analgesic is obtained from

(a)   Taxus brevifolia

(b)   Berberis nilghiriensis

(c)   Cinchona officinalis

(d)   Papaver somniferum

Ans: (d)

36. Pebrine is a disease of

(a)   fish

(b)   honey bee

(c)   silk worm

(d)   lac insect

Ans: (c)

37. Factor govering the earth surface is

(a)   topographic

(b)   edaphic

(c)   temperature

(d)   bioitc

Ans: (a)

38. The direction of energy flow is

(a)   Producers → Herbivores → Decomposers → Omnivores

(b)   Producers → Carnivore → Herbivores → Decomposes

(c)   Decomposers → Carnivores → Herbivores → Producers

(d)   Producers → Herbivores → Carnivores → Decomposers

Ans: (d)

39. If the Bengal tiger become extinct

(a)   hyenas and wolves will become scarce

(b)   its gene pool will be lost forever

(c)   the wild areas will be safe far man and domestic

(d)   the population of beautiful animals like deers will get stabilised

Ans: (b)

40. Biological treatment of water pollution is done with the help of

(a)   fungi

(b)   lichen

(c)   phytoplanktons

(d)   None of the above

Ans: (c)

English

Directions (Q. No. 1-5) In the following questions, sentences are given with blanks to be filled in with an appropriate word. Four alternatives are suggested from each questions. Choose the correct alternative out of the four.

1. The little girl ……. for the light switch in the dark.

(a)   groped

(b)   grappled

(c)   gripped

(d)   groveled

Ans: (a)

2. The summit meeting provided him the much……shot in the arm.

(a)   required

(b)   desired

(c)   needed

(d)   urgent

Ans: (c)

3. We must……..the tickets for the movie in advance.

(a)   draw

(b)   buy

(c)   remove

(d)   take

Ans: (b)

4. The State Transport Corporation has ……..a loss of Rs. 5crore this year.

(a)   obtained

(b)   derived

(c)   incurred

(d)   formulated

Ans: (c)

5. …. and you know who among them is the culprit.

(a)   look

(b)   deep

(c)   sight

(d)   gaze

Ans: (a)

Directions (Q. Nos. 6-10) In the following questions, out of the four alternatives, Choose the one which best express the meaning of the given word.

6. GAINSAY

(a)   Advantage

(b)   Proposal

(c)   Contradict

(d)   Suggestion

Ans: (c)

7. PROFOUND

(a)   Profuse

(b)   Boundless

(c)   Deep

(d)   Fathomless

Ans: (c)

8. FLAK

(a)   Adventure

(b)   Advice

(c)   Criticism

(d)   Praise

Ans: (c)

9. HOODLUM

(a)   Pioneer

(b)   Criminal

(c)   Devotee

(d)   Scholar

Ans: (b)

10. SPASMODIC

(a)   Continuous

(b)   Gradual

(c)   Intermittent

(d)   Spontaneous

Ans: (c)

Directions (Q. Nos. 11-15) In the following questions, choose the word opposite in meaning to the given word.

11. FILTHY

(a)   Stainless

(b)   Shining

(c)   Sterilised

(d)   Clean

Ans: (d)

12. CROWDED

(a)   Deserted

(b)   Lonely

(c)   Empty

(d)   Barren

Ans: (a)

13. VAGUE

(a)   Known

(b)   Published

(c)   Popular

(d)   Definite

Ans: (d)

14. SUPERVISE

(a)   Overlook

(b)   Misdirect

(c)   Neglect

(d)   Forget

Ans: (b)

15. MAGNANIMOUS

(a)   Selfish

(b)   Naive

(c)   Generous

(d)   Small

Ans: (a)

Directions (Q. Nos. 16-20) In the following questions, four alternatives are given for the idiom/phrase printed in bold in sentence. Choose the alternative which best express the meaning of the idiom/phrase.

16. We have to keep our finger crossed till the final result is declared

(a)   keep praying

(b)   feel suspicious

(c)   wait expectantly

(d)   feel scared

Ans: (c)

17. The members of the group were at odds over the selection procedure.

(a)   acting foolishly

(b)   in dispute

(c)   unanimous

(d)   behaving childishly

Ans: (b)

18. The popularity of the yesteryears’ superstar is one the wane.

(a)   growing more

(b)   at its peak

(c)   growing less

(d)   at rock -bottom

Ans: (c)

19. His father advised him to the fair and square in this dealings lest he should fall into trouble.

(a)   considerate

(b)   upright

(c)   careful

(d)   polite

Ans: (b)

20. There is no love lost between the two neighbours.

(a)   close friendship

(b)   intense dislike

(c)   a love-hate relationship

(d)   cool indifference

Ans: (b)

Directions (Q. Nos. 21-25) In the following questions, a part of the sentence is printed in bold. Below are given alternatives to the bold part at (a), (b), (c) which may improve the sentence. Choose the correct alternative. In case no improvement is needed, your answer (d).

21. Ravi has got many friends because he has got much money.

(a)   Enough money

(b)   A lot of money

(c)   Bags of money

(d)   No improvement

Ans: (b)

22. You must try making him to understand.

(a)   Make him understand

(b)   To making him understand

(c)   To make him understand

(d)   No improvement

Ans: (c)

23. He has cooked that meal so often he can do it with his eyes closed.

(a)   Mind blank

(b)   Eyes covered

(c)   Hands full

(d)   No improvement

Ans: (d)

24. Not a word they spoke to the unfortunate wife about it.

(a)   They had spoken

(b)   Did they speak

(c)   They will speak

(d)   No improvement

Ans: (b)

25. There is sufficient fund to meet the requirement of the entire schools in our zone.

(a)   Schools

(b)   All the schools

(c)   All of the schools

(d)   No improvement

Ans: (b)

Directions (Q. Nos. 26-30) In the following questions, out of the four alternatives, choose the one which can be substituted for the given words/sentence.

26. Word for word reproduction.

(a)   Copying

(b)   Mugging

(c)   Verbatim

(d)   Photostat

Ans: (c)

27. A person who collects coins.

(a)   Philatelist

(b)   Numismatist

(c)   Narcissist

(d)   Fatalist

Ans: (b)

28. That which is perceptible by touch

(a)   Tangible

(b)   Tenacious

(c)   Contagious

(d)   Contingent

Ans: (a)

29. One who possesses many talents.

(a)   Versatile

(b)   Gifted

(c)   Exceptional

(d)   Nubile

Ans: (a)

30. A person who studies the formation of the Earth.

(a)   Meteorologist

(b)   Anthropologist

(c)   Geologist

(d)   Seismologist

Ans: (c)

Directions (Q. Nos. 31-35) In the following questions, the first and the last parts of the sentence are numbered 1 and 6. The rest of the sentence is split into four parts named P, Q, R and S. These parts are not given in their proper order. Rearrange these parts in their proper order and find out which of the given four combination is correct?

31. (1) In reply to a question

(P) that securing extradition

(Q) operating from the UK soil remained

(R) of anti-India elements

(S) The spokesman said

(6) New Delhi’s first priority.

(a)   PRQS

(b)   QSPR

(c)   RQSP

(d)   SPRQ

Ans: (d)

32. (1) The first component is

(P) and vocational training

(Q) so as to enable them

(R) the provision of further technical

(S) so both rural and urban youth

(6) to secure employment in industry and the services sector.

(a)   PRSQ

(b)   RPSQ

(c)   RSQP

(d)   SRPQ    

Ans: (b)

33. (1) The move to revert to a six-day week

(P) among the employees

(Q) while the leaders represented to the Chief Minister

(R) that they be taken into confidence

(S) led to an animated decision

(6) before any decision was taken.

(a)   QPSR

(b)   RSPQ

(c)   SPQR

(d)   SQPR

Ans: (c)

34. (1) It was obvious

(P) made by him

(Q) submitted at the meeting

(R) from the comments

(S) on the draft proposals

(6) that he was not satisfied with them.

(a)   PSRQ

(b)   QRSP

(c)   RPSQ

(d)   SQRP

Ans: (c)

35. (1) The Minister of state for power

(P) in conservation of electricity in industries

(Q) has written to his counterparts in State Government

(S) on bringing about improvement

(6) by introduction of energy efficient equipment.

(a)   QPSR

(b)   RPSQ

(c)   SPQR

(d)   SQPR

Ans: (b)

Directions (Q. Nos. 36-40) Read the following passage carefully and answer the questions given below it.

In this work of incessant and feverish activity, men have little time to think, much less to consider ideals and objectives. Yet how are we to act, even in the present, unless we know which way we are going and what our objectives are? It is only in the peaceful atmosphere of a university that these basic problems can be adequately considered.

It is only when the young men and women, who are in the university today and on whom the burden of life’s problems will fall tomorrow, learn to have clear objectives and standards of values that there is hope for the next generation. The past generation produced some great men but as a generation led the world repeatedly to disaster. World Wars IInd are the price that has been paid for the lack of wisdom on man’s part in this generation.

I think that there is always a close and intimate relationship between the end we aim at and the mean adopted to attain it.

Even, if the end is right, but the means are wrong, it will vitiate the end or divert us in a wrong direction. Means and ends are thus intimately and inextricably connected and cannot be separated.

That indeed has been the lesson of old taught us by many great men in the past, but unfortunately it seldom remembered.

36. People have little time to consider ideals and objectives because

(a)   they consider these ideals meaningless

(b)   they do not want to burden themselves with such ideas

(c)   they have no inclination for such things

(d)   they are excessively engaged in their routine activities

Ans: (d)

37. ‘The burden of life’s problems’ in the fourth sentence refers to

(a)   the incessant and feverish activities

(b)   the burden of family responsibilities

(c)   the onerous duties of life

(d)   the sorrows and sufferings

Ans: (c)

38. The world Wars IInd are the price that man paid due to

(a)   the absence of wisdom and sagacity

(b)   his not caring to consider the life’s problems

(c)   his ignoring the ideals and objectives of life

(d)   his excessive involvement in feverish activities.

Ans: (a)

39. According to the writer the adoption of wrong means even for the right end would

(a)   not let us attain our goal

(b)   bring us dishonour

(c)   impede our progress

(d)   deflect us from the right path

Ans: (d)

40. The word ‘vitiate’ used in the second paragraph means

(a)   negate

(b)   debase

(c)   tarnish

(d)   destroy

Ans: (b)

JIPMER Medical Entrance Exam Previous Year Question Paper 2014 With Answer Key

JIPMER

Medical Entrance Exam Previous Year – 2014

Physics

1. A ball is released from certain height which losses 50% of its kinetic energy on striking the ground, it will attain in height again

(a)   1/4th of initial height

(b)   1/2th of initial height

(c)   3/4th of initial height

(d)   None of the above

Ans: (b)

2. If applied torque on a system is zero, i.e., τ = 0, then for that system

(a)   ω = 0

(b)   α = 0

(c)   J = 0

(d)   F = 0

Ans: (b)

3. A steel wire of 1 m long and 1 mm2 cross section area is hanged from rigid end when weight of 1 kg is hang from it, then change in length will be

(Young’s coefficient for wire Y = 2 × 1011 N/m2)

(a)   0.5 mm

(b)   0.25 mm

(c)   0.05 mm

(d)   5 mm

Ans: (c)

4. When plate voltage in diode valve is increased from 100V to 150V. Then, plate increases from 7.5 mA to 12 mA, then dynamic plate resistance will be

(a)   10 kΩ

(b)   11 kΩ

(c)   15 kΩ

(d)   11.1 kΩ

Ans: (d)

5. If the sun rays are incidenting at 60° angle and intensity is I. If the sun rays are made incident at 30° angle, then what will be the intensity?

(a)    

(b)  

(c)    

(d)    

Ans: (b)

6. The reason of current flow in p-n junction in forward bias is

(a)   drifting of charge carriers

(b)   drifting of minority charge carriers

(c)   diffusion of charge carriers

(d)   All of the above

Ans: (c)

7. A charged particle is accelerated by a potential of 200 V. If its velocity is 8.4 × 108 m/s, then value of e/m for that particle is

(a)    

(b)    

(c)    

(d)    

Ans: (c)

8. When an open organ is dipped in water upto half of its height, then its frequency will becomes

(a)   half

(b)   double

(c)   remain same

(d)   four time

Ans: (c)

9. A sound source producing waves of frequency 300 Hz and wavelength 1 m observer is stationary, while source is going away with the velocity 30 m/s, then apparent frequency heared by the observer is

(a)   270 Hz

(b)   273 Hz

(c)   383 Hz

(d)   300 Hz

Ans: (b)

10. A particle moves towards east for 2s with velocity 15 m/s and move towards north for 8 s with velocity 5 m/s. Then, average velocity of the particle is

(a)   1 m/s

(b)   5 m/s

(c)   7 m/s

(d)   10 m/s

Ans: (b)

11. Relation between wavelength of photon and electron of same energy is

(a)    

(b)    

(c)    

(d)   

Ans: (a)

12. Match the following.

(a)   C – 2, D – 1

(b)   A – 4, B – 3

(c)   A – 3, C – 2

(d)   B – 2, A – 1

Ans: (b)

13. If we increase kinetic energy of a body 300%, then per cent increase in its momentum is

(a)   50%

(b)   300%

(c)   100%

(d)   150%

Ans: (c)

14. Change in acceleration due to gravity is same upto a height h from each other the earth surface and below depth x, then relation between x and h is (h and x < < < Re)

(a)   x = h

(b)   x = 2h

(c)   x = h/2

(d)  x = h2

Ans: (b)

15. A mass of 1 kg is suspended from spring of force constant 400 N, executing SHM total energy of the body is 2J, then maximum acceleration of the spring will be

(a) 4 m/s2

(b) 40 m/s2

(c) 200 m/s2

(d) 400 m/s2

Ans: (b)

16. Two capacitors of capacities C1 and C2 are charged upto the potential V1 and V2, then condition for not flowing the charge between on connected them in parallel is

(a)    

(b)   

(c)    

(d)    

Ans: (b)

17. Find equivalent resistance between X and Y

(a)   R

(b)   R/L

(c)   2R

(d)   5R

Ans: (a)

18. Vibrations of rope tied by two rigid ends shown by equation y = cos 2πt sin 2πx, then minimum length of the rope will be

(a)   1 m

(b)   1/2m

(c)   5 m

(d)   2 π m

Ans: (b)

19. If we change the value of R, then

(a)   voltage does not change on L

(b)   voltage does not change on LC combination

(c)   voltage does not change on C

(d)   voltage changes on LC combination

Ans: (d)

20. If V = ary, then electric field at appoint will be proportional to

(a)  r 

(b)  r−1 

(c)  r−2

(d)  r2

Ans: (d)

21. Electric field at point 20 cm away from the centre of dielectric sphere is 100 V/m, radius of sphere is 10 cm, then the value of electric filed at a distance 3 cm from the center is

(a)   100 V/m

(b)   125 V/m

(c)   120 V/m

(d)   0

Ans: (d)

22. 50 g ice at 0℃ in insulator vessel, 50 g water of 100℃ is mixed in it, then final temperature of the mixture is (neglect the heat loss)

(a)   10℃

(b)   

(c)   20℃

(d)   above 20℃

Ans: (a)

23. Real power consumption in a circuit is least when it contains.

(a)   high R, low L

(b)   high R, high L

(c)   low R, high L

(d)   high R, low C

Ans: (c)

24. Linear density of a string of is 1.3 × 104 kg/m and wave equation is y = 0.021 sin)x + 30t). Find, the tension in the string where, x in metre and t in second

(a)   0.12 N

(b)   0.21 N

(c)   1.2 N

(d)   0.012 N

Ans: (a)

25. Magnetic field at point O will be

(a)    

(b)    

(c)    

(d)    

Ans: (a)

26. If prism angle α = 1°, μ = 1.54, distance between screen and prism (D) = 0.7 m, distance between prism and source a = 0.3 m, λ = 180π nm, then in Fresnal biprism find the value of β(fringe width).

(a)    

(b)   

(c)    

(d)    

Ans: (b)

27. According to Bohr’s model of hydrogen atom, relation between principal quantum number n and radius of stable orbit is

(a)    

(b)    

(c)    

(d)    

Ans: (d)

28. An observer is approaching with velocity υ towards a light source. If the velocity of light is c, then velocity of light with respect to observer will be

(a)   c – v

(b)   c

(c)   c + v

(d)    

Ans: (c)

29. Magnetic field of the earth is H = 0.3 g. A magnet is vibrating 5 oscillations per min then the dippreciation required in the magnetic field of the earth of increase time period upto 10 oscillations per minute is

(a)   2.25 g

(b)   0.6 g

(c)   0.9 g

(d)   0.12 g

Ans: (a)

30. Work function of a metal is 5.2 × 1018, then its threshold wavelength will be

(a)   736.7 Å

(b)   760.7 Å

(c)   301 Å

(d)   344.4 Å

Ans: (d)

31. Remaining quantity (in%) of radioactive element after 5 half lives is

(a)   4.125%

(b)   3.125%

(c)   31.1%

(d)   42.125%

Ans: (a)

32. A engine pumps up 100 kg of water through a height of 10 m in 5s. Given that, the efficiency of engine is 60%. If g = 10 ms2, the power of this engine is

(a)   3.3 kW

(b)   0.33 kW

(c)   0.033 kW

(d)   33 kW

Ans: (a)

33. The angular amplitude of a simple pendulum is θ0. The maximum tension in its string will be

(a)    

(b)  

(c)    

(d)    

Ans: (d)

34. If the electric flux entering and leaving an enclosed surface respectively are ϕ, and ϕ2, the electric charge inside the surface will be

(a)    

(b)    

(c)    

(d)   

Ans: (d)

35. During an adiabatic process, the pressure of a gas is found to be proportional to the cube of absolute temperature. The ratio  for the gas is

(a)   4/3

(b)   2

(c)   5/3

(d)   3/2

Ans: (d)

36. Three points charges +q, −2q and +q are placed at points (x = 0, y = a, z = 0), (x = 0, y = 0, z = 0) and (x = a, y = 0, z = 0), respectively. The magnitude and direction of the electric dipole moment vector of this charge assembly are

(a)   √2q along + y direction

(b)   √2qa along the line joining points (x = 0, y = 0, z = 0) and (x = a, y = a, z = 0)

(c)   qa along the line joining points (x = 0, y = 0, z = 0) and (x = a, y = a, z = 0)

(d)   √2aq along + x direction

Ans: (b) 

37. A block B is pushed momentarily along a horizontal surface with an initial velocity υ. If μ is the coefficient of vlicling friction between B and the surface, block B will come to rest after a time

(a)   v/gμ

(b)   gμ/v

(c)   g/v

(d)   v/g

Ans: (a)

38. To get three images of a single object, we should have two plane mirrors at an angle of

(a)   60°

(b)   90°

(c)   120°

(d)   30°

Ans: (b)

39. A particle of mass M and charge Q moving with velocity v describes a circular path of radius R when subjected to a uniform transverse magnetic field of induction B. The work done by the field, when the particle completes one full circle, is

(a)    

(b)   zero

(c)   BQ2πR

(d)   BQv2πR

Ans: (b)

40. A particle of mass 100 g is thrown vertically upward with a speed of 5 m/s. The work done by the force of gravity during the time the particle goes up is

(a)   −0.5 J

(b)   −1.25 J

(c)   1.25 J

(d)   0.5 J

Ans: (b)

Chemistry

1. The relative reactivities of acyl compounds towards nucleophilic substitution are in the order of

(a)   acyl chloride > acid anhydride > ester > amide

(b)   ester > acyl chloride > amide > acid anhydride

(c)   acid anhydride > amide > ester > acyl chloride

(d)   acyl chloride > ester > acid anhydride > amide

Ans: (a)

2. Clemmensen reduction of a ketone is carried out in the presence of which of the following?

(a)   Zn-Hg with HCl

(b)   LiAlH4

(c)   H2 and Pt. as catalyst

(d)   Glycol with KOH

Ans: (a)

3. In Duma’s method of estimation of nitrogen, 0.35 g of an organic compound gave 55 mL of nitrogen collected at 300 K temperature and 715 mm pressure. The percentage composition of nitrogen in the compound would be

(Aqueous tension at 300 K = 15 mm)

(a)   16.45

(b)   17.45

(c)   14.45

(d)   15.45

Ans: (a)

4. Which of the following is not correct?

(a)   Ammonia is used as refrigerant

(b)   A mixture of Ca(CN)2 and C is known as nitrolim

(c)   A mixture of Ca(H2PO4)2 and CaSO4 ∙ 2H2O is known as superphosphate of lime

(d)   Hydrolysis of NCl3 gives NH3 and HOCl

Ans: (b)

5. Which one of the following is aromatic?

(a)   Cyclopentadienyl cation

(b)   Cyclooctatetraene

(c)   Cycloheptatriene

(d)   Cycloheptatrienylcation

Ans: (d)

6. The complexes [Co(NH3)6]C [Cr(CN)6] and [Cr(NH3)6] [Co(CN)6] are the examples of which type of isomerism?

(a)   Ionisation isomerism

(b)   Coordination isomerism

(c)   Geometrical isomerism

(d)   Linkage isomerism

Ans: (b)

7. A solution with negative deviation among the following is

(a)   ethanol-acetone

(b)   chlorobenzene-bromobenzene

(c)   chloroform-acetone

(d)   benzene-toluene

Ans: (c)

8. Which of the following compounds is soluble in benzene but almost insoluble in water?

(a) 

(b)   

(c)    

(d)   

Ans: (d)

9. The correct order of magnitude of bond angles among the compounds CH4, NH3 and H2O is

(a)    

(b)    

(c)    

(d)    

Ans: (b)

10. Which one of the following reactions does not form gaseous product?

(a)    

(b)    

(c)    

(d)  

Ans: (c)

11. Observe the following reaction

2A + B → C

The rate of formation of C is

2.2 × 10−3 mol L−1 min−1. What is the value of  (in mol L−1 min−1)

(a)    

(b)   

(c)    

(d)   

Ans: (c)

12. Which one of the following is employed as antithistamine?

(a)   Diphenyl hydramine

(b)   Norethndrone

(c)   Omeprazole

(d)   Chloramphenicol

Ans: (a)

13. If x is amount of adsorbate and m is amount of adsorbent, which of the following relations is not related to adsorption process?

(a)    

(b)   

(c)    

(d)    

Ans: (c)

14. Standard electrode potential of three metals X, Y and Z are −2 V, +0.5 V and −3.0 V respectively. The reducing power of these metals will be

(a)   Y > X > Z

(b)   Z > X > Y

(c)   X > Y > Z

(d)   Y > Z > X

Ans: (b)

15. Which of the following compounds has the lower melting point?

(a)    CaBr2

(b)    CaI2

(c)    CaF2

(d)    CaCl2

Ans: (b)

16. The dissociation equilibrium of a gas AB2 can be represented as

2AB2(g) ⇌ 2AB (g) + B2(g)

The degree of dissociation is ‘x’ and is small compared to 1. The expression relating the degree of dissociation (x) with equilibrium constant Kp and total pressure p is

(a)    

(b)    

(c)   

(d)   

Ans: (b)

17. A buffer solution is prepared in which the concentration of NH3 is 0.30 M and the concentration of NH4+ is 0.20 M. If the equilibrium constant, Kb for NH3 equals 1.8 × 105, what is the pH of this solution?

(log 2.7 = 0.43)

(a)   9.43

(b)   11.72

(c)   8.73

(d)   9.08

Ans: (a)

18. If a gas expands at constant temperature indicates that

(a)   kinetic energy of molecules decreases

(b)   pressure of the gas increases

(c)   kinetic energy of molecules remains as same

(d)   number of the molecules of gas increase

Ans: (c)

19. If n = 6, the correct sequence for filling of electrons will be

(a)   ns→(n – 1)d →(n – 2)f → np

(b)   ns → (n – 2) f → np → (n – 1)d

(c)   ns → np → (n – 1)d → (n – 2)f

(d)   ns → (n – 2)f → (n – 1)d → np

Ans: (d)

20. Which of the following is least likely to behave as Lewis base?

(a)    NH3

(b)    BF3

(c)    OH

(d)    H2O

Ans: (b)

21. In DNA, the complementary bases are

(a)   adenine and thymine; guanine and cytosine

(b)   adenine and thymine; guanine and uracil

(c)   adenine and guanine, thymine and cytosine

(d)   uracil and adenine; cytosine and guanine

Ans: (a)

22. Which of the following is not a resonating form of benzaldehyde?

(a) 

 

(b) 

 

(c) 

 

(d) 

 

Ans: (b)

23. Which one of the following does not undergo iodoform reaction?

(a)   Secondary butyl alcohol

(b)   Iso-propyl alcohol

(c)   Diethyl ketone

(d)   Ethyl alcohol

Ans: (c)

24. Percentage of free space in a body centre cubic unit cell is

(a)   30%

(b)   32%

(c)   34%

(d)   28%

Ans: (b)

25. For the reaction, N­2(g) + O2(g) ⇌ 2NO(g), the equilibrium constant is K1. The equilibrium constant is K2 for the reaction, 2NO(l) + O2(g) ⇌ 2NO2(g). What is K for the reaction,  

(a)    

(b)    

(c)    

(d)    

Ans: (b)

26. Acidified K2Cr2O7 solution turns green when Na2SO3 is added to it. This is due to the formation of

(a)    

(b)   

(c)    

(d)    

Ans: (d)

27. Among the following the least thermally stable is

(a)    

(b)   

(c)    

(d)   

Ans: (d)

28. Which one of the following is ‘d’-block element?

(a)   Gd

(b)   Hs

(c)   Es

(d)   Cs

Ans: (b)

29. The angular shape ozone molecule (O3) consists of

(a)   1 sigma and 2 pi bonds

(b)   2 sigma and 2 pi bonds

(c)   1 sigma and 1 pi bonds

(d)   2 sigma and 1 pi bonds

Ans: (d)

30. If the E°cell for a given reaction has a negative value then which of the following gives the correct relationships for the values of ∆G° and Keq?

(a)    

(b)    

(c)    

(d)    

Ans: (c)

31. Among the following the molecule possessing highest dipole moment is

(a)    

(b)    

(c)    

(d)     

Ans: (c)

32. An electron is moving in Bohr’s fourth orbit. Its de-Broglie wave length is λ. What is the circumference of the fourth orbit?

(a)   2/ λ

(b)   2λ

(c)   4λ

(d)   4/ λ

Ans: (c)

33. The standard enthalpies of formation of A(NH3), B(CO2), C(HI) and D(SO2) are −19, −393.4, +24.94 and −296.9 kJ mol1 respectively. The increasing order of their stability is

(a)   B < D < A < C

(b)   C < A < D < B

(c)   D < B < C < A

(d)   A < C < D < B

Ans: (b)

34. Green chemistry means such reactions which

(a)   produce colour during reactions

(b)   reduce the use and production of hazardous chemicals

(c)   are related to the depletion of ozone layer

(d)   study the reactions in plants

Ans: (b)

35. Which of the following compounds gives carbylamines test?

(a)   N-methyl-o-methyl aniline

(b)   N, n-dimethyl aniline

(c)   2 4-diethyl aniline

(d)   p-methyl-N-methyl benzylamine

Ans: (c)

36. Which of the following is an example for heterogeneous catalysis reaction?

(a)    

(b)   Hydrolysis of aqueous sucrose solution in the presence of a aqueous mineral acid

(c)    

(d)   Hydrolysis of liquid in the presence of aqueous mineral acid

Ans: (c)

37. Which of the following pair of metals is purified by van-Arkel method?

(a)   Zr and Ti

(b)   Ag and Au

(c)   Ni and Fe

(d)   Ga and ln

Ans: (a)

38. Name the type of the structure of silicate in which one oxygen atom of [SiO4]4 is shared?

(a)   Sheet silicate

(b)   Pyrosilicate

(c)   Three dimensional silicate

(d)   Linear chain silicate

Ans: (b)

39. In Fischer-Ringe’s method of separation of nobles gas mixture from air, …… is used

(a) 

(b)   coconut charcol

(c)   soda lime + potash solution

(d)    

Ans: (a)

40. Which one of the following is present as an active ingredient in bleaching powder for bleaching action?

(a)    

(b)    

(c)    

(d)    

Ans: (a)

Zoology

1. The most recent and direct pre-historic ancestor is

(a)   cro-magnon

(b)   neanderthal

(c)   pre-neaderthal

(d)   None of these

Ans: (a)

2. Rh-factor can produce disease

(a)   AIDS

(b)   Tumer’s syndrome

(c)   Erythroblastosis foetalis

(d)   Sickle cell anaemia

Ans: (c)

3. Penumotoxic centre which an moderate the function of the respiratory rhythm centre is present at

(a)   pons region in brain

(b)   thalamus

(c)   spinal cord

(d)   right cerebral hemisphere

Ans: (a)

4. Which type of cells are absent in sponges?

(a)   Trophocytes

(b)   Myocytes

(c)   Archeocytes

(d)   Cnidocytes

Ans: (d)

5. Zero growth means

(a)   natality balance mortality

(b)   natallity is more than mortality

(c)   natality is less than mortlity

(d)   natality is zero

Ans: (a)

6. Which of the following hormone secretes a gastric secretion?

(a)   Entero gastrone

(b)   Gastrin

(c)   CCK-PZ

(d)   Villikinin

Ans: (b)

7. The protein α-1 antitrypsin is used to treat which of the following disease?

(a)   Cancer

(b)   Rhematoid arthritis

(c)   Emphysema

(d)   ADA deficiency disease in children

Ans: (c)

8. Darwins finches represent

(a)   morphological variations

(b)   geographical isolations

(c)   climate variations

(d)   reproductive isolation

Ans: (b)

9. Food storage in Leucosolenia occur by

(a)   ostia

(b)   osculam

(c)   thesocyte

(d)   spongocoel

Ans: (c)

10. Hardy-Weinberg equilibrium is known to be affected by gene flow, genetic drift, mutation, genetic recombination and

(a)   evolution

(b)   limiting factors

(c)   saltation

(d)   natural selection

Ans: (d)

11. Which one of the following in birds indicates their reptilian ancestry?

(a)   Scales on their hindlimbs

(b)   Four-chambered heart

(c)   Two special chambers crop and gizzards in their digestive tract 

(d)   Egg with a calcareous shell

Ans: (d)

12. Darwinsim explains all the following except

(a)   within each species, there are variations

(b)   organisms tend to produce more number of offspring that can survive

(c)   offspring with better traits that overcome competition are best suited for the environment

(d)   variations are inherited from parents to offspring through genes

Ans: (d)

13. Which of the following disorder is an outcome of irregularities in metabolism of the nitrogenous waste?

(a)   Osteoporosis

(b)   Gouty arthritis

(c)   Osteroarthritis

(d)   Rehumatoid arthritis

Ans: (b)

14. During urine formation, which of the following process create thigh osmotic pressure in the uriniferous tubule?

(a)   Active Na+ absorption, followed by absorption of Cl

(b)   Active Cl absorption, followed by absorption of Na+

(c)   Active secretion of Na+ into efferent arteriole followed by absorption of Cl into efferent renal arteriole

(d)   Active secretion of Cl and absorption of Na+ into efferent renal arteriole

Ans: (a)

15. Steroid hormones work as

(a)   they enter into target cells and binds with specific receptor and ativate4s specific genes to form protein

(b)   they binds to cell membrane

(c)   they catalyse formation of cAMP

(d)   None of the above

Ans: (b)

16. In the following diagram showing axon terminal and synapse A, B, C, D and E respectively represents

(a)   A-axon terminal, B-synaptic cleft, C-synaptic vesicles, D-neurotransmitters, E-receptors

(b)   A- axon terminal, B-synaptic vesicles, C-synaptic cleft, D-receptors, E-neurotransmitters

(c)   A-synaptic cleft, B-synaptic vesicles, C-axon terminal, D-neurotransmitters, E-receptors

(d)   A-synaptic vesicles, B-axon terminal, C-synaptic vesicles, D-neurotransmitters, E-receptors

(e)   A-synaptic vesicles, B-axon terminal, C-synaptic cleft, D-receptors, E-neurotransmitters

Ans: (b)

17. Adrenaline and noradrenaline are hormones that act as

(a)   energy producing agents

(b)   food storage materials

(c)   neurotransmitters

(d)   energy storing substances

Ans: (c)

18. If due to some injury the chordate tendinae of the tricuspid valve of the human heart is partially non-functional, what will be the immediate effect?

(a)   The flow of blood into the aorta will be slowed down

(b)   The ‘pace maker’ will stop working

(c)   The blood will tend to flow back into the left atrium

(d)   The flow of blood into the pulmonary artery will be reduced

Ans: (a)

19. The vector for T-DNA is

(a)   Thermus aquaticus

(b)   Salmonella typhimurium

(c)   Agrobacterium tumefaciens

(d)   Escherichia coli

Ans: (c)

20. Match the enzyme in column I with its function in column II and choose the correct option.

(a)   A-2, B-1, C-4, D-3

(b)   A-3, B-2, C-1, D-4

(c)   A-2, B-4, C-1, D-3

(d)   A-1, B-2, C-4, D-3

Ans: (b)

21. In the central nervous system, myelinated fibres form the ….., while the non-myelinated fibre cells form the ……

(a)   grey matter, white matter

(b)   white matter, grey matter

(c)   ependymal cells, neurosecretory cells

(d)   neurosecretory cell, ependymal cells

Ans: (b)

22. Haemoglobin is

(a)   an oxygen carrier in human blood

(b)   a protein used as food supplement

(c)   an oxygen scavenger in root nodules

(d)   a p lant protein with high lysine content

Ans: (a)

23. Which of the following tissue is a vascular?

(a)   Cowpait bone

(b)   Skeletal muscle

(c)   Stratified squamous epithelium

(d)   Adipose tissue

Ans: (c)

24. Many elements are found in living organisms either free or in form of compounds. One of the following is negligible is living organisms

(a)   silicon

(b)   magnesium

(c)   iron

(d)   sodium

Ans: (a)

25. Which of the following represent uridylic acid?

(a)   Uracil + Ribose

(b)   Uridine + Phosphoric acid

(c)   Uracil + Phosphoric acid

(d)   Uridine + Ribose + Phosphoric acid

Ans: (d)

26. Consider the following statements with two blanks X and Y and select the option which correctly fills up these blanks

In the centre of the inter vertebrel disc, a soft area is present called ….X….. .

Which is supposed to be remnant of …. Y.. .

(a)   X-Nucleus pulposus ,         Y-Nerve cord

(b)   X-Centrum                ,         Y-Noto cord        

(c)   X-Nucleus pulposus ,         Y-Noto cord

(d)   X-Centrum                ,         Y-Nerve cord

Ans: (c)

27. Six highly chitinous plate-like teeth are found in ……. of cockroach

(a)   mandible

(b)   gizzard

(c)   mexilla

(d)   rectum

Ans: (b)

28. The statement All biological catalysts are protein is no longer valid after the discovery of

(a)   ribonuclease

(b)   ribozyme

(c)   RNAs

(d)   DNAs

Ans: (b)

29. The structure present in all adult vertebrates is

(a)   notochord

(b)   dorsal tubular nerve cord

(c)   pharyngeal gill slits

(d)   All of these

Ans: (b)

30. Which one of the following is not a vertebrate?

(a)   Sea cow

(b)   Sea lion

(c)   Sea horse

(d)   Sea hare

Ans: (d)

31. In human during fertilization a sperm firstly comes in contact with which layer of ovum

(a)   zona pellucida

(b)   vitelline membrane

(c)   jelly coat

(d)   zona radiata

Ans: (d)

32. Newly formed Biosphere Reserve of India (2013) is

(a)   Pachmarhi

(b)   Himalaya

(c)   Cold desert

(d)   Panna

Ans: (d)

33. Which statement is correct for bacterial transduction?

(a)   Transfer of some genes from one bacteria to another bacteria through virus

(b)   Transfer of genes from one bacteria to another bacteria by conjugation.

(c)   Bacteria obtain DNA directly

(d)   Bacteria obtain DNA from other external source

Ans: (a)

34. Theory of pangenesis was given by

(a)   Darwin

(b)   Lamarck

(c)   Hugo de Vries

(d)   Oparin

Ans: (a)

35. Which part of animal virus is not produced in multiple copies?

(a)   Capsid

(b)   Proteins

(c)   Envelops

(d)   Ribosome

Ans: (c)

36. Viroids have

(a)   ssRNA not enclosed by protein coat

(b)   ssDNA not enclosed by protein coat

(c)   dsDNA enclosed by protein coat

(d)   dsRNA enclosed by protein coat

Ans: (a)

37. Biotic potential or potential natality means

(a)   natural increase of population under ideal/optimum conditions

(b)   potential of organism in a biome

(c)   number of organisms in a biome

(d)   species of maximum number in a population

Ans: (a)

38. The world’s highly prized wool yielding ‘Pashmina’ breed is

(a)   sheep

(b)   goat

(c)   goat-sheep cross

(d)   Kashmiri sheep-Afghan sheep cross

Ans: (b)

39. Probes, used in DNA fingerprinting, are initially

(a)   single stranded RNA

(b)   mini-satellite

(c)   19base long oligonucleotide

(d)   All of the above

Ans: (b)

40. Satellite DNA is useful tool in

(a)   organ transplantation

(b)   sex determination

(c)   forensic science

(d)   genetic engineering

Ans: (c)

Botany

1. Which of the following is non-symbiotic nitrogen fixer organism?

(a)   VAM

(b)   Azotobacter

(c)   Anabaena

(d)   Rhizobium

Ans: (b)

2. Aleurone grains are rich in

(a)   fat

(b)   protein

(c)   carbohydrates

(d)   auxins

Ans: (b)

3. Match the following columns and choose the correct option.

(a)   A-5, B-4, C-1, D-2

(b)   A-5, B-4, C-1, D-3

(c)   A-4, B-2, C-3, D-1

(d)   A-1, B-2, C-4, D-5

Ans: (b)

4. The rupture and fractionation do not usually occur in water column in vessel/trachieds during the ascent of sap because of

(a)   lignified thick walls

(b)   cohesion and adhesion

(c)   weak grauitational pull

(d)   transpiration pull

Ans: (b)

5. Stability of ecosystem depends upon

(a)   primary productivity

(b)   interchange between producers and consumers

(c)   number of producers

(d)   number of consumers

Ans: (b)

6. The material, which arrests cell division, is obtained from

(a)   Crocus

(b)   Colchicum

(c)   Dalbergis

(d)   Chysanthomum

Ans: (b)

7. Viscum album grown on trees. This is an example of

(a)   symbiosis

(b)   parasitism

(c)   commensalism

(d)   predation

Ans: (b)

8. Which of the following is responsible for biological nitrogen-fixation?

(a)   Nife gene

(b)   Nitrogenase

(c)   Yeast alanin tRNA synthetase

(d)   RNA synthetase

Ans: (a)

9. The respiratory quotient during cellular respiration would depend on the

(a)   nature of enzymes involved

(b)   nature of the substrate

(c)   amount of carbon dioxide released

(d)   amount of oxygen utilized

Ans: (b)

10. Which of the following supports a dense population of plankton and littoral vegetation?

(a)   Oligotrophic

(b)   Eutrophic

(c)   Lithotrophic

(d)   Agroecotrophic

Ans: (b)

11. Which of the following is not true for a species?

(a)   Members of a species can interbreed

(b)   Variations occur among members of species

(c)   Each species is reproductively isolated from every other species

(d)   Gene flow does not occur between the populations of a species

Ans: (d)

12. Given below is the chemical formula of

(a)   palmitic acid

(b)   glycerol

(c)   galactose

(d)   stearic acid

Ans: (a)

13. Choose the wrong pair

(a)   Cenchrus-Savanna

(b)   Abies – Coniferous forest

(c)   Quercus – Broad leaf forest

(d)   Tectona – Temperate forest

Ans: (d)

14. Rarely among angiosperms, the pollen grains influenced the endosperm. This is called as

(a)   meta xenia

(b)   nemec phenomenon

(c)   xenia

(d)   mesogamy

Ans: (c)

15. Three floral diagrams are given here. Their respective families are assigned in the answer key. Find out the families to which these diagrams belong to

(a)   A-Liliaceae, B-Asteraceae, C-Solanaceae

(b)   A-Asteraceae, B-Solanaceae, C-BRassicaceae

(c)   A-Asteraceae, B-Solanaceae, C-Asteraceae

(d)   A-Poaceae, B-Solanaceae, C-Asteraceae

Ans: (b)

16. Tyloses an out growth from ray or axial parenchyma cell into the lumen of a vessels, which partially or completely blocks the cavity are present in

(a)   periderm

(b)   heart wood

(c)   sap wood

(d)   secondary cortex

Ans: (b)

17. Overlapping region between two ecosystem is called

(a)   biome

(b)   ecotone

(c)   niche

(d)   photic zone

Ans: (b)

18. Phage genome site on bacterial chromosome resulted in the structure

(a)   nucleic acid

(b)   heterocyst

(c)   prophage

(d)   None of these

Ans: (c)

19. ‘Sun basket’ is

(a)   the device to utilize sun rays directly to meet the requirement of heat energy

(b)   the sufficient amount of sunlight stored in a cell

(c)   a device of taking sunbath

(d)   All of the above

Ans: (b)

20. The dominant epistasis ratio is

(a)   9:3: 3:1

(b)   12:3:1

(c)   9:3:4

(d)   9:6:1

Ans: (b)

21. Benthic organisms are found in

(a)   surface of marine water

(b)   middle of water in sea

(c)   bottom of sea

(d)   on ground

Ans: (c)

22.  Ethylene is related with

(a)   aerobic respiration

(b)   climacteric phenomenon

(c)   anaerobic respiration

(d)   fermentation

Ans: (b)

23. Select the correct statement from the one’s given below with respect to dihybrid cross.

(a)   tightly linked gene on the same chromosome show higher recombinations

(b)   genes for apart on same chromosomes show very few recombination

(c)   genes loosely linked on the same chromosomes show similar recombination as lightly linked ones

(d)   tightly linked genes on the same chromosomes show very few recombination

Ans: (d)

24. Maximum green house gases are released by

(a)   India

(b)   Britain

(c)   USA

(d)   France

Ans: (c)

25. In a cola fired power plant, electrostatic precipitators are installed to control emission of

(a)   SO2

(b)   NOx

(c)   SPM

(d)   CO

Ans: (c)

26. Tow genes R and y are located very close on the chromosome linkage map on maize plant. When RRYY and rryy genotypes are hybridized, then F2 segregation will show

(a)   higher number of the recombinant types

(b)   Segregation with expected 9:3:3:1 ratio

(c)   Segregation is 3:1 ratio

(d)   higher number of the parental types

Ans: (d)

27. A lake with an inflow of domestic sewage rich in organic waste may result in

(a)   drying of the lake very soon due to algal bloom

(b)   an increased production of fish due to lot of nutrients

(c)   death of fish due to lack of oxygen

(d)   increased population of aquatic food web organisms

Ans: (c)

28. Allelic sequence variation where more than one variant allele at a locus in a human population with a frequency greater than 0.01 is referred to as

(a)   DNA polymorphism

(b)   multiple allelism

(c)   SNP

(d)   EST

Ans: (a)

29. Besides dung, the weed that can be used in biogas production is

(a)   Hydrilla

(b)   Solanum- nigrum

(c)   Eichhornia Crassipes

(d)   Parthenium hysterophorus

Ans: (c)

30. The following is the diagram of TS of anther. Identify the parts labeled as A, B and C.

(a)   A-Connective, B-Endothecium, C-Pollen grain

(b)   A-Endothecium, B-Connective, C-Pollen grain

(c)   A-Pollen grain, B-Connective, C-Endothecium

(d)   A-Endothecium, B-Pollen grain, C-Connective

Ans: (a)

31. Bacillus thuringiensis (Bt) strains have been used for designing novel

(a)   bio metallurgical technique

(b)   bio mineralization process

(c)   bio insecticidal plants

(d)   bio fertilizers

Ans: (c)

32. 

In the given diagram, parts labeled as A, B, C, D,  E and F are respectively indentified as

(a)   synergids, polar nuclei, central cell, antipodals, filiform apparatus and egg

(b)   polar nuclei, egg, antipodals, central cell, filiform apparatus and synergids

(c)   egg, synergids, central cell, filiform apparatus, antipodals and polar nuclei

(d)   central cell, polar nuclei filiform apparatus, antipodals, synergids and egg

Ans: (a)

33. In some organisms, karyokinesis is not followed by cytokinesis as a result of which, multinucleate condition arises leading to the formation of syncytium. The perfect example for this is

(a)   appearance of a furrow in cell membrane

(b)   liquid endosperm in coconut

(c)   sexual reproduction

(d)   fertilization

Ans: (b)

34. Jacob and Monod name some enzymes as allosteric whose activity is regulated by

(a)   end product

(b)   substrate

(c)   by product

(d)   coenzyme

Ans: (a)

35. The mobile genetic element is

(a)   transposon

(b)   mutation

(c)   endonuclease

(d)   variation

Ans: (a)

36. In recombinant DNA technique, the term vector refers to

(a)   donor DNA, is identified and picked up through electrophoresis

(b)   plasmid, transfers DNA into living cell

(c)   collection of entire genome in form of plasmid

(d)   enzyme, cuts the DNA at specific sites

Ans: (b)

37. The net requirement of assimilatory power for the formation of 6 hexose molecules in maize plant is

(a)   72 ATP, 48 NADPH

(b)   90 ATP, 60 NADPH

(c)   108 ATP, 72 NADPH

(d)   180 ATP, 72 NADPH

Ans: (d)

38. When two unrelated individuals or lines are crossed, the performance of F1 hybrid in often superior to both of its parents. This phenomenon is called

(a)   transformation

(b)   heterosis

(c)   splicing

(d)   meta morphosis

Ans: (b)

39. Given below is a sample of portion of DNA strand given the base sequence on the opposite strand. What is so special show in it?

5’−GAATTC….3’

3’−CTTAAG … 5’

(a)   Detection mutation

(b)   Start codon at 5′ end

(c)   Palindromic sequence of base pairs

(d)   Replication completed

Ans: (c)

40. Rauwolffia serpentina is used in

(a)   curing high blood pressure

(b)   kidney failure

(c)   eye defect

(d)   diabetes

Ans: (a)

English

Directions (Q. Nos. 1-5) Choose the word/group of words which is the most nearly SAME in meaning as the word/group of words given in bold as used in the passage.

1. SINCERE

(a)   Open

(b)   Earnest

(c)   Dissolute

(d)   Upright

Ans: (d)

2. ENCOURAGE

(a)   Animate

(b)   Urge

(c)   Stimulate

(d)   Dissuade

Ans: ()

3. SELF-SUFFICIENT

(a)   Adequate

(b)   Dependent

(c)   Overflowing

(d)   Self-reliant

Ans: (d)

4. FOCUS

(a)   Adjust

(b)   Concentrate

(c)   Meditate

(d)   Circulate

Ans: (b)

5. SUPERFICIAL

(a)   Artificial

(b)   Shallow

(c)   Complete

(d)   Profound

Ans: (b)

Directions (Q. Nos. 6-15) Pick out the most effective word/phrase from the given words to fill in the blank to make the sentence meaningfully complete.

6. He knelt at his side and comforted him with ……. words.

(a)   harsh

(b)   silent

(c)   kind

(d)   cruel

Ans: (c)

7. A man who is perpetually …… which of the two things he will do first, will do neither,

(a)   confused

(b)   forced

(c)   orders

(d)   hesitating

Ans: (d)

8. We cannot ……. of life without suitable environment.

(a)   live

(b)   buy

(c)   extract

(d)   think

Ans: (d)

9. He thought the boy ……. to benefit the blacksmith.

(a)   wants

(b)   wanting

(c)   desire

(d)   harm

Ans: (a)

10. The fisherman gladly ……. up the baby and took it home.

(a)   loaded

(b)   picked

(c)   dragged

(d)   pushed

Ans: (b)

Directions (Q. Nos. 11-15) Read each sentence to find out whether there is any grammatical error or idiomatic error in it. The error, if any, will be in one part of the sentence. The number of that part is the answer. If there is no error, the answer is (d) i.e., ‘No error (Ignore errors of punctuation, if any).

11. He is neither in (a) favour of arms race or in favour of (b)/simple nuclear disarmament. (c)/ No error (d)

(a)   a

(b)   b

(c)   c

(d)   d

Ans: (a)

12. Naturalization is the process by which (a)/ a immigrant becomes a citizen (b)/ of his new country. (c)/ No error (d)

(a)   a

(b)   b

(c)   c

(d)   d

Ans: (b)

13. A high fat diet not only increases the risks (a)/ of heart ailments (b) / however also that of other disorders. (c)/ No error (d)

(a)   a

(b)   b

(c)   c

(d)   d

Ans: (a)

14. When two vowel (a) / sounds occurs in direct succession, (b)/ the transition between them is often difficult to make. (c)/ No error (d)

(a)   a

(b)   b

(c)   c

(d)   d

Ans: (b)

15. The solutions is to avoid the impasse altogether, (a)/ by taking a (b) / slightly different route. (c) / No error (d)

(a)   a

(b)   b

(c)   c

(d)   d

Ans: (a)

Directions (Q. Nos. 16-20) Rearrange the following sentences to make a meaningful paragraph and then answer the questions given below them.

A. A classroom discussion can be initiated in order to answer this very question.

B. An electric current could not be made to traverse distilled water

C. Yet when salt and distilled water were mixed, then the solution became a liquid through which electricity could pass with ease.

D. Neither would solid salt offer free passage to electricity.

E. How could one explain this strange behavior of solution.

F. And, as the current passed through this solution, a deep seated decomposition took place.

16. Which of the following will be the FIFTH sentences?

(a)   A

(b)   C

(c)   B

(d)   E

Ans: (d)

17. Which of the following will be the FOURTH sentence?

(a)   A

(b)   B

(c)   F

(d)   E

Ans: (c)

18. Which of the following will be the LAST (SIXTH) sentence?

(a)   C

(b)   A

(c)   B

(d)   E

Ans: (b)

19. Which of the following will be the THIRD sentence?

(a)   C

(b)   D

(c)   F

(d)   A

Ans: (a)

20. Which of the following will be the FIRST sentence?

(a)   A

(b)   D

(c)   B

(d)   C

Ans: (c)

Directions (Q. Nos. 21-25) Which of the phrases (a), (b) and (c) given below each sentence should replace the phrase printed in bold in the sentence to make it grammatically correct? If the sentence is correct as is given and no correction is required, mark (d) as the answer

21. Your doctor may explain the importance of a proper and balanced diet in the human body.

(a)   an proper and balanced

(b)   a proper or balance

(c)   a prosperous and balance

(d)   No correction required

Ans: (d)

22. English today is closer to been a world language than any other language has been in history.

(a)   is closer for been

(b)   is closer upon being

(c)   is closer to being

(d)   No correction required

Ans: (c)

23. In almost every occupation on needs simple a understanding of electricity.

(a)   needed simple an

(b)   need a simple

(c)   needs a simple

(d)   No correction required

Ans: (c)

24. When ice and water existed together on the same volume, the temperature remains constant.

(a)   exist together in

(b)   will exist together on

(c)   existed together in

(d)   No correction required

Ans: (a)

25. Scientific method as a rote item in the syllabus had little valued.

(a)   had belittled value

(b)   has little value

(c)   have little value

(d)   No correction required

Ans: (b)

Directions (Q. Nos. 26-30) In each sentence below three words have been printed in bold which are number (a), (b) and (c). One of these words may be misspelt or inappropriate in the context of the sentence. Find out the wrongly spelt or inappropriate word. The number of that word is the answer. If all the words are correctly spelt and are appropriate the answer is (d) i.e., all correct

26. The importanse (a) given to content-oriented approach has affected the methodology (b) of this project. (c) All correct (d)

(a)   a

(b)   b

(c)   c

(d)   d

Ans: (a)

27. Almost all risk-taking (a) work involve (b) decision making under uncertainty. (c) All correct (d)

(a)   a

(b)   b

(c)   c

(d)   d

Ans: (d)

28. In developing (a) countries there is increasing (b) concern for fostering human potential. (c) All correct (d)

(a)   a

(b)   b

(c)   c

(d)   d

Ans: (b)

29. I want to express (a) my appreciation (b) of the help offered by my former colleages. (c) All correct (d)

(a)   a

(b)   b

(c)   c

(d)   d

Ans: (c)

30. The research (a) reported in this valume (b) assumes importance. (c) All correct (d).

(a)   a

(b)   b

(c)   c

(d)   d

Ans: (b)

Directions (Q. Nos. 31-40) In the following passage there are blanks, each of which has been numbered. These numbers are printed below the passage and against each, four words are suggested, one of which fits the blank appropriately. Find out the appropriate word in each case.

Working under the psychometric approach, both scientists and practitioners have placed undue emphasis upon a unitary concept of intelligence as reflected in the single I.Q. They seem to have ……(31)…… too much attention of the  products of intelligent behavior rather than the processes used to acquire …..(32)……products. Such attention to product rather than to process tends to mark qualitative differences in the processes by …..(33)…. individuals interact with their environment and to …..(34)…. attention away from the possibility of qualitative changes in the nature of these processes ….(35)… the curse of cognitive development.

Haywood points out that there is no such thing as intelligence. There are in fact many intelligences. There are in fact many intelligences. Factor analysts, ….(36)…. have studied the nature of intellect by examining its apparent structure across different age groups and different segments of the population have to a great …..(37)…. discredited the unitary concept of intelligence. Even so, the structure theories of intelligence are …..(38)…. concerned with products and not with processes.

Development (Cognitive Development) approach. It ….(39)…. upon the cognitive processes used to receive, code and …..(40)…. Information.

31. (a) gives         (b) gave

       (c) forced       (d) given

Ans: (d)

32. (a) those    (b) these

      (c) that        (d) raw

Ans: (b)

33. (a) how         (b) way

     (c) speech      (d) which

Ans: (d)

34. (a) draw         (b) seek

      (c) force         (d) drag

Ans: (d)

35. (a) at            (b) plan

      (c) during      (d) follow

Ans: (c)

36. (a) who       (b) should

    (c) might        (d) will

Ans: (a)

37. (a) meaning  (b) person

      (c) extend      (d) extent

Ans: (d)

38. (a) uptill   (b) till

      (c) until    (d) still

Ans: (d)

39. (a) focus (b) focuses

      (c) jumps (d) emphasise

Ans: (b)

40. (a) pack   (b) transmits

      (c) store   (d) stocked

Ans: (c)

 

JIPMER Medical Entrance Exam Previous Year Question Paper 2015 With Answer Key

JIPMER

Medical Entrance Exam Previous Year – 2015

Physics

1. The angle between two linear trans-membrane domains is defined by following vectors 

(a) 

(b) 

(c) 

(d) 

Ans: (b)

2. The distance x(in μm) covered by a molecule starting from point A at time t = 0 and stopping at another point B in given by the equation 

The distance between A and B (in μm) is closed to

(a)   10.7

(b)   20.7

(c)   40.7

(d)   50.7

Ans: (a)

3. A tangential force acting on the top of sphere of mass m kept on a rough horizontal place as shown in figure.

 

If the sphere rolls without slipping, then the acceleration with which the centre of sphere moves, is

(a) 

(b) 

(c) 

(d) 

Ans: (a)

4. The density of a rod having length l varies as ρ = c + dx, where x is the distance from the left end. The centre of mass is

(a) 

(b) 

(c) 

(d) 

Ans: (a)

5. One end of a massless spring of constant 100 N/m and natural length 0.5 m is fixed and the other end is connected to a p article of mass 0.5 kg lying on a frictionless horizontal table. The spring remains horizontal. If the mass is made to rotate at angular velocity of 2 rad/s, then elongation of spring is

(a)   0.1 m

(b)   10 cm

(c)   1 cm

(d)   0.01 cm

Ans: (c)

6. A block slides down on an incline of angle 30° with an acceleration g/4. Find the kinetic friction coefficient.

(a) 

(b) 

(c) 

(d) 

Ans: (c)

7. Two long straight wires, each carrying an electric current of 5 A, are kept parallel to each other at a separation of 2.5 cm. Find the magnitude of the magnetic force experiment by 10 cm of a wire.

(a)   4.0 × 104 N

(b)   3.5 × 106 N

(c)   2.0 × 105 N

(d)   2.0 × 109 N

Ans: (c)

8. A wire of resistance 10 Ω is bent to form a complete circle. Find its resistance between two diametrically opposite point.

(a)   5 Ω

(b)   2.5 Ω

(c)  1.25 Ω

(d) 

Ans: (b)

9. Find the resistance of a hollow cylindrical conductor of length 1.0 mm and 2.0 mm respectively. The resistivity of the material is 2.0 × 10−8 Ωm.

(a)   2.1 × 103 Ω

(b)   1.3 × 104 Ω

(c)   3.2 × 104 Ω

(d)   4.6 × 102 Ω

Ans: (a)

10. Three equal charges, each having a magnitude of 2.0 × 106 C, are placed at the three corners of a right angled triangle of sides 3 cm 4 cm and 5 cm. The force (in magnitude) on the charge at the right

(a)   50 N

(b)   26 N

(c)   29 N

(d)   45.9 N

Ans: (d)

11. A diatomic gas (γ = 1.4) does 200 J of work when it is expanded isobarically. Find the heat given to the gas in the process.

(a)   500 J

(b)   700 J

(c)   600 J

(d)   900 J

Ans: (b)

12. A uniform ring of mass m and radius a is  placed directly above a uniform sphere of mass m and of equal to radius. The centre of the ring is at a distance √3a from the centre of the sphere. The gravitational force (F) exerted by the sphere on the ring is

(a) 

(b) 

(c) 

(d) 

Ans: (a)

13. A projectile is fired with a velocity u at angle θ with the ground surface. During the motion at any time it is making an angle α with the ground surface. The speed of particle at this time will be

(a)   u cos θ sec α

(b)   u cos θ ∙ tan α

(c)   u2 cos2 α sin2 α

(d)   u sin θ ∙ sin α

Ans: (a)

14. The earth receives solar radiation at a rate of 8.2 J cm2 min1. If the sun radiates as the black bodies, the temperature at the surface of the sun will be (the angle subtended by sun on the earth in suppose 0.53° and Stefan constant is σ = 5.67 × 108 Wm2 K4)

(a)   5800 K

(b)   6700 K

(c)   8000 K

(d)   7800 K

Ans: (a)

15. The rms speed (in m/s) of oxygen molecules of the gas at temperature 300 K, is

(a)   483

(b)   504

(c)   377

(d)   346

Ans: (a)

16. A horizontal tube of length l closed at both ends, contains an ideal gas of molecular weight M. The tube is rotated at a constant angular velocity ω about a vertical axis passing through an end. Assuming the temperature to be uniform and constant. If p1 and p2 denote the pressure at free and the fixed end respectively, then choose the correct relation.

(a) 

(b) 

(c) 

(d) 

Ans: (a)

17. The parts of two concentric circular arcs joined by two radial lines and carries current i. The arcs subtend an angle θ at the centre of the circle. The magnetic field at the centre O, is

(a) 

(b) 

(c) 

(d) 

Ans: (a)

18. 1 kg of water is converted into steam at the same temperature and at 1 atm (100 kPa). The density of water and steam are 1000 kgm3 and 0.6 kgm3 The latent heat of vaporization of water is 2.25 × 106 J kg1, What will be increase in energy?

(a)   3 × 105 J

(b)   4 × 106 J

(c)   2.08 × 106 J

(d)   None of these

Ans: (c)

19. The ammeter shown in figure consists of a 480 Ω coil connected in parallel to a 20Ω shunt. The reading of ammeter is

(a)   0.125 A

(b)   1.67 A

(c)   0.13 A

(d)   0.67 A

Ans: (a)

20. A lead ball at 30℃ is dropped from a height of 6.2 km. The ball is  heated due to the air resistance and it completely melts just before reaching the ground. The molten substance falls slowly on the ground. If the specific heat of lead = 126 Jkg−11 and melting point of lead = 130℃ and suppose that any mechanical energy lost is used to heat the ball, then the latent heat of fusion of lead is

(a)   2.4 × 104 J kg1

(b)   3.6 × 104 J kg1

(c)   7.6 × 102 J kg1

(d)   4.2 × 103 J kg1

Ans: (a)

21. A inductor (L- 20 H), a resistor (R = 100 Ω) and a battery (E = 10 V) are connected in series. After a long time, the circuit is short-circuited and then the battery is disconnected. Find the current in the circuit at 1 ms after short circuiting.

(a)   4.5 × 105 A

(b)   3.2 × 105 A

(c)   9.8 × 105 A

(d)   6.7 × 104 A

Ans: (d)

22. Two charges of +10μC and +20μC are separated by a distance 2 cm. The net potential (electric) due to the pair at the middle point of the line joining the two changes, is

(a)   27 MV

(b)   18 MV

(c)   20 MV

(d)   23 MV

Ans: (a)

23. A copper rod of length 20 cm and cross-sectional area 2 mm2 is joined with a similar aluminium rod as shown below

The resistance of pair of rods is

Al = 2.6 × 108 Ω-m and ρCu = 1.7 × 108 Ω-m)

(a)   1.0 mΩ

(b)   2.0 mΩ

(c)   3.0 mΩ

(d)   None of these

Ans: (a)

24. A particle is subjected to two simple harmonic motions along X-axis while other is along a line making angle 45° with the X-axis. The two motions are given by x = x0 sin ωt and s = s0 sin ω

The amplitude of resultant motion is

(a) 

(b) 

(c) 

(d) 

Ans: (d)

25. What is the change in the volume of 1.0 L kerosene, when it is subjected to an extra pressure of 2.0 × 105 Nm−2 from the following data? Density of kerosene = 800 kg m−3 and speed of sound in kerosene = 1330 ms−1.

(a)   0.97 cm3

(b)   0.66 cm3

(c)   0.15 cm3

(d)   0.59 cm3

Ans: (c)

26. A 4 kg block is suspended from the ceiling of an elevator through a spring having a linear mass density of 19.2 × 103 kg m3. Find the speed with respect to spring with which a wave pulse can proceed on the spring if the elevator accelerates up at the rate of 2.0 ms2. Take g = 10 ms2

(a)   30 m/s

(b)   42 m/s

(c)   46 m/s

(d)   50 m/s

Ans: (d)

27. The lower end of capillary tube is immersed in mercury. The level of mercury in the tube is found to be 2 cm below the outer level. If the same tube is immersed in water, upto what height will the water rise in the capillary?

(a)   5.9

(b)   4.9

(c)   2.9

(d)   1.9

Ans: (c)

28. Find the increase in pressure required to decrease the volume of water sample by 0.01% Bulk modulus of water  = 2.1 × 109 Nm−2.

(a)   4.3 × 104 N/m2

(b)   1.8 × 107 N/m2

(c)   2.1 × 105 N/m2

(d)   3.7 × 104 N/m2

Ans: (c)

29. Water level is maintained in a cylindrical vessel upto a fixed height H. The vessel is kept on a horizontal plane. At what weight above the bottom should a hole be made in the vessel, so that the water stream coming out of the hole strikes the horizontal p lane of the greatest distance from the vessel.

(a) 

(b) 

(c) 

(d) 

Ans: (a)

30. Figure shows spring + block + pulley system which are light. The time period of mass would be

(a) 

(b) 

(c) 

(d) 

Ans: (c)

31. A pendulum having a bob of mass m is hanging in a ship sailing along the equator form east to west. When the strip is stationary with respect to water, the tension in the string is T0. The difference between T0 and earth attraction on the  bob, is

(a) 

(b) 

(c) 

(d) 

Ans: (d)

32. A solid sphere is set into motion on a rough horizontal surface with a linear speed v in the forward direction and an angular speed v/R in the anticlockwise direction as shown in figure. Find the linear speed of the sphere when it stops rotating and 

(a)   3v/5

(b)   2v/5

(c)   4v/3

(d)   7v/3

Ans: (a)

33. Two blocks of mass m1 and m2 are connected by a spring of spring constant k. The block of mass m2 is given a sharp empulse so that it acquires a velocity v0 towards right. Find the maximum elongation that the spring will suffer.

(a) 

(b) 

(c) 

(d) 

Ans: (a)

34. A ball of mass m hits the floor with a speed v making an angle of incidence θ with the normal. The coefficient of restitution is e.

The speed of reflected ball and the angle of reflection of the ball will be

(a) 

(b) 

(c) 

(d) 

Ans: (a)

35. A particle slides on surface of a fixed smooth sphere starting from topmost point. The angle rotated by the radius through the particle,, when it leaves contact with the sphere, is

(a) 

(b) 

(c) 

(d) 

Ans: (b)

36. What is the radius of curvature of the parabola traced out by the projectile in the previous problem at a point where the particle velocity makes and angle θ/2 with the horizontal?

(a) 

(b) 

(c) 

(d) 

Ans: (a)

37. A block of mass 2 kg is pushed against rough vertical wall with a force of 40 N, coefficient of static friction being 0.5. Another horizontal force of 15 N, is applied on the block in a direction parallel to the wall. If the block will move, then its direction would be

(a)   15° with 15 N force

(b)   53° with 15 N force

(c)   45° with 15 N force

(d)   75° with 15 N force

Ans: (b)

38. A block is kept on the floor of an elevator at rest. The elevator starts descending with an acceleration of 12 m/s2. Find the displacement of the block during the first 0.2 s after the start. (Take, g = 10 m/s2)

(a)   30 cm

(b)   zero

(c)   20 cm

(d)   25 cm

Ans: (c)

39. A monkey of mass 15 kg is climbing on a rope with one end fixed to the ceiling. If it wishes to go up with an acceleration 1 m/s2, how much force should it apply to the rope if rope is 5 m long and the monkey starts from rest?

(a)   150 N

(b)   > 160 N

(c)   165 N

(d)   150 < T ≤ 160 N

Ans: (c)

40. A square loop is made y a uniform conductor wire as shown in figure?

The net magnetic field at the centre of the loop if side length of the square is a

(a) 

(b) zero

(c) 

(d) None of these

Ans: (b)

41. The electron of an H-atom is revolving around the nucleus in circular orbit having radius  The current  produced due to the motion of electron is

(a) 

(b) 

(c) 

(d) 

Ans: (d)

42. Two small balls, each carrying a charge q are suspended by equal insulator strings of length l m form the hook of a stand. This arrangement is carried in a satellite in space. The tension in each string will be

(a) 

(b) 

(c) 

(d) 

Ans: (b)

43. A vessel of depth t is half filled with a liquid having refractive index n1 and the other half is filled with water of having refractive index n2. The apparent depth of the vessel as viewed from top is

(a) 

(b) 

(c) 

(d) 

Ans: (c)

44. In photoelectric effect, the number of photo-electrons emitted is proportional to

(a)   velocity of incident beam

(b)   frequency of incident beam

(c)   intensity of incident beam

(d)   work function for cathode material

Ans: (c)

45. A change of 8.0 mA in the emitter current brings a change of 7.9 mA in the collector current. The value of α will be

(a)   0.96

(b)   0.93

(c)   0.90

(d)   0.99

Ans: (d)

46. The half-life of 198Au is 2.7 days. The average life is

(a)   4 days

(b)   3.4 days

(c)   3.9 days

(d)   None of the above

Ans: (c)

47. The de-Broglie wavelength of electron falling on the target in an X-ray tube is λ. The cut-off wavelength of the emitted X-ray is

(a) 

(b) 

(c) 

(d) 

Ans: (c)

48. If MO is the mass of an oxygen isotope 8O17, MP and Mn are the masses of a proton and a neutron, respectively, the nuclear binding energy of the isotope is

(a) 

(b) 

(c) 

(d) 

Ans: (d)

49. A nucleus disintegrates into two nuclear parts which have their velocities in the ratio 2 : 1. The ratio of their nuclear size will be

(a) 

(b) 

(c) 

(d) 

Ans: (d)

50. The given p-V diagram shows gases during adiabatic process. Plots 1 and 2 should correspond respectively to

(a)    He and O2

(b)    O2  and He

(c)    He and Ar

(d)    O2 and N2

Ans: (a)

51. For an adiabatic expansion of a mono atomic perfect gas, the volume increases by 24%. What is the percentage decrease in pressure?

(a)   24%

(b)   40%

(c)   48%

(d)   71%

Ans: (b)

52. A body weighing 8 g when placed in one pan and 18 g when placed on the other pan of a false balance. If the beam is horizontal when both the pans are empty, then the true weight of the body is

(a)   13 g

(b)   9 g

(c)   22 g

(d)   12 g

Ans: (d)

53. A rod PQ of length l is moving with ends remaining in contact with frictionless wall and floor. If at the instant, shown the velocity of end Q is 2 m/s towards negative direction of x. The speed of end P will be

(a) 

(b) 

(c) 

(d) 

Ans: (b)

54. Universal time is based on

(a)   rotational effect of the earth about its axis

(b)   vibrations of cesium atom

(c)   orbital motion of the earth around the sun

(d)   oscillation of quartz crystal

Ans: (a)

55. A slab consists of portions of different materials of same thickness and having the conductivities K1 and K2. The equivalent thermal conductivity of the slab is

(a) 

(b) 

(c) 

(d) 

Ans: (c)

56. Two rigid boxes containing different ideal gases are placed on table. Box A contains one mole of nitrogen at temperature T0, while box B contains 1 mole of helium at temperature 7/3 T0. The boxes are then put into thermal contact with each other and heat flows between them until the gases reach a common final temperature (ignore the heat capacity of boxes) then the final temperature of gases, Tf in terms of T0 is

(a) 

(b) 

(c) 

(d) 

Ans: (c)

57. The temperature of the cold junction of thermocouple is 0℃ and the temperature of hot junction is T℃. The emf is E = 16T – 0.04 T2 μ The inversion temperature Ti is

(a)   300℃

(b)   200℃

(c)   500℃

(d)   400℃

Ans: (d)

58. The galvanometer resistance is 30Ω and it is connected to 2 V battery along with a resistance 2000 Ω in series. A full scale deflection of 25 divisions is obtained. In order to reduce this deflection to 20 divisions, the resistance in series should be

(a)   2470 Ω

(b)   2320 Ω

(c)   2180 Ω

(d)   2210 Ω

Ans: (a)

59. A thin bar magnet of length 2 L is bent at the mid-point so that the angle between them  is 60°. The new length of the magnet is

(a) 

(b) 

(c) 

(d) 

Ans: (c)

60. The magnetic flux through each turn of a coil having 200 turns is given as (t2 – 2t) × 103 Wb, where t is in second. The emf induced in the coil at t = 3 s is

(a)   0.7 V

(b)   1.2 V

(c)   0.8 V

(d)   0.9 V

Ans: (c)

Chemistry

1. Given that the reduced temperature, 

the reduced pressure, 

the reduced volume, 

Thus, it can be said that the reduced equation of state may be given as

(a) 

(b) 

(c) 

(d) 

Ans: (d)

2. The suitable reaction steps to carry out the following transformation is

(a) 

(b)  

(c)  

(d) 

Ans: (c)

3. 

X + H2S → Z

Here, Z is

(a)   SO3

(b)   colloidal sulphur

(c)   gaseous sulphur

(d)   solid sulphur

Ans: (b)

4. For preparing 3.00 L of 1 M NaOH by mixing portions of two stock solutions (A and B) of 2.50 M NaOH and 0.40 M NaOH respectively. Find out the amount of B stock solution (in L) added.

(a)   8.57 L

(b)   2.14 L

(c)   1.28 L

(d)   7.51 L

Ans: (b)

5. Sodium sulphite is used in preserving squashes and other mildly acidic foods due to

(a)   potassium salt has preservative action

(b)   potassium metabisulphite prevents oxidation

(c)   potassium metabisulphite is not influcenced by acid

(d)   sulphur dioxide and sulphurous acid formed kill bacteria and germs

Ans: (d)

6. The Vividh Bharti Station of All India Radio, Delhi, broadcasts on a frequency of 1,368 kHz (Kilohertz). Calculate the wavelength (λ) of the electromagnetic radiation emitted by transmitter. Which part of the electromagnetic spectrum does it belong to

(a)   319.4 m and X-rays

(b)   319.4 m and raiowave

(c)   219.3 m and microwave

(d)   219.3 m and radiowave

Ans: (d)

7. Which of the following soap/detergent is least, reduce space biodegradable?

(a) 

(b) 

 

(c)  

(d) 

Ans: (c)

8. In an atom, an electron is moving with a speed of 600 m/s with an accuracy of 0.005%. Certainty with which the position of the electron can be located is

(Given, h = 6.6 × 10−34 kg m2 s−1, mass of electron em = 9.1 × 10−31 kg)

(a)    2.15 × 103 m

(b)    2.78 × 103 m

(c)    1.92 × 103 m

(d)    3.24 × 103 m

Ans: (c)

9. Buna-N, a synthetic rubber is copolymer of

(a) 

(b)  

(c)  

(d)  

Ans: (b)

10. What would be the heat released when an aqueous solution containing 0.5 mole of HNO3 is mixed with 0.3 mole of OH?

(enthalpy of neutralization is −57.1 kJ)

(a)   28.5 kJ

(b)   17.1 kJ

(c)   45.7 kJ

(d)   1.7 kJ

Ans: (b)

11. The number average molar mass and mass average molar mass of a polymer are respectively 30,000 and 40,000. The polydispersity index (PDI) of the polymer is

(a)   −1

(b)   0

(c)   > 1

(d)   < 1

Ans: (c)

12. The charge/size ratio of a cation determines its polarizing power. Which one of the following sequences represents the increasing order of the polarizing power of the cationic species, K+, Ca2+, Mg2+, Be2+?

(a)

(b)

(c) 

(d)

Ans: (a)

13. In an amino acids, the carboxyl group ionizes at and ammonium ion at  The isoelectric point of the amino acid is at pH

(a)   4.32

(b)   3.34

(c)   9.46

(d)   5.97

Ans: (d)

14. 40 g of helium in a bulb at a temperature of T K had a pressure of p atm. When the bulb was immersed in water bath at temperature 50 K more than the first one, 0.08 g of gas had to be removed to restore the original pressure.

(a)   500 K

(b)   400 K

(c)   600 K

(d)   200 K

Ans: (d)

15. What percentage of β-D(+)-glucopyranose is found at equilibrium in the aqueous solution?

(a)   64%

(b)   36%

(c)   ≈100%

(d)   ≈50%

Ans: (a)

16. Which of the following is correctly arranged in order of increasing weight?

(a)  0.0105 equivalent of  of Fe < 0.006 g atom of 

(b) 0.625 g of Fe < 0.0105 equivalent of atoms of Zn < 0.006g atom of Ag

(c) 0.625 g of  atoms of Zn < 0.006 g atom of Ag < 0.0105 equivalent of 

(d) 0.0105 equivalent of  atom of atoms of Zn < 0.625 g of Fe

Ans: (c)

17. The correct order of basic strength of the following are

(a)   I > II > IV > III

(b)   IV > III > II > I

(c)   II > III > IV > I

(d)   III > IV > II > I

Ans: (d)

18. If for a given substance, melting point is TB and freezing points is TA then correct variation of entropy by graph between entropy change and temperature is

(a) 

(b) 

(c) 

(d) 

Ans: (b)

19. When a mixture of 1-hexanol and hexanoic acid in diethyl ether is shaken with an aqueous NaHCO3 solution, then which of the following is right distribution?

(a)   In ether-Sodium hexanoate, In sodium bicarbonate solution-1hexanol

(b)   In ether-1-hexanol, In sodium bicarbonate solution-Hexanoic acid

(c)   In ether-1-hexanol, In sodium bicarbonate solution-Sodium hexanoate

(d)   In ether-Sodium hexanoate, In sodium bicarbonate solution-1hexanol

Ans: (c)

20. ∆H and ∆S for the reaction,  are 30.56 kJ mol−1 and 66.00 JK−1 mol−1 respectively. The temperature at which the free energy change for the reaction will be zero, is

(a)   3528 K

(b)   463 K

(c)   73 K

(d)   144 K

Ans: (b)

21. 

The major product P will be

(a) 

(b) 

(c) 

(d) 

Ans: (c)

22. If Ag+ + 2NH3 ⇌ Ag (NH3)2+; K1 = 1.7 × 107

Ag+ + Cl ⇌ AgCl; K2 = 5.4 × 109

Then, for AgCl + 2NH3 ⇌ [Ag(NH3)2]+ + Cl equilibrium constant will be

(a)    4.68 × 103

(b)    5.2 × 1017

(c)    0.31 × 102

(d)    3.1 × 102

Ans: (c)

23. What is product of the following sequence of reactions?

(a) 

(b) 

(c) 

(d) 

Ans: (d)

24. For the following equilibrium (omitting charges)

I. M + Cl → MCl, Keq = β1

II. MCl + Cl → MCl2, Keq = β2

III. MCl2 + Cl → MCl3, Keq = β3

IV. M + 3Cl → MCl3, Keq = K

then relationship between K, β1, β2 and β3 is

(a)  

(b)   

(c)   

(d)   

Ans: (d)

25. R―CH2―CH2―OH can be converted into RCH2CH2COOH by the following sequence of steps..

(a)  

(b)   

(c)  

(d)  

Ans: (b)

26. When KMnO4 acts as an oxidizing agent and ultimately forms MnO42, MnO2, Mn2O3 and Mn2+, then the number of electrons transferred in each case respectively are

(a)   1, 3, 4, 5

(b)   3, 2, 1, 4

(c)   1, 5, 3, 7

(d)   4, 3, 2, 1

Ans: (a)

27. The product P of the given reaction is

(a) 

(b) 

(c) 

(d) 

Ans: (d)

28. In acidic medium, dichromatic ion oxidizes ferrous ion to ‘ferric ion. If the gram molecular weight of potassium dichromate is 294 g, its gram equivalent weight (in grams) is

(a)   24.5

(b)   49

(c)   125

(d)   250

Ans: (b)

29. Find out the correct stereoisomeric product for the following reaction,

(a)   d-form

(b)   I-from

(c)   meso-form

(d)   racemic mixture

Ans: (d)

30. Ferrous oxide has a cubic structure. The length of edge of the unit cell is 5 Å and the density of the oxide is 4.0 g cm−3. Then the number of Fe2+ and O2 ions present in each unit cell will be

(a)

(b)  

(c) 

(d) 

Ans: (b)

31. In the reaction  

(a)   o-bromotoluene

(b)   m-bromotoluene

(c)   p-bromotoluene

(d)   3-bromo-2, 2, 6-trichlorotoluene

Ans: (b)

 

32. Which of the following arrangements correctly represents hexagonal and cubic close packed structure respectively?

(a)   ABAB …………. and ACBACB ………….

(b)   ABCABC ……….. and ABAB ………

(c)   Both have ABCABC …….. arrangement

(d)   Both have ABAB ………… arrangement

Ans: (a)

33. 0.001 mole of [Co(NH3)5(NO3)(SO4)] was passed through a cation exchanger and the acid coming out of it required 20 mL of 0.1 M NaOH for neutralization. Thus, the complex is

(a)  

(b)  

(c)  

(d)

Ans: (a)

34. The molal freezing point depression constant for benzene (C6H6) is 4.90 K kg mol1. Selenium exists as a polymer of the type Sex. When 3.26 g of selenium is dissolved in 226g of benzene, the observed freezing point is 0.112℃ lower than that of pure benzene. The molecular formula of selenium is

(atomic mass of Se = 78.8 g mol−1)

(a)    Se8

(b)    Se6

(c)    Se4

(d)    Se2

Ans: (a)

35. In the complexes [Fe(H2O)6]3+ , Fe(CN)6]3, [Fe(C2O4)3]3 and [FeCl6]3, mor stability is shown by

(a)  

(b)  

(c)  

(d) 

Ans: (b)

36. For an ideal binary liquid solution with  in which relation between Xx( mole fraction of X in liquid phase) and Yx (mole fraction of X in vapour phase) is correct, Xy and Yy are mole fraction of Y in liquid and vapour phase respectively

(a)

(b) 

(c) 

(d) 

Ans: (c)

37. Point out the incorrect reaction from the following.

(a) 

(b) 

(c)  

(d)  

Ans: (b)

38. 

 

X, Y and Z respectively are

(a) 

 

(b) 

(c) 

(d) 

 

 

Ans: (a)

39. A student made the following observation in the laboratory.

I. Clean copper metal did not react with 1 molar Pb(NO3)2 solution

II. Clean lead metal dissolved in a 1 molar AgNO3 solution and crystals of Ag metal appeared

III. Clean silver metal did not react with 1 molar Cu(NO3)2 solution

The order of decrease in reducing character of three metals is

(a)   Pb, Cu, Ag

(b)   Pb, Ag, Cu

(c)   Cu, Ag, Pb

(d)   Cu, Pb, Ag

Ans: (a)

40. Choose the correct alkyne and reagents for the preparation of

(a) 

(b) 

(c) 

(d) 

Ans: (a)

41. The conductivity of 0.001028 mol L−1 acetic acid is 4.95 × 10−5 S cm−1. Find out its dissociation constant if ⋀m for acetic acid is 390.5 S cm−1 mol−1.

(a)  

(b)  

(c)  

(d)  

Ans: (b)

42. In order to prepare one litre 1N solution of KMnO4, how many grams of KMnO4 are required, if the solution to be used in acid medium for oxidation?

(a)   128 g

(b)   41.75 g

(c)   31.60 g

(d)   62.34 g

Ans: (c)

43. Which of the following represents the correct order of decreasing number of S = O bonds?

(a) 

(b) 

(c)

(d) 

Ans: (c)

44. A hypothetical reaction, X2 + Y2 → 2 XY follows the following mechanism

X2 ⇌ X + X ….. fast

X + Y2 → XY + Y …….. slow

X+ Y → XY ……… fast

The order of the overall reaction is

(a)   2

(b)   3/2

(c)   1

(d)   0

Ans: (b)

45. The major role of fluorspar (CaF2) which is added in small quantity in the electrolytic reduction of alumina dissolved in fused cryolite (Na3AlF6) is

I. as a catalyst.

II. to make the fused mixture very conducting.

III. to lower the temperature of melting.

IV. to decrease the rate of oxidation of carbon at the anode.

(a)   I, II

(b)   II, III

(c)   I, II, III

(d)   III, IV

Ans: (b)

46. The variation of concentration of the product P with time in the reaction, A → P is shown in following graph.

The graph between  and time will be of the type

(a) 

(b) 

(c) 

(d) 

Ans: (a)

47. Point out the correct statement.

(a)   Below 710℃, C is better reducing agent than CO

(b)   Below 710℃, CO is better reducing agent than C

(c)   Below 710℃, CO is an oxidizing agent

(d)   Below 710℃, CO2 is a reducing agent

Ans: (b)

48. Which of the following represents physical adsorption?

(a) 

(b) 

(c) 

(d) 

Ans: (d)

49. KMnO4 reacts with ferrous sulphate according to the following equation,

MnO4 + 5Fe2+ + 8H → Mn2+ + 2Fe3+ + 4H2O

Here, 10 mL of 0.1 M KMnO4 is equivalent to

(a)   50 mL of 0.1 M

(b)   20 mL of 0.1 M

(c)   40 mL of 0.1 M

(d)   30 mL of 0.1 M

Ans: (a)

50. In an experiment, addition of 4.0 mL of 0.005 M BaCl2 to 16.0 mL of arsenious sulphide sol just causes complete coagulation in 2 h. The flocculating value of the effective ion is

(a) 

(b) 

(c) 

(d) 

Ans: (a)

51. Which of the following atomic and physical properties of hydrogen is false?

(a)   Hydrogen > Deuterium > Tritium; (melting point/K)

(b)   Hydrogen < Deuterium < Tritium; (boiling point/K)

(c)   Hydrogen < Deuterium < Tritium;  (density/gL−1)

(d)   Hydrogen > Deuterium > Tritium; (% relative abundance)

Ans: (a)

52. Eutrophication of a lake means, it

(a)   is low is nutrients

(b)   is high in nutrients

(c)   has a high temperature

(d)   has excess amount of organic matter

Ans: (b)

53. When H2O2 is added to ice cold solution of acidified potassium dichromate in ether and the contents are shaken and allowed to stand

(a)   a blue colour is obtained in ether because of formation of CrO5

(b)   a blue colour is obtained in ether because of formation of CrO3

(c)   a blue colour is obtained in ether because of formation of Cr2(SO4)3

(d)   chromyl chloride is formed

Ans: (a)

54. 500 mL of a sample of water required 19.6 mg of K2Cr2O7 for the oxidation of dissolved organic matter in it in the presence of H2SO4. The COD of water sample is

(a)   3.2 ppm

(b)   7.2 ppm

(c)   6.4 ppm

(d)   4.6 ppm

Ans: (c)

55. 

The chemical formulae of X, Y and Z are

Ans: (d)

56. For carbanion

Stability order will be

(a)   P > Q > S > R

(b)   P > R > Q > S

(c)   Q > R > S > R

(d)   S > R > Q > P

Ans: (b)

57. The aqueous solution of an unknown sodium salt gives the following reactions.

I. It decolourises a solution of iodine in potassium iodide.

II. It gives white turbidity with dil. HCl solution.

III. It gives a white precipitate with AgNO3 solution which changes colours and finally becomes black on standing.

The unknown sodium salt is

(a)   sodium thiosulphate

(b)   sodium bisulphite

(c)   sodium sulphite

(d)   sodium sulphide

Ans: (a)

58. The catenation tendency of C, Si and Ge is in the order Ge < Si < C. The bond energies (in kJ mol1) of C―C, Si―Si and Ge―Ge bonds, respectively are

(a)   348, 167, 180

(b)   348, 180, 167

(c)   167, 180, 348

(d)   180, 167, 348

Ans: (b)

59. What is correct about the following structure?

(a)   Total stereoisomers = 4

(b)   Number of chiral carbons = 1

(c)   Number of optical isomers = 2

(d)   Number of meso compounds = 2

Ans: (c)

60. Number of oxygen atoms shared per SiO44 tetrahedron in

I. two dimensional sheet structured silicates

II. cyclic silicates and

III. single strand chain silicates respectively are

(a)   3, 3, 2

(b)   3, 2, 2

(c)   4, 3, 1

(d)   4, 3, 2

Ans: (b)

Biology

1. Which of the following is the most primitive ancestor of man?

(a)   Ramapithecus

(b)   Homo habilis

(c)   Australopithecus

(d)   Homo sapiens neanderthalensis

Ans: (a)

2. Vitamin-B12 is absorbed primarily in the

(a)   stomach

(b)   duodenum

(c)   jejunum

(d)   ileum

Ans: (d)

3. The faster breathing in high fever is due to the

(a)   additional requirement of O2 for the invader germs

(b)   high temperature of the body

(c)   mental worry of a patient

(d)   loss of appetiate

Ans: (b)

4. Which of the following animals possess non-elastic lungs with elastic air sacs connected to them?

(a)   Reptiles

(b)   Birds

(c)   Amphibians

(d)   Mammals

Ans: (b)

5. Which of the following is not correctly matched?

(a)   Trichomonas vaginalis – Leishmaniasis

(b)   Glossina palpalis – Sleeping sickness

(c)   Aedes agypti – Yellow fever

(d)   Culex pipceins – Filariasis

Ans: (a)

6. Carotid labyrinth contains

(a)   olfactory receptors

(b)   baroreceptors

(c)   chemorreceptors

(d)   phonoreceptors

Ans: (b)

7. Occupational lung disease that occurs in humans, among those given below is

(a)   dyspnea

(b)   anthracosis

(c)   atelectasis

(d)   cyanosis

Ans: (b)

8. Epiphyseal plates at the extremities of long bones help in

(a)   bone moulding

(b)   elongation of bone

(c)   bone formation

(d)   formation of Haversian canal

Ans: (b)

9. The strongest muscle in the human is

(a)   biceps

(b)   gluteus maximus

(c)   stapedius

(d)   masseter

Ans: (d)

10. Parkinsonism is related with

(a)   brain

(b)   spinal nerve

(c)   cranial nerves

(d)   All of these

Ans: (a)

11. Meniere’s disease is associated with

(a)   ear

(b)   eye

(c)   nose

(d)   throat

Ans: (a)

12. If the threshold for hearing increasing 1000 times, the hearing loss is

(a)   40 decibels

(b)   50 decibels

(c)   60 decibels

(d)   None of these

Ans: (d)

13. Gonads are derived from

(a)   ectoderm

(b)   mesoderm

(c)   endoderm

(d)   None of the above

Ans: (b)

14. The number of chromosomes in a primary spermatocyte is

(a)   same as in spermatid

(b)   same as in spermatogonium

(c)   help of that in spermatogonium

(d)   same as in secondary spermatocyte

Ans: (b)

15. The portion of the endometrium that covers the embryo and is located between the embryo and the uterine cavity is the

(a)   decidua basalis

(b)   desidua umbilicus

(c)   desidua capsularis

(d)   deciduas functionalis

Ans: (c)

16. Depo-provera refers to

(a)   injectible contraceptive

(b)   intra uterine device

(c)   implant

(d)   oral contraceptive

Ans: (a)

17. What base is responsible for hot spots for spontaneous pint mutations?

(a)   Adenine

(b)   Guanine

(c)   5-bromouracil

(d)   5-methyl cytosine

Ans: (c)

18. During which geological period of evolution did the greatest diversification of life occurred on the earth?

(a)   Permian

(b)   Jurassic

(c)   Cambrian

(d)   Ordovician

Ans: (c)

19. Specific radioactive identification of ribosomal RNA can be achieved by using C14 labelled

(a)   guanine

(b)   uracil

(c)   thymine

(d)   cytocine

Ans: (b)

20. Which of the following is the Pribnow box?

(a)   5′-TATAAT-3′

(b)   5′-TAATAT-3′

(c)   5′-AATAAT-3′

(d)   5′-ATATTA-3′

Ans: (a)

21. The genome of Caenorhabditis elegans consists of

(a)   3 billion base pairs and 30,000 genes

(b)   12 million base pairs and 6000 genes

(c)   4.7 million base pairs and 4000 genes

(d)   97 million base pairs and 18,000 genes

Ans: (b)

22. About how long ago was the earth formed?

(a)   3 billion years ago

(b)   10 billion years ago

(c)   4.6 billion years ago

(d)   20 billion years ago

Ans: (c)

23. Gas gangrene is caused by

(a)   Clostridium botulinum

(b)   Xanthomonas campestris

(c)   Pseudomonas

(d)   Clostridium perfringens

Ans: (d)

24. Who received Nobel Prize in 1951 for the development of vaccine for yellow fever?

(a)   Max Theiler

(b)   Ronald Ross

(c)   Max Delbruck

(d)   Francis Peyton Rous

Ans: (a)

25. Continuous exposer to vinyl chloride may cause cancer of the

(a)   liver

(b)   spleen

(c)   vagina

(d)   prostate gland

Ans: (a)

26. Which of the following T-cells and destroyed by HIV?

(a)   Cytotoxic T-cells

(b)   Killer T-cells

(c)   Suppressor T-cells

(d)   Helper T-cells

Ans: (d)

27. An autoimmune disease is

(a)   AIDS

(b)   haemophilia

(c)   allergy

(d)   myasthenia gravis

Ans: (d)

28. Which hormone produces calorigenic effect?

(a)   Thyroxine

(b)   FSH

(c)   Insulin

(d)   All of these

Ans: (a)

29. Which of the following act as an antigen, but do not induce antibody production?

(a)   Haustra

(b)   Histones

(c)   Haptens

(d)   None of these

Ans: (c)

30. Haldane effect refers to

(a)   more acidity in the blood

(b)   less acidity in the blood

(c)   more basicity in the blood

(d)   less basicity in the blood

Ans: (a)

31. The plant whose seeds are known to leave the longest viability period is

(a)   Carica papaya

(b)   Triticum aestivum

(c)   Zizypus mauritiana

(d)   Nelumbo nucifera

Ans: (d)

32. Jut is a

(a)   bast fibre from secondary xylem

(b)   bast fibre from primary xylem

(c)   bast fibre from secondary pholem

(d)   bast firbe from primary pholem

Ans: (c)

33. Chlorosis is produced in the leaves due to the deficiency of Fe, Mg, Mn, S or N of these essential elements, those that are exclusive constituents of chlorophyll molecule are

(a)   Fe, S

(b)   N, S

(c)   Mg, S

(d)   Mg, N

Ans: (d)

34. Read the following statements and choose the correct option.

I. Leptosporangiate development of sporangium is found in all members Pteropsida.

II. Seed habit is shown by sellaginella.

III. Gnetum leaves are monomorphic and pinnately compound.

IV. Sporic meiosis is found in Volvolx, Chalmydomonas and Ulothrix.

Choose the correct option.

(a)   I and IV

(b)   I and II

(c)   I, II and IV

(d)   All of these

Ans: (c)

35. Which of the following characteristics out of I, II and III are exhibited by C4 plants?

I. Kranz anatomy.

II. The product of photosynthesis is oxalo acetic acid.

III. Both PEP carboxylase and ribulose-bis phosphate carboxylase act as carboxylating enzyme.

Choose the correct option.

(a)   I and II, but not III

(b)   II and III, but not I

(c)   I and III, but not II

(d)   All of these

Ans: (d)

36. Match the following column I and II for organismic respiration.

(a)   A-2, B-3, C-1

(b)   A-3, B-2, C-1

(c)   A-1, B-3, C-2

(d)   A-3, B-1, C-2

Ans: (b)

37. Compare the statements I and II and choose correct option.

Statement (I) Auxin promote apical dominance by suppressing the activity of lateral  buds.

Statement (II) In moriculture, periodic pruning of shoot tips is done to make mulberry plants bushy.

Choose the correct options

(a)   I is false, but II is true

(b)   II is false, but I is true

(c)   Both I and II are true

(d)   Both I and II are false

Ans: (c)

38. The one advantage of cleistogamy is

(a)   it leads to greater genetic diversity

(b)   seed dispersal is more efficient and wide spread

(c)   seed set is not dependent on pollinators

(d)   each visit of a pollinator results in transfer of hundreds of pollen grains

Ans: (c)

39. In coconut, the liquid endosperm is formed because

(a)   karyokinesis is not followed by cytokinesis

(b)   karyokinesis is followed by cytokinesis

(c)   formation of liquid endosperm is not dependent upon karyokinesis and cytokinesis

(d)   None of the above

Ans: (c)

40. Keeping is view the structure of cell membrane, which one of the following statements is correct with respect to the movements of liquid and proteins from one liquid monolayer to the other (flip flop movement).

(a)   While proteins can flip flop, liquids can not

(b)   Neither lipids, nor proteins can flip flop

(c)   Both lipids and proteins can flip flop

(d)   While lipids can rarely flip flop, proteins can not

Ans: (d)

41. Ribosomes are particles about 200 Å units in diameter consisting of protein and RNA. The percentage of protein and RNA is respectively

(a)   80% and 20%

(b)   60% and 40%

(c)   50% and 50%

(d)   40% and 60%

Ans: (b)

42. Match the biological molecule listed under column I with their biological function listed under column II. Choose the answer which gives correct combination of alphabet of the two columns.

(a)   A-3, B-2, C-4, D-1

(b)   A-4, B-2, C-1, D-3

(c)   A-2, B-4, C-3, D-1

(d)   A-4, B-3, C-1, D-2

Ans: (d)

43. In peal plants, yellow seeds are dominant to green. If a heterozygous yellow seeded plant is crossed with a green seeded plant, what ratio of yellow and green seeded plants could you expect in F1-generation.

(a)   9 : 1

(b)   1 : 3

(c)   3 : 1

(d)   50 : 50

Ans: (d)

44. In a mutational event, when adenine is replaced by guanine, it is a case of

(a)   frame shift mutation

(b)   transcription

(c)   transition

(d)   transversion

Ans: (c)

45. Independent assortment of gene occurs due to the orientation of chromosomes at

(a)   metaphase-I of mitosis

(b)   metaphase-I of meiosis

(c)   metaphase-II of meiosis

(d)   any phase of the cell division

Ans: (b)

46. The non-disjunction, in meiosis may result is extra copy of a chromosome in a sperm cell. During which phase the above phenomenon may occur?

(a)   Prophase-I, prophase-II

(b)   Metaphase-I, anaphase-II

(c)   Anaphase-I, anaphase-II

(d)   Anaphase-I, telophase-II

Ans: (c)

47. Which of the following is true regarding the phase lambda, a virus which infects bacteria?

(a)   In the lytic cycle, the bacterial host replicates viral DNA, passing it on to daughter cells during binary fission

(b)   In the lysogenic cycle, the bacteria host replicates viral DNA, passing it onto daughter cells during binary fission

(c)   In the lytic cycle, viral DNA is integrated into the host genome

(d)   In the lysogenic cycle, the host bacterial cell burst, releasing phases

Ans: (b)

48. The part of the bacterial chromosomes sharing homology with genome fragment transferred from the recipients to cell during emrozygote formation is known as

(a)   eugenic

(b)   exogenate

(c)   endogenate

(d)   dysgenic

Ans: (c)

49. In 1944, Avery, McCarty and MacLeod isolated substance from heat killed virulent form of bacteria and added to non-virulent form of bacteria which changed the non-virulent to virulent from this substance can be destroyed by

(a)   DNAse

(b)   protease

(c)   lipase

(d)   amylase

Ans: (a)

50. Thermococcus, Methanococcus and Methanobacterium are groups of

(a)   bacteria containing a cytoskeleton and all membrane bound organelles

(b)   archaebacteria with peptidoglycan in their cell wall

(c)   archaebacteria that consists of protein homologous to eukaryotic core histones

(d)   most advanced type of bacteria

Ans: (c)

51. Match the following Column I with Column II.

(a)   A-4, B-1, C-3, D-2

(b)   A-1, B-2, C-3, D-4

(c)   A-4, B-1, C-2, D-3

(d)   A-1, B-3, C-2, D-4

Ans: (b)

52. Which of the following sequences represent a possible pathway in the production of a secretory protein?

(a)   Rough ER → Secretory vesicle → Ribosome → Golgi apparatus

(b)   Ribosome → Rough ER → Golgi apparatus → Secretory vesicle

(c)   Secretory vesicle → Golgi apparatus → Ribosomes → Rough ER

(d)   Rough ER → Ribosomes → Secretory vesicles → Golgi appratus

Ans: (d)

53. A connecting link between plant and animal kingdom is

(a)   Paramecium

(b)   Chlamydomonas

(c)   Chlorella

(d)   Euglena

Ans: (d)

54. Which of the following statements is false regarding SDS-polyacrylamide gel electrophoresis?

(a)   Proteins are separated by molecular weight

(b)   SDS is a detergent which gives charge to protein

(c)   Large protein move more slowly through gel

(d)   SDS is used to maintain the 3-dimentional structure of protein

Ans: (d)

55. If the free energy change of a reaction is greater than zero, then the reaction is

(a)   spontaneous

(b)   non-spontaneous

(c)   at equilibrium

(d)   endothermic

Ans: (b)

56. Organisms who are able to freely interbreed producing fertile off-springs and having similar blue print for making these organisms are referred to as

(a)   species

(b)   tribe

(c)   genus

(d)   sub-genus

Ans: (a)

57. The enzyme hexokinase which catalysis glucose to a glucose-6-phosphate in glycolysis is inhibited by glucose-6-phophate. This is an example of

I. Competitive inhibition

II. non-competitive inhibition

III. feedback allosteric inhibition

Which of the above statements is/are correct?

(a)   I and II

(b)   I and III

(c)   Only III

(d)   All of these

Ans: (c)

58. The protein products of tumour suppressor gene may

(a)   be present in non-cancerous cells

(b)   cause signal cell death

(c)   regulate the cell cycle

(d)   All of the above

Ans: (a)

59. The mitotic cell cycle is divided typically into four phases; G1, S, G2 and M. Considering a mitotic cycle time of 18 hrs; the distribution of period of time (in hrs) for each of these phases will be

(a) 

(b) 

(c) 

(d) 

Ans: (c)

60. Match the following column I with column II.

(a)   A-1, B-2, C-4, D-3

(b)   A-3, B-1, C-4, D-2

(c)   A-2, B-3, C-1, D-4

(d)   A-4, B-1, C-3, D-2

Ans: (a)

Quantitative Aptitude & English

1. When the bus reaches Shivani’s house, it faces South. After starting from Shivani’s house to the school, it turns twice to its left and once to its right. In which direction it is running now?

(a)   North

(b)   West

(c)   East

(d)   South

Ans: (c)

2. A men travels 12 km West, then 3 km towards South and then 8 km towards East. How far is he form the start?

(a)   23 km

(b)   20 km

(c)   15 km

(d)   5 km

Ans: (d)

3. From the given alternative words, select the one which can be formed using the letters of the given word EXAMINATION.

(a)   ANIMAL

(b)   EXAMINER

(c)   NATIONAL

(d)   ANIMATION

Ans: (d)

4. If ANCE can b e coded as 3, 7, 29, 11, then BOIL will be code as

(a)   5, 29, 19, 27

(b)   5, 29, 19, 25

(c)   5, 31, 21, 25

(d)   5, 31, 19, 25

Ans: (d)

5. QPRS : TUVW : JIKL : ?

(a)   NMOP

(b)   NMPO

(c)   MNPO

(d)   MNOP

Ans: (c)

Directions (Q. No. 6-7) Find the odd letter pair from

6. (a) Wool   (b) Feather

    (c) Hair     (d) Grass

(a)   a

(b)   b

(c)   c

(d)   d

Ans: (d)

7. (a) Sport : Ground  (b) Cinema : Screen

    (b) Drama : Stage  (d) Rubber : Erase

(a)   a

(b)   b

(c)   c

(d)   d

Ans: (d)

8. If + means, −, − means ×, × means ÷ and ÷ means +, then 48 × 4 ÷ 7 + 8 – 2 = ?

(a)   3

(b)   −5

(c)   35

(d)   16

Ans: (a)

9. If February 1, 2004 is Wednesday, what day is March 3, 2004?

(a)   Monday

(b)   Sunday

(c)   Saturday

(d)   Friday

Ans: (c)

10. How many triangles are there in the given figure?

(a)   5

(b)   4

(c)   3

(d)   8

Ans: (d)

Directions (Q. Nos. 11-12) Out of the four alternatives choose the one which best expresses the meaning of the given word.

11. Instigate

(a)   Initiate

(b)   Incite

(c)   Force

(d)   Cause

Ans: (b)

12. Voracious

(a)   Quick

(b)   Angry

(c)   Hungry

(d)   Wild

Ans: (c)

Directions (Q. No. 13-14) Choose the word opposite in meaning to the given word.

13. Epilogue

(a)   Dialogue

(b)   Prelude

(c)   Post script

(d)   Epigram

Ans: (b)

14. Indiscreet

(a)   Reliable

(b)   Honest

(c)   Prudent

(d)   Stupid

Ans: (c)

Directions (Q. No. 15) Out of the alternatives choose the one which can be substituted for the given words/sentence.

15. The absence of law and order

(a)   Rebellion

(b)   Anarchy

(c)   Mutiny

(d)   Revolt

Ans: (b)

Directions (Q. Nos 16-18) In the following question, a sentence has divided into four parts. Arrange these parts to make the sentence meaningful.

16. In favour of English,

P : has chances of securing employment

Q : we may say that

R : in all p arts of India and in foreign countries

S : an English knowing Indian

(a)   QSPR

(b)   SPQR

(c)   SRQP

(d)   QRPS

Ans: (a)

17. The hungry man

P : and said

Q : replied in the negative

R : that he only wanted a meal

S : to his question

(a)   SQPR

(b)   QSPR

(c)   SPRQ

(d)   QPRS

Ans: (b)

18. It is

P : that people read fewer books today

Q : then they did

R : even about a decade ago

S : a matter of grave concern

(a)   PSRQ

(b)   SPRQ

(c)   PSQR

(d)   SPQR

Ans: (d)

Directions (Q. Nos. 19-20) Groups of four words are given. In each group, one word is correctly spelt. Find the correctly spelt word.

19. (a) Parapharnelia (b) Parsimonious

      (c) Peccadilo        (d) Peadialriis

(a)   a

(b)   b

(c)   c

(d)   d

Ans: (b)

20. (a) Tussel (b) Tunnle

(c) Tumble     (d) Trable

(a)   a

(b)   b

(c)   c

(d)   d

Ans: (c)

JIPMER 2016

JIPMER:

          Just like All India Institute of Medical Science (AIIMS), JIPMER is also declared as an autonomous medical institution by a Indian Parliament act, 2008. JIPMER (Jawaharlal Institute of Postgraduate Medical Education and Research) is a top most medical institute under the Government of India since 1956. This Institute is located at urban areas of Pondicherry which is 170 Kms away from Chennai.

          It is a residential institute which has its own hostels for students, faculties and staff members. It has a wide spread area covering 192 acres in Pondicherry. Before this institution was declared as an Institution of national importance in the year 2008, it was working under Ministry of Health and Family Welfare and Directorate General of Health Service.

Vision and Mission of JIPMER:

           Vision of JIPMER is to be a model for all the medical institutions in India by proving standard innovative education, patient-oriented services and excellence in all service. It has country’s best health care facilities which help all types of people from the poorest to affordable.

           Mission of JIPMER is to encourage young talents in the field of medical and research. To provide best health professionals who can be a dedicative lifelong learners and leaders in Health care profession. Other missions are allowing innovative techniques in the education system, partnerships with the organizations with best ideals and emphasizing quality and value in the education.

About JIPMER Hospital:

           JIPMER is best in providing all types of latest health care facilities and all types of super specialties at an affordable cost to the society.

JIPMER treats all kinds of people right from poorest to most affordable ones.

Thus, JIPMER is considered as the best medical institution for education as well as treatment.

Below is the list of JIPMER’s achievement in the year 2014-2015

No of out-patients treated daily: 20, 70,536

No. of old cases treated: 15, 14,861

No. of new cases treated: 5, 55,675

No. of Admissions: 83,780

No. of discharges: 80,449

No. of deliveries conducted: 15,018

No of live births: 14,472

No. of still births: 546

No of death in a year: 3,267

No. of Autopsies: 898

 

JIPMER Education system:

                 Every year JIPMER admits 145 UG students and 75 PG students. Sources say, it has 3000 administrative support staffs, 200 faculties and 360 physicians within the campus of JIPMER. JIPMER provides UG, PG, Super specialty and PhD programs in various disciplines to help dedicated medical students to undertake medical research for the welfare of the society.

JIPMER provides PG courses (MS/MD) in 22 disciplines such as:

Doctor of Medicine (MD)

  • Microbiology
  • General Medicine
  • Emergency Medicine
  • Forensic medicine
  • Nuclear medicine
  • Physiology
  • Psychiatry
  • Radiotherapy
  • Anatomy
  • Biochemistry
  • Anesthesiology
  • Pediatrics, etc.,

 

Master of Surgery (M.S)

  • Ophthalmology
  • General Surgery
  • Obstetrics and Gynaecology
  • Orthopedic Surgery
  • N.T

It offers Ph.D courses in 7 disciplines such as:

  • Pharmacology
  • Clinical Immunology
  • Microbiology
  • Biochemistry
  • Physiology
  • Pathology
  • Medicine
  • Physiology

 

It offers 8 super specialty programs such as:

  • M (Neurology)
  • M (Cardiology)
  • M (Nephrology)
  • M (Neonatology)
  • M ( Medical Oncology)
  • M (Endocrinology)
  • M (Clinical Pharmacology)
  • M (Clinical Hematology)
  • M (Pediatric critical care)

 

           Along with this, JIPMER is conducting M.Sc (Medical Biochemistry) for the past 32 years. Also, JIPMER conducts courses such as M.R.O (Medical Record Officer) and M.R.T (Medical Record Technician) for admission in the Department of Medical Records.

JIPMER MD/MS 2016 Exam:

             JIPMER MD/MS exam for 2016 academic year has been officially announced. You can find the application form for JIPMER MD/MS 2016 exam in the below link: http://jipmer.edu.in/

         

            JIPMER has become one of the top five medical institutions in India to provide excellent education and standardized treatment for the patients. Thus, it welcomes students who are really dedicated and talented in the field of medicine.

 

 

JIPMER Entrance Exam Preparation Tips

JIPMER Exam Preparation:

                         JIPMER (Jawaharlal Institute of Postgraduate Medical Education and Research) is a top most medical institution which was established under the Government of India since 1956. JIPMER conducts medical entrance test for the admission to the MBBS, PG, MD courses in JIPMER medical institute, Pondicherry. This exam is very important medical entrance exam for the aspirants who wish to pursue MBBS degree in India. This is because JIPMER stands in the fourth position among top most medical institutes of India next to AIIMS, CMC and AFMC. Students from JIPMER are recognized with high reputation all over the world.

JIPMER examination:

                   JIPMER allocates around 150 MBBS seats through JIPMER examination. Overseas Citizen of India (OCI) can also apply for this examination.

Advantages of taking part in JIPMER medical exam are:

  • Level of difficulty of the paper is easier when compared to the other medical entrance examinations such as AIIMS, AIPMT, etc.,
  • No negative mark is applicable for incorrect attempts.
  • Pursuing MBBS degree in JIPMER gives high reputation from all over the world. Thus, starting medicine career after your MBBS becomes easier if you pursue MBBS from JIPMER.

Preparation methods for JIPMER:

               Before getting into how to prepare for JIPMER, let’s check out the pattern for the examination. Knowing this is very important for a candidate to crack this exam successfully.

Total duration of this exam: 2 hours and 30 minutes

Total number of questions: 200

Types of questions:

                Only multiple choice of questions in which the candidate have to select the right one.

Language of the question booklet: English

Subjects covered for this exam:

                       Physics: 60 questions

                       Chemistry: 60 questions

                       Biology: 60 questions

                       English and comprehension: 10 questions

                       Logical reasoning: 10 questions

No negative marks for any incorrect attempts. Each correct attempt carries 1 mark.

Now let’s check out the special ways to prepare for JIPMER:

  • Start to prepare seriously from NCERT books. Every medical aspirant must be thorough in the syllabus of JIPMER. NCERT books cover most of syllabus of JIPMER.
  • Practice a lot with previous years question papers. Solving all the previous year question paper will really help you a lot to crack JIPMER with high scores.
  • Derive a self study plan strategy according to your capacity. Time management is very important.
  • Try to attend every possible question since there is no negative marking scheme.
  • Concentrate well on English and Logical reasoning. This is easier section when compared to other section. Why lose marks in easy sections?
  • Try to study every subject on a logical basis than theoretically. This means, understand the core concept of the topic and try to solve different types of questions from the same topic. This will really help you a lot to tackle MCQ (Multiple choice of Questions) very easily.
  • Join any best coaching institute which can help you to prepare according to your strengths.
  • Prepare special hints or notes for “Last minute preparations”. But however, remember never make yourself excited during the last minute of the exam.
  • First, try to solve all the known questions in the exam. Then, go for the unknown ones.
  • The last thing and very important thing is, Keep yourself cool and relaxed. Never get anxious about the exam.

 

Best coaching institutes for JIPMER:

                 Let’s check out the best coaching centers in India for JIPMER:

  1. DAMS, Bangalore:

           A Delhi Academy of Medical science (DAMS) is a well-established coaching institute for medical coaching for more than 16 years. DAMS are the top most center for medical entrance coaching in India and a highly recommended coaching institute by the toppers of JIPMER.

  1. TIME coaching centre:

             This is also a best institute in providing coaching for JIPMER. It has branches in many

cities of Tamil Nadu and also in Puducherry.

  1. Aakash Institute:

           This is also one of the best coaching institutes for medical entrance examination. Students are admitted for this coaching class only according to marks secured in the 12th board examination.

  1. Career-Point:

           This is a coaching institute which offers class room coaching as well as distance learning facilities for preparation of the examinations like JIPMER, AIMPT, AIIMS, etc.

 

Books to refer:

       Following are the list of books referred by the toppers of JIPMER:

  • Sanjeev Gupta JNP Objective Physics
  • Pradeep Objective Chemistry
  • Pramod Agarwal’s Objective Physics
  • NCERT Biology volume 1 and volume 2
  • U.Charaya Pragati mega study package Objective Botany
  • S.P.Kurl Objective Subjective Zoology
  • NCERT Physics volume 1 and volume 2
  • Agarwal’s Concept of competition Physics

 

           Thus, follow the above instructions sincerely. Be dedicated to your work. Prepare a time table for yourself and count down the number of days for the upcoming JIPMER examination. Concentrate more on multiple choice or objective questions in all subjects. Try to rate yourself by attending mock test through the internet.

Surely you can win this JIPMER examination.

 

 

JIPMER 2016 Exam Pattern

JIPMER Exam pattern:

             JIPMER is a most important medical entrance examination which is conducted on June month of every year. It has equal importance to other important medical exams such as AIIMS, AIPMT, etc. JIPMER 2015 was conducted on June 2, 2015. Results were declared on June 12, 2015. Merit list of students are released in the official website of JIPMER.

          These merit lists are calculated according to the student’s percentile scored in the JIPMER examination. Every year, the number of students participating in this exam is increasing drastically. Thus, competition becomes heavier year by year. So, students who are going to take part in this exam must be thorough in the pattern and syllabus of the JIPMER examination.

JIPMER Pattern:

               Pattern for any exam is very important since it helps in testing the capabilities of student in all possible subjects. A question pattern checks the student’s strengths and weakness in every subject and gives a clear idea about the student’s ability to the examination coordinators. Thus, a student must surely know the pattern thoroughly before start preparing for any kind of exam.

Below is the pattern for the JIPMER exam:

Total duration of the exam: 150 minutes (Two and half hour)

Total number of questions: 200

Types of question: Only multiple choice or objective type of question.

Language of the question booklet: Only English

Subject wise questions:

            Physics: 60 questions each carrying 1 mark

              Chemistry: 60 questions each carrying 1 mark

             Biology: 60 questions each carrying 1 mark

             English: 10 questions each carrying 1 mark

             Logical reasoning: 10 questions each carrying 1 mark

No negative marks are applicable for any incorrect attempt.

Total number of marks: 200

Exam will be conducted during the morning session from 10.00 A.M to 12.30 P.M.

 

Mode of the examination:

            This exam is conducted only in online mode. It is a computer based test. Syllabus of JIPMER will be covered from class XI and class XII (State board, CBSE board) science subjects. Detailed list of the syllabus covered for JIPMER is as below:

http://recruitmentresult.com/jipmer-syllabus/

 

Test centers for JIPMER:

           JIPMER is conducted in seven cities in Northern India. They are:

  1. Lucknow
  2. Meerut
  3. Delhi
  4. Faridabad
  5. Ghaziabad
  6. Greater Noida
  7. Gurgoan

In southern region JIPMER is conducted in 17 cities such as:

  1. Chennai
  2. Cochin
  3. Bengaluru
  4. Coimbatore
  5. Hyderabad
  6. Namakkal
  7. Nagercoil
  8. Mangalore
  9. Mysore
  10. Pondicherry
  11. Salem
  12. Thirunelveli
  13. Trichy
  14. Trivandrum
  15. Vishakapatnam
  16. Vijayawada
  17. Thiruvannamalai

 

JIPMER is also conducted in eight cities of Eastern zone and four cities of western zone in India.

 

Important dates for JIPMER MBBS 2016: (Tentative)

Online registration of JIPMER starts from: 11th March 2016

Closing date for online registration: 02 May 2016

JIPMER 2016 exam is to be conducted on: June 8th, 2016

However, please wait for the official announcement of JIPMER MBBS 2016 exam from the site of JIPMER. On the whole, students who are going to start prepare for JIPMER exam must be clear about the pattern of the exam. Knowing the pattern very well will help you to concentrate more on the subjects which weighs high marks. Thus, it makes you to get through JIPMER easily.

 

 

JIPMER 2016 Entrance Exam

JIPMER Entrance Test (MBBS/UG):

           JIPMER is a national level medical entrance test carried out by Jawaharlal Institute of Post graduate Medical Education and Research. This exam is conducted every year in the month of June for filling 150 MBBS seats in JIPMER Pondicherry.

JIPMER Seat allocation:

     JIPMER seat allocation for the year 2015 is:

Number of MBBS seats for General category: 50

Number of MBBS seats for OBC category: 28

Number of MBBS seats for SC category: 16

Number of MBBS seats for ST category: 11

Remaining seats are for Pondicherry residents and NRI quota.

JIPMER 2014 seat allocation is similar to that of JIPMER 2015.

 

Eligibility criteria:

  • The candidate must be completed 17 years as on 31st December 2016.
  • Candidate must be Indian.
  • Candidate must have secured 50% in all science subjects such as Physics, Chemistry, Biology or Biochemistry and English in 12th Board examination from a recognized University or College.
  • Candidate who is currently pursuing 12th standard can also apply for this exam. But however, he has to submit all his original certificates during the time of admission.

Pattern and syllabus of JIPMER:

Total number of questions: 200

Types of question: Multiple choice objective questions

Marking scheme:

          Each correct attempt carries one mark. No negative mark is applicable. No Mark is considered for “no answer” or “multiple answers”.

Duration of the examination: 150 minutes

Subject wise questions:

                           English and comprehension: 10

                           Physics: 60

                         Chemistry: 60

                           Biology/Biochemistry: 60

                           Logic and quantitative questions: 10

 

How to apply for JIPMER?

               Those who wish to apply for JIPMER please check the below details:

  • Fees for the application form (General Candidates) : Rs.1000/-
  • Fees for the application form (reserved category): Rs.880/-
  • Candidates can apply for this exam only through online mode.
  • The application fees can be paid either by debit card, credit card or e-challan.
  • Please fill in the required details of the candidates very carefully. These details are reflected in your admit card.
  • Click on submit button.
  • Check the status of your application form every time in the official website of JIPMER.

 

JIPMER official contact address:

JIPMER,

Dhanvantri Nagar,

Gorimedu,

Puducherry-605006

Official website is: http://jipmer.edu.in

 

Important dates for JIPMER 2016: (Tentative)

Applications availability dates: Between first week of March 2016 to first week of May 2016.

Candidates have to apply only through online mode for this exam.

Last date of the registration: First week of May 2016

Admit cards available from: Second week of May 2016

Examination to be held on: June 2016 (Tentative by now)

 

Admit card for JIPMER 2016:

           Admit card for JIPMER 2016 will be available only before a month of the examination. Admit card may contain all the important details of the student taking part in the examination. Informations such as Name, Choice of examination city, registration number, application number, date of birth and so on will be included in the admit card. This card is a very important document without which a student is not allowed inside the examination hall.

           Admit card is a main document which must be submitted during the counselling or admission process. Thus, students must keep it safely even after the exam gets over. You can download and take multiple copies of admit card when it is released on the official site prior the examination. Along with this admit card, students please note to take your stamp size and passport size photographs always with you.

 

Result declaration:

           JIPMER 2016 exam is to be held on first week of June 2016. Results of JIPMER 2016 are expected to be declared on second week of June 2016. But, however, keep checking the official site of JIPMER for any official information. Result declaration will have candidates name, roll number, total marks in all subjects and subject wise marks in Physics, Chemistry, Biology, English and Logical reasoning.

 

JIPMER 2015 Result analysis:

                   JIPMER 2015 was conducted on June 7, 2015. Sources say, total number of students who participated in this exam are 1, 37,000. This exam was taken place in 75 cities with 581 exam centres across India. Students felt that the examination level was moderate to tough. Physics were tough for few whereas English comprehension and reasoning part was tricky for other students. Biology was easiest compared to other sections.

Counselling:

            As soon as the results are announced, merit list of the students are prepared and displayed in the official website of JIPMER. Selected students will get a call from the JIPMER to attend the counselling session. Student must be present at the respected venue during the time of counselling to get admission in their desired course in JIPMER.

           JIPMER stands fourth position in the top most five institutions of India next to AIIMS (Delhi), CMC (Vellore) and AFMC (Pune). Just like other prestigious institutes, it is great to get admission in JIPMER medical institute. So, students try to prepare well for this exam. All the best to crack this JIPMER.

 

 

© Copyright Entrance India - Engineering and Medical Entrance Exams in India | Website Maintained by Firewall Firm - IT Monteur